On the destruction of America’s best high school

[C]hildren with special abilities and skills need to be nourished and encouraged. They are a national treasure. Challenging programs for the “gifted” are sometimes decried as “elitism.” Why aren’t intensive practice sessions for varsity football, baseball, and basketball players and interschool competition deemed elitism? After all, only the most gifted athletes participate. There is a self-defeating double standard at work here, nationwide.
—Carl Sagan, The Demon-Haunted World (1996)

I’d like you to feel about the impending destruction of Virginia’s Thomas Jefferson High School for Science and Technology, the same way you might’ve felt when the Taliban threatened to blow up the Bamyan Buddhas, and then days later actually did blow them up. Or the way you felt when human negligence caused wildfires that incinerated half the koalas in Australia, or turned the San Francisco skyline into an orange hellscape. For that matter, the same way most of us felt the day Trump was elected. I’d like you to feel in the bottom of your stomach the avoidability, and yet the finality, of the loss.

For thousands of kids in the DC area, especially first- or second-generation immigrants, TJHS represented a lifeline. Score high enough on an entrance exam—something hard but totally within your control—and you could attend a school where, instead of the other kids either tormenting or ignoring you, they might teach you Lisp or the surreal number system. Where you could learn humility instead of humiliation.

When I visited TJHS back in 2012 to give a quantum computing talk, I toured the campus, chatted with students, fielded their questions, and thought: so this is the teenagerhood—the ironically normal teenagerhood—that I was denied by living someplace else. I found myself wishing that a hundred more TJHS’s, large and small, would sprout up across the country. I felt like if I could further that goal then, though the universe return to rubble, my life would’ve had a purpose.

Instead, of course, our sorry country is destroying the few such schools that exist. Stuyvesant and Bronx Science in New York, and the Liberal Arts and Science Academy here in Austin, are also under mortal threat right now. The numerous parents who moved, who arranged their lives, specifically so that these schools might later be available for “high-risk” kids were suckered.

Assuming you haven’t just emerged from 30 years in a Tibetan cave, you presumably know why this is happening. As the Washington Post‘s Jay Matthews explains, the Fairfax County School Board is “embarrassed” to have a school that, despite all its outreach attempts, remains only 5% Black and Latino—even though, crucially, the school also happens to be only 19% White (it’s now ~75% Asian).

You might ask: so then why doesn’t TJHS just institute affirmative action, like almost every university does? It seems there’s an extremely interesting answer: they did in the 1990s, and Black and Hispanic enrollment surged. But then the verdicts of court cases, brought by right-wing groups, made the school district fear that they’d be open to lawsuits if they continued with affirmative action, so they dropped it. Now the boomerang has returned, and the School Board has decided on a more drastic remedy: namely, to eliminate the TJHS entrance exam entirely, and replace it by a lottery for anyone whose GPA exceeds 3.5.

The trouble is, TJHS without an entrance exam is no longer TJHS. More likely than not, such a place would simply converge to become another of the thousands of schools across the US where success is based on sports, networking, and popularity. And if by some miracle it avoided that fate, still it would no longer be available to most of the kids who‘d most need it.

So yes, the district is embarrassed—note that the Washington Post writer explains it as if that’s the most obvious, natural reaction in the world—to host a school that’s regularly ranked #1 in the US, with the highest average SATs and a distinguished list of alumni. To avoid this embarrassment, the solution is (in effect) to burn the school to the ground.

In a world-historic irony, the main effect of this “solution” will be to drastically limit the number of Asian students, while drastically increasing (!!!) the number of White students. The proportion of Black and Hispanic students is projected to increase a bit but remain small. Let me say that one more time: in practice, TJHS’s move from a standardized test to a lottery will be overwhelmingly pro-White, anti-Asian, and anti-immigrant; only as a much smaller effect will it be pro-underrepresented-minority.

In spite of covid and everything else going on, hundreds of students and parents have been protesting in front of TJHS to try to prevent the school’s tragic and pointless destruction. But it sounds like TJHS’s fate might be sealed. The school board tolerated excellence for 35 more years than it wanted to; now its patience is at an end.

Some will say: sure, the end of TJHS is unfortunate, Scott, but why do you let this stuff weigh on you so heavily? This is merely another instance of friendly fire, of good people fighting the just war against racism, and in one case hitting a target that, yeah, OK, probably should’ve been spared. On reflection, though, I can accept that only insofar as I accept that it was “friendly fire” when Bolsheviks targeted the kulaks, or (much more comically, less importantly, and less successfully) when Arthur Chu, Amanda Marcotte, and a thousand other woke-ists targeted me. With friendly fire like that, who needs enemy fire?

If you care about the gifted Black and Hispanic kids of Fairfax County, then like me, you should demand a change in the law to allow the reinstatement of affirmative action for them. You should acknowledge that the issue lies there and not with TJHS itself.

I don’t see how you reach the point of understanding all the facts and still wanting to dismantle TJHS, over the desperate pleas of the students and parents, without a decent helping of resentment toward the kind of student who flourishes there—without a wish to see those uppity, “fresh off the boat” Chinese and Indian grinds get dragged down to where they belong. And if you tell me that such magnet programs need to end even though you yourself once benefitted from them—well, isn’t that more contemptible still? Aren’t you knowingly burning a bridge you crossed so that a younger generation can’t follow you, basically reassuring the popular crowd that if they’ll only accept you, then there won’t be a hundred more greasy nerds in your tow? And if, on some level, you already know these things about yourself, then the only purpose of this post has been to remind you of them.


As for the news that dominates the wires and inevitably preempts what I’ve written: I wish for his successful recovery, followed by his losing the election and spending the rest of his life in New York State prison. (And I look forward to seeing how woke Twitter summarizes the preceding statement—e.g., “Aaronson, his mask finally off, conveys well-wishes to Donald Trump”…)


See further discussion of this post on Hacker News.

219 Responses to “On the destruction of America’s best high school”

  1. Jr Says:

    I don’t think one needs to be overly devoted to public choice theory to wonder whether increasing the white enrollment might not be seen as a benefit by some who propose this move.

    I am no fan of affirmative action in government programs, but one reason I am not totally opposed to it is that indirect efforts to increase racial/ethnic diversity often seem to create more harm than straighforward quotas.

  2. Scott Says:

    Jr #1: That’s exactly it. Experience has shown that math and science can easily survive, and thrive, in the presence of affirmative action for underrepresented or historically marginalized populations. What they can’t survive is the total destruction of the concept of merit that many people seem to find necessary after such affirmative action is no longer allowed.

  3. DS Says:

    Maybe this isn’t a bad outcome after all?

    Instead of having a large number of 8th graders go through a ridiculous amount of pressure (and in general encouraging test prep more than anything else, which also favors those who can afford it more than others):

    AS LONG AS THE CURRICULUM (both content and work expectations) at TJHS isn’t watered down, there will be a strong level of self-selection among the applicants. In addition, with the 3.5 gpa being a basic filter, incoming students of all backgrounds who are willing to work hard and challenge themselves, with the right level of help (in lieu of “affirmative action”), could succeed really well.

    I just don’t see the point of 13-year-olds duking it out and the gap between “in” and “out” being as low as 0.1% or something egregious like that.

  4. Gerard Says:

    I agree with DS. I don’t see why it should be obvious that a single high stakes entrance exam is a better way of selecting students than their prior performance in course work.

    At worst it seems like it should be an interesting experiment. How important is it really to select only the very highest performers on some exam versus giving a random chance of selection to a larger field of candidates ? If this is done and the school continues to outperform then I think we’ll have learned something fairly important about high level academic success. If it reverts to the mean, as you predict, then that will also be important information.

  5. Alexandre Zani Says:

    #2:
    “What [math and science] can’t survive is the total destruction of the concept of merit[…]”

    I don’t think that’s quite true. Merit implies the concept of deserts which I don’t think makes much sense here. Why does being good at math imply that you *deserve* a better math education than someone who is not as good at it?

    I think the concept of matching captures much more accurately what we want to talk about here. We don’t need a test for these schools because some students are more deserving than others. We need a test because different students are better or worse matches for these schools. A kid who mastered calculus is a better match for some introductory real analysis than a kid who struggles with trigonometry.

    It’s important for a variety of reasons, one of them being the test design.

    If I want to see who is most meritorious, I’m going to aim for a test that is very hard. It doesn’t really matter in what way it’s hard. If enough kids fail, it’s a success. If I want a test that provides good matching information for an advanced math and science program, then the success criteria will be how many of the kids who are accepted succeed and how many of the kids who were not accepted would have succeeded. I bet that to some degree, schools that think of themselves as selecting on “merit” make their tests hard in the wrong way. So there might be some room for improvement there.

    One way this is almost certainly true is that unequal access to test prep will make your test biased towards false positives for kids who have access to more test prep and false negatives for kids who don’t. So equalizing test prep access could be helpful.

    We don’t need to get rid of selectivity, but we need to move away from the idea that we’re selecting on “merit”. It’s not really the purpose and it’s a bad criteria. Let’s instead select for good match.

  6. Josephina Says:

    BRAVO Scott. You said exactly what has to be said about this issue.

  7. Jacob Steinhardt Says:

    DS #3: I think you’re simply wrong about the gap between TJ and, say, a typical magnet school; it’s much more than a 0.1% gap. I went to a magnet school in middle school, and then TJ for high school. TJ was the first point at which I had a real friend group.

    I don’t really see how you can change the student body without changing the standards. It simply isn’t an option to fail 30% of the class. Even with the current study body, our AP Physics teacher was made to “revise” his grading policies because too many people were getting low grades (he had previously insisted on teaching AP physics at the level of MIT’s freshman physics course, which for me was life-changing as it was the first class where I really had to grapple with difficult problem-solving).

    Incidentally, the humanities at TJ was also first-rate, but a lot of that comes down to the passion of the students. And most of what I learned came from my peers pushing me to excel *outside* of class, and doing the same for the next generation of students.

  8. Eli Says:

    Now the boomerang has returned, and it’s time for a more drastic remedy: namely, to eliminate the TJHS entrance exam entirely, and replace it by a lottery for anyone whose GPA exceeds 3.5.

    Excuse me if I just don’t see anything wrong with this. Sorry, but we all know these scores are less meaningful on the extreme high end. If you can scoop up everyone two standard deviations above the mean and select randomly from them, you’re doing fine, and the poor kids get a break from the the grueling regime of trying to rank higher than the other kids who are trying to rank higher than the other kids who are trying to rank higher.

  9. ZLC Says:

    Scott, I’m not sure I understand you correctly. Would you want this reform to happen or not?

    If yes, then clearly you are anti-Asian, for you are unfairly depriving us of this natural place at a top school as objectively measured by the entrance exam. If no, then clearly you are anti-Black and anti-Latino, perpetuating the system that unfairly shuns them from the best available education.

    This is a key aspect of the Left (the one we’re stuck with at the moment anyway) ; you cannot win. You of all people should know, having been repeatedly targeted by the “sneer club” for everything and nothing at the same time. They get angry if you helpfully try to explain yourself, they double down when you concede. There is no winning because they aren’t after genuine benevolent community policing, but after intimidation and humiliation (much like the Bolsheviks before them indeed).

    You keep using terms like “pro-white”, “right-wing”, “anti-immigrant”, “affirmative action”. Those are the wrong tools to get at the problem. By using those terms you’re already playing into their hands, and tacitly describing everything in terms of race and identity politics.

    It seems to me that there is a pretty simple truth to the situation. A set of people of all races and backgrounds, short and tall, rich and poor, fat and slim, got in a room and subjected themselves to an objective test of their intellectual abilities. The best of them got into TJHS. As a result, TJHS is one of the top schools in the country, and the students there are some of the top performing, and I’d wager, happiest. They get to realise their full potential unhindered among the brightest of their peers, and in fact helped, at a critical time in their lives. A chance which I’m afraid neither you nor I were really offered in our time. Which makes the closure of TJHS all the more cruel and salient in my mind, for I know very well what it is we are losing, and on a personal level, how bad the alternative environment that we are dooming future students to is.

    This isn’t about pro-whites. Indeed, if their numbers are increasing after this reform, couldn’t that indicate they were unfairly shunned from education before? They do make up more than 19% of the broader population after all. Or I could use the exact same logic to argue that in fact the 75% Asian majority we had was incontrovertible proof of “Asian supremacy” propagating a system of oppression. We both know this to be bullshit. One gets nowhere with that kind of logic, because it is bringing race into something that is completely orthogonal to it ; math, abstract thinking, and the love of science and natural thirst for knowledge.

    Do not assume that every over- or under-representation of demographics is caused by racism or injustice or evil intent, and do not fall into the trap of blaming pro-white groups (or indeed whites themselves) for this. The test results are what they are, and do not add or remove to the humanity we owe each other as humans anyway. After all, there are demographic under-representations in sports performance, and no one really seems to be particularly bothered with it. They are treated as normal humans like anyone else. Could there be natural tendencies for certain demographics to be disproportionately represented in analytical ability? Maybe. It is of course blasphemy to say as much in today’s highly-polarized, election-year America, but it is ultimately a statistical and empirical question.

    If you happen to have any geneticists as close trusted friends, I can only implore you to have a private chat with them. This is the elephant in the room as far as they’re concerned, and they might be (understandably) reluctant to openly talk about it, thanks to the current witch-hunting climate reminiscent of Galileo. But it is a fundamental thing to understand not to get your feelings misplaced in ad-hoc ideological fights, placed there by people who despise you (sneer club), pitting you against the people whose ideals (love of science and objective truth) are the closest to you.

    Sincerely, a fellow HE physicist and long time reader of your blog from across the pond, who wishes nothing but the best for you in those troubled times (and indeed who shall this time remain remain anonymous, as I have no intention of becoming the next unfortunate target of the cancel mob).

  10. Scott Says:

    DS #3: Like Jacob Steinhardt, I don’t see how you can almost completely swap out the students while keeping the curriculum the same. E.g., how will you continue teaching math past AP Calculus if you don’t have enough students to fill the seats?

    But what worries me even more is TJHS giving up on the ideal of “regardless of who you are, if you score high enough on an impersonal exam, you can come here.” To illustrate what I mean: I’d consider it way less than 50% as destructive if they’d said that from now on, 50% of admission slots would be based on GPA, marching band, badminton, and how powerful your parents are, and only 50% would be based on test scores. That wouldn’t be noticeably worse than Harvard or Stanford’s policy! From a nerdy kid’s point of view, it would raise the testing bar to get in but not to infinity, and reduce the number of students you’d probably associate with once you were there but not to zero.

  11. Scott Says:

    Alexandre Zani #5: That’s totally fine. I have zero objection to calling it “matching” rather than “merit” if people prefer. Likewise, to hark back to Carl Sagan’s quote, LeBron James might or might not be an “inherently more meritorious person” than I am, but he’s clearly a better “match” than me for the NBA.

  12. Scott Says:

    ZLC #9:

      I’m not sure I understand you correctly. Would you want this reform to happen or not?

    ROTFL reread the post?!? I thought I was screaming from the rooftops what I wanted to not happen.

  13. Disney Says:

    I’m not super familiar with your views, but I’d be willing to take a gamble and say that you think long term success is not influenced by childhood environment beyond certain minimum standards that avoid active harm. That the difference between a decent researcher and a Nobel prize winner is more to do with genetics and innate intelligence than which high school they went to, assuming they both went to high schools that had books, computers, a minimum of violence, and other necessary resources.

    So from that, it seems like your biggest issue with the loss of TJHS is the loss of the unique culture low on bullying, rather than a unique ability to make smart kids successful. Smart kids are already going to be successful. You say “[TJHS without admissions testing] would simply converge to become another of the thousands of schools across the US where success is based on sports, networking, and popularity.” and “[TJHS allows students to] attend a school where, instead of the other kids either tormenting or ignoring you, they might teach you.”

    But that doesn’t capture the specificity of it, because you also say “And if by some miracle it avoided that fate, still it would no longer be available to most of the kids who‘d most need it.”

    Obviously, if the people who need TJHS are ‘people who are bullied’, then most people who need it already aren’t allowed in, because most people who are bullied are not able to pass the entrance exam, as a consequence of the fact that most people aren’t able to pass the entrance exam.

    So it seems like you’re specifically talking about the need for schools like TJHS to protect smart children who are bullied. If that’s the case, I’m less sympathetic. The goal of preventing bullying is noble, but if the goal is to prevent bullying only in cases where the victims are very smart, that seems substantially less noble. It also seems unlikely to me that a dollar spent opening a new school for brilliant children reduces bullying more than that same dollar spent on other anti-bullying initiatives, so I don’t think it would be a valid response to say that preventing bullying of smart children is simply an effective way of preventing bullying generally.

    I guess you could also argue that it’s uniquely important to prevent smart children from being bullied because childhood bullying is an active harm that might make them less successful as adults and lead to worse outcomes for society as a whole.

  14. Scott Says:

    Disney #13:

      I guess you could also argue that it’s uniquely important to prevent smart children from being bullied because childhood bullying is an active harm that might make them less successful as adults and lead to worse outcomes for society as a whole.

    Yes. Just to calibrate you, while some people might now think of me as “successful,” I don’t think I’m anywhere close to recovering from the bullying I endured between the ages of roughly 12 and 16. I’m now 39 years old and married with two kids.

    And if TJHS saves only the very smartest kids from bullying and not the rest, then to me the solution seems clear: open a thousand more magnet schools with admission based on demonstrated abilities (both because that lets in most potential victims, and because it keeps out most bullies). Save as many kids as you fucking can.

  15. asdf Says:

    > I wish for his successful recovery, followed by his losing the election and spending the rest of his life in New York State prison.

    Headline: “Aaronson coddles Trump, wants to keep him out of federal prison…”

  16. Elmer Fudd Says:

    DS, firstly, you’re overrating the role of prep. Smart and hardworking kids do well if you give them a book. I’m sceptical that expensive tutors add much to that.
    If the school doesn’t water down it’s curriculum – this is subject to the Lucas critique. I expect many mediocre but aspirational students with 3.5s will apply, the median student’s aptitude will fall steeply, and teachers will adjust to teach the students before them.
    North American literati are loath to acknowledge that endowed aptitude and interests vary widely. The holocaust shocked them into banning any mention of inherited traits. This taboo causes us to enact poor policy and poisons our discourse.

  17. JM Says:

    DS #3:

    Maybe this isn’t a bad outcome after all?

    Instead of having a large number of 8th graders go through a ridiculous amount of pressure […]

    I’m not going to go into much detail here, because it’s personal & embarrassing, and I haven’t quite climbed out of the hole I dug myself into yet. It’s also embarrassing for me to share this much and I kind of hate myself for having written the below because I sound like a stuck-up braggart. But this decision by the school board has made me sad enough that I have to at least try and make you understand.

    Please believe me when I tell you I wouldn’t have fucking killed for a chance at going through the “pressure” of applying to TJ rather than have no option but go to a somewhat selective, very expensive (less so with my academic scholarship) private school where everything was painfully easy. One of my big regrets is not at least trying to explain to parents & teachers that my life depended on my social solitary confinement and intellectual sensory deprivation coming to an end, because I thought I could just pretend I was happy until college (where surely things would be different, right…?). I probably would have died (I would like to clarify that I am being neither hyperbolic nor figurative) had I been born before the era of free online textbooks.

    I think you’re really underestimating how much of this is about having a peer group that you can actually talk to about things you find interesting. I could count on one hand the number of kids that I could actually have something close to a normal conversation with where I wasn’t pretending the whole time (and, while we’re on this subject, I’m not exactly shocked that my high school was overwhelmingly white and TJ very much isn’t). Even then, while I’m forever grateful for having met them, I also wish I had known someone that I could have a conversation with that was neither fundamentally fake nor me-as-teacher-them-as-student.

    I don’t want to share any more details so I’m ending it here…

    (I’ll just add that I’m sick of culturally powerful white people doing things with the goal of helping white people but phrased with the language and aesthetics of social justice. (I’d rather they pretend to care then proudly not care, needless to say.) I guess in this case we can call them NIMSDers—not-in-my-school-distict-ers.)

  18. Elmer Fudd Says:

    Disney, it’s not just about bullying. Kids with high aptitude need to be around other people with high aptitude. Being a smart kid in a dumb kid environment is very isolating. Worse, it causes the kid to spend their time watching sports and sitcoms to figure out what everyone else is talking about instead of pursuing their interests because they have no one to share them with. It’s frustrating at best, derailing at worst.

  19. Scott Says:

    Gerard #4: I’d say the “experiment” has already been tried many times; in a statistical sense we already know its outcome.

    As might or might not be clear by now, my central avuncular concern is toward a group of kids with the following property: that as long as their path to success depends on pleasing the people who currently have power over them (whether that means teachers, other kids, guidance counselors, admissions officers…), they’ll probably do poorly; but as long as the path depends on a test score they’ll probably do well. It’s not that these kids are antisocial or misanthropic; it’s just that they might never have met people who are a match for them. My goal is to get these kids, as quickly as possible, into environments that do have intellectual matches for them—both to deflate their pretensions of being the smartest kid on the planet, and to give them a shot at something like normal social development.

  20. Tim Abbott Says:

    Just posting to agree with Jacob Steinhardt (#7) that TJ was a unique experience in American education, and that like MIT, the student body was just as important to my education there as the teaching was. I posted more detailed thoughts on Hacker News: https://news.ycombinator.com/item?id=24682119.

  21. anonymous Says:

    If I have to choose between being politically correct, diverse and minority supporting to the levels that would strip people of any concept of merit and reduce them to a gender and skin color, or to be as racist as Trump, I think I’d rather be as racist as Trump.

    In some way I think Trump is actually far less racist than the people behind this move, as Trump doesn’t reduce people to gender and race statistical numbers.

  22. Peter Shenkin Says:

    @zlc #9, among others.

    I’ve been part of these discussions for years with fellow Bronx-High-of-Science alums. (I’m l’m class of ’63.)

    You say “It seems to me that there is a pretty simple truth to the situation. A set of people of all races and backgrounds, short and tall, rich and poor, fat and slim, got in a room and subjected themselves to an objective test of their intellectual abilities. The best of them got into TJHS.”

    This is not quite right, at least for Bronx Science. Actually very few Black and Latino kids even show up for the exam. In many cases, they had no idea that BHSS (or Stuyvesant or Brooklyn Tech). Many more showed up in the ’90s than do now. I don’t know if there was affirmative action program at that time in NY.

    In discussions, some of the Black alums said that the elementary and JHS teachers, out of overprotectiveness, don’t want to expose their kids to an environment where they don’t feel the kids would be comfortable. And at Science, there are so few Black kids that they really don’t have friends at home that also go there. Brooklyn Tech is a much bigger school, and though the Black and Hispanic population is only slightly hire, there are enough of them to form groups of friends that help each other.

    You might say that they should be able to form friends with anyone, but I have to tell you that, riding over an hour each way on the subway to get to Bronx Science, I didn’t make a single new friend in the 3 years I was there. My friends were the kids I knew in my neighborhood growing up. (Starting 20 years later, with class reunions, mere acquaintance from that past have become friends.)

    The kind of affirmative action I can get behind is encouraging bright students with a technical bent of whatever background to apply. Maybe there was more of that in the ’90s; I don’t know. But I have always supported retaining the entrance exam.

    As far as prep goes, my guess is that it can help, especially in self confidence, which is very important, because it can give a kid a realization that “I can do it.” In NY, I came across a reference that led me to believe that the city has prep courses that kids can enroll in for free.

    Finally, I think the sports analogy is apt. When learning to play a sport, it’s really helpful being able to work with kids at your own level. If you jump into an outdoor court with kids who are much better, you stand a good chance of being humiliated and you won’t learn anything. Of course if you’re much better than they are, it won’t be fun for you and you still won’t learn anything. Back in my day, when the Jews were today’s Asians, Bronx HS of Science was in the air from childhood in my circles. It was always an aspiration of my parents that I would get in, and I did, though my mom was never a “tiger mom.” But when you grow up in an environment where such aspirations are either little known or actively discouraged, it must be tough. Hence I would push awareness, encouragement, and exposure. If that’s affirmative action, then it doesn’t bother me.

  23. TJ Says:

    Regular lurker but not commenter here. While I strongly agree with the sentiment and overall conclusion of this post, I wanted to just add that I do think you are painting too rosy a picture of the modern TJHS experience. A lot of my friends there had fairly negative experiences, due to the high-level of pressure and competition. Quite a few didn’t want to be in STEM but ended up in STEM, many others got counselling, others burnt out at university, and lots more just generally disliked the whole environment. While many also had positive experiences, it comes with its share of problems for even the most brilliant students there.

  24. Ethan Says:

    Scott #10

    I don’t want to get into the matter at hand because it’s a very complex issue. Being a public school, there is the taxes of everybody at play so while I agree that in principle talent should be nourished, there is no right that it be nourished at the taxpayer expense so taxpayers do have a say in how that money gets to be spent.

    I just want to comment on this,

    “regardless of who you are, if you score high enough on an impersonal exam, you can come here.”

    There is no “regardless of who you are”. Already by being born in the right family at the right place (such as an second generation Asian immigrant raised in the district that makes you eligible to attend this school) you have a boost in your chances of doing well in this exam. Warren Buffet likes to say that his parents biggest gift to him is being born in the United States.

    You say that currently the student body is ~75% Asian. I am inferring that it’s mostly children of Chinese and Indian immigrants and that these students were brought up in homes that valued hard work and academics.

    I hate to say it but this but all exams have an intrinsic bias towards those who, for whatever reason, do well in them (intrinsic talent, preparation, both). This notion that exams give people a chance sounds eerily similar to Charles Murray praise of IQ and SAT tests. In his recent interventions he has changed his tune somehow (here https://www.youtube.com/watch?v=UWCkxw4gmbE you can listen to him acknowledging the role that luck plays in one’s personal success) but for a long time he was the biggest proponent of the notion that if you didn’t succeed in life, it was all your own fault and that you have no right to complain.

    I said this not taking sides one way or another, just saying that it seems to me the issue is not as simple as you portray it.

    There is one country in Europe that believes the mantra of “an exam open to anyone” to the core, France -a mindset born out of the French Revolution-, and the end result is that the public funds a system that has become de-facto an hereditary aristocracy https://www.nytimes.com/2005/12/18/world/europe/elite-french-schools-block-the-poors-path-to-power.html (I think quoting this is covered by the fair use doctrine)

    “These schools — officially there are 200 but only a half dozen are the most powerful — have their roots in the French Revolution and the Napoleonic Empire. Just as the SAT’s were meant to give all American students an equal shot at top universities, the examination-based grandes écoles were developed to give the bourgeoisie a means of rising in a society dominated by the aristocracy….But children of blue-collar workers, who made up as much as 20 percent of the student body of the top grandes écoles 30 years ago, make up, at best, 2 percent today. Few are minority students….the vast majority of students entering the grandes écoles today come from special two-year preparatory schools, which draw their students primarily from high schools in the country’s wealthiest neighborhoods”

  25. David Karger Says:

    Scott, before I get too upset, I’d like to know (serious question, not rhetorical) if there is any evidence at all (beyond the anecdotal) that selective high schools like this offer benefit to society at large that justify the inequality they create. It’s clear that they offer benefit to those who attend—but do those benefits spread?

  26. Scott Says:

    anonymous #21:

      In some way I think Trump is actually far less racist than the people behind this move, as Trump doesn’t reduce people to gender and race statistical numbers.

    Right, Trump goes straight to the point and just burns the entire world to the ground, rather than this or that magnet school! And while he’s at it, brutalizes the whole concept of “competence” outside of “obedience to him,” and “truth” outside of “what he and his supporters want.”

    Living now, as we do, in a world where the Trumpists and the wokeists drown everybody else out, it’s easy to forget that not only is there a third way forward, but that far from being a utopian fantasy, that way was once responsible for the Enlightenment and for virtually all the scientific and moral progress of the human race.

  27. David Karger Says:

    ZLC #9 and Scott #10. You are using words like “objective” and “impersonal” as if testing students is the same as measuring the speed of light. While performance on a high stakes exam is certainly correlated with academic ability, there are tons of confounds. I’d bet performance correlates strongly with ability of the parents to pay for high quality preparation. And as Shenkin #22 points out, you can only do well if you show up for the test, and that correlates strongly with being part of the upper class community that knows it’s good to show up for the test. Ramanujan won’t ever hear about it.

    Ridgway, in his paper, “Dysfunctional Consequences of Performance Management” points out: “Quantitative measures of performance are tools, and are undoubtedly useful. But research indicates that indiscriminate use and undue confidence and reliance in them result from insufficient knowledge of the full effects and consequences. Judicious use of a tool requires awareness of possible side effects and reactions. Otherwise, indiscriminate use may result in side effects and reactions outweighing the benefits.”

  28. CC Says:

    Scott, you are lobbying for resources for people like you which is fine. Others lobby for their own interests. This is normal in a democracy and voting is how things are decided (and of course the constitution and its interpretation by the courts). You did not make an intellectual case other than the fact that you and people like you had a great time there and thrived. Various groups can say similar things about their own experience (such as secluded neighborhoods etc) and ask to preserve that status and you will agree with some and not with others based on your biases and experience and interests. You can try to step back and think more holistically beyond your own experience and see how competing interests for resources should be taken into account in a democracy. For instance if someone had a special needs children and want more resources for those children and closing TJHS can provide those resources, should one decide for or against? There are no simple answers to these. I understand your sadness about changes at an institution which helped you thrive.

  29. JDewey Says:

    Hey Scott, someone pointed me to this article earlier today as being on Hackernews, and I realized you were the one who came to our school to give that quantum talk when I was a TJ senior in 2012! Very cool to see this intersection again, 8 years later!

    From my time there, I can say that many of my classmates paid for test preparation prior to taking the TJ exam (which would be followed by an essay and application package to be submitted; not sure if they do this the same way today.) Additionally, this commenter on HackerNews references a poll from the FB alumni page that samples 500 respondents and finds that at least 50% of respondents from recent classes paid for test prep: https://news.ycombinator.com/item?id=24683815 (I can verify the existence of the poll since I’m also someone who took it.) And test prep centers are very prolific nowadays – I could rattle off the locations of test centers that advertise preparation for the TJ exam in places like Annandale, Centerville, Chantilly and elsewhere.

    The problem with test prep is that its 1) locked behind the paywall of paying for private tutoring and 2) it’s almost always a great boost to your exam score. I would know – I paid over $500 for a week of TJ exam-specific training over the summer to prepare for me the test, and while it’s been quite a long time, I know the delta of the score on the mock test they gave us at the beginning and the end of the exam was substantial. And I know other students who were admitted paid more money (easily in the $1000’s range), for even more preparation (weeks or more.) This doesn’t strike me as a meritocracy.

    Given the proliferation of paid test preparation prior to taking the exam, which I am confident will give almost any student an edge when taking the exam, do you think that’s evidence that using standardized testing is flawed as a method of entry for a school like TJ? Because it seems that way to me, so seeing you defend the exam like this is puzzling.

    For the record, I’m not a fan of switching to a lottery system either, but I’ve never viewed the exam as being a great tool either given how relatively easy it is to game it with money. I did the same paid prep course prior the SAT in Junior year, and saw a 300 point jump between the start and end of the program in mock tests (this was when the SAT was out of 2400 points); I ended up with a pretty good SAT score, and I know the specific techniques they taught us for the exam came in very handy when I took the real SATs, much the same way the TJ exam prep did. Given that only 3% of TJ’s population qualified for free and reduced price lunch while I was there (compared to at least 20% in Fairfax County at the same), I don’t think it’s a stretch to say that there’s some element of pay to win here. Does this strike you as a meritocratic system? Do you think a poor smart kid is gonna be able to get into TJ as easily as a middle class or wealthy smart kid, if we keep the test? And in a county where Black and Latino families are overrepresented among those the poor and working class, how does the test that’s so easily gameable, sound meritocratic?

  30. Scott Says:

    CC #28: The central difference, as I see it, is that there are already special needs programs at virtually every school in the US—indeed, there’s a legal entitlement to them (which is great, and which I strongly support). Even kids who attend private schools are entitled to remedial education at taxpayer expense if they need it—my mom worked for decades as a city-employed remedial reading teacher at a Catholic school in Philadelphia.

    By contrast, in the US, “gifted” programs are often nonexistent or a joke (e.g., extra busywork of the same kind that was the problem in the first place). In a country of 300+ million people, there are maybe 5 or 10 schools that take any interest in nurturing future Feynmans and von Neumanns and Turings. And those 5 or 10, which for geographic reasons serve only a few percent of those in need, have been under attack for decades and are now slated for demolition, not because of their failure but because of their success.

    Do you agree that this is a relevant difference? And do you agree that, even if nurturing future Feynmans and von Neumanns and Turings isn’t the wider society’s main interest, it clearly ought to be one of them?

    If there’s anything whatsoever that unifies the … err … “Shtetl-Optimized community,” surely it’s that we actually care about mysteries like P vs NP and quantum gravity; we actually want to see them solved. I submit that one of the central things any of us can do toward that goal is to nurture the talent pool, to race against time to keep as many young minds as possible open before the world closes them. So it wouldn’t surprise me if a single high school like TJHS or Bronx Science contributes more to the advancement of science than any professional society or granting agency! It seems to me that any of us who care about science ought to urge the founding of a hundred more such schools. But in any case, yes, if this blog has ever stood for anything, it stands for not gutting the few we have.

  31. Mark Says:

    When Gamala Harris offs Biden in the first term, you’ll get a whole lot more of this. But don’t worry, we’ll still laugh at your Trump jokes.

  32. Anon93 Says:

    David Karger #23:

    Is there any evidence at all (beyond rhetorical) that these schools create inequality? At least when you look at Stuy and Bronx Science, they seem to reduce inequality. Asian-Americans are the poorest group in NYC. When my grandpa, a working-class Jewish guy, went to Bronx Science, was it also creating inequality?

  33. Ethan Says:

    Scott #29

    Again, I am not taking sides but the biggest flaw in your argument is that while I do agree that it would be nice to be able to identify the next Feynman, and von Neumann, and Turing and have answers to those questions, the record of identifying those people in advance when they are children is very poor (in addition the only American among the three, Feynman, went to a non magnet high school). In retrospect, all three look geniuses, but Feynman was notoriously surprised that his registered IQ in high school was a mere 125. Lewis Terman, a man obsessed like none in American history with gifted children, tested and failed to identify the only two people who went on to win a Nobel Prize among the children he subjected to his tests (Luis Alvarez and William Shockley).

    I have read many biographies of geniuses -some in more detail than others. There are very few of them that were identified in high school beyond being “smart” like tens of thousands of students who are identified as “smart’ every year in the US who end up following successful -but traditional/non genius- careers. Take John Nash https://mathshistory.st-andrews.ac.uk/Biographies/Nash/ “Johnny’s teachers at school certainly did not recognise his genius, and it would appear that he gave them little reason to realise that he had extraordinary talents. “. He wasn’t always good in math competitions and yet Richard Duffin notoriously recommended him as “He is a mathematical genius”.

    By the mere fact of being born in the US, with all the opportunities at their disposal in the course of their lifetimes- future Feynmans and von Neumanns and Turings have already won the jackpot. The biggest threat to the development of these aspiring geniuses talents is not the lack of serious gifted education at the high school level but the recruitment, starting in the early 1980s, of America’s most promising young minds by finance and management consulting companies. A problem compounded by big tech companies targeting the same cohort of people starting in the 2000s. It’s tack of opportunities to make a decent living in basic scientific research that is the problem in my opinion. I haven’t met anyone in love with the idea of being poor for the sake of science.

  34. Scott Says:

    David Karger #25: If you or others want to bring social science research into the discussion, I’m happy to look! For my part, I don’t claim to base my stance on anything more than a lifetime of talking to people who grew up in the US and eventually succeeded in STEM. With unbelievable regularity, those who report liking high school, or even just finding it tolerable, turn out to have gone to one of a handful of STEM magnet programs.

    Of course, that observation, no matter how overwhelming, still doesn’t tell us the causality. Do magnet schools help make kids successful scientists? Do they merely avoid crushing their curiosity and ambitions? Would most of the kids who attend these schools have succeeded intellectually anyway, with the schools’ main contribution being social and emotional (hence the characteristically fond recollections)?

    I’m someone who never got the magnet school experience, but had to chart a rougher and more unusual course, which included “escaping” to college at age 15 and picking up a GED along the way. So I’ve naturally wondered about these questions for a long time. But on reflection, I don’t see how my central recommendation—namely, to preserve the magnet programs we have, and open many more like them—depends much on their answers! I’d like the kids I meet today, at places like Mathcamp, to have every opportunity to be smarter than me and/or to be happier than me. 🙂

  35. Scott Says:

    JDewey #29: Thanks for that very informative comment! What you write surprises me a little, since in the case of the SAT, I thought the research was pretty unequivocal that fancy prep courses improve scores by only a tiny amount, if anything, compared to spending the same time studying on your own with cheaply available prep books. Of course, many students won’t actually do the latter—it can be hard to motivate yourself!—but at least the option is there. Personally, I think it should be the responsibility of the test-makers to publish free prep materials that equalize the playing field as much as possible. What was the situation with the TJHS entrance exam: could one get free (or cheap) study material?

    More fundamentally, my position has long been that, sure, standardized test scores can be gamed a bit by affluent parents—but literally every other metric can be gamed a lot more! Grades? Extracurriculars? Essays? Recommendation letters? Are you kidding me? 🙂

    Also, if the SAT is so easy to game, then why, in the college admissions scandal a few years ago, did the super-elite parents like Lori Loughlin have to resort to outright falsification of SAT scores? Why not just pay $100K for the world’s best prep course? Grades and almost everything else could be bought by working the system, but there were two notable things that even the most privileged families had to outright fake or steal: (1) athletic achievements and (2) SAT scores.

  36. Jair Says:

    I have no strong opinions on how good of a filter standardized testing is to find those who would excel in a challenging academic environment – I’d think, pretty good, but not perfect. But that to me is part of a larger goal of judging students based on *what they can do* rather than *what hoops they are able to successfully jump through.*

    Scott’s worry has been with the very strongest students, but on the other end of the academic success scale, I had friends in high school who could ace exams but consistently failed math classes and were kept years behind their ability because, for whatever combination of factors (lack of self-control or maturity, family problems, etc.) they did not do enough of the homework. Consequently they were even *less* motivated to keep up with homework now that it was not only easy but insultingly easy, and exceedingly dull. It was as if, as an adult, you were made to write the alphabet a thousand times over, and if you did not complete it, it would be taken as further evidence of your illiteracy.

    As for bullying, I wonder if it could be alleviated with greater freedom. Teenagers of a certain age should be treated like adults. My preferred model would be to abolish high school altogether – or rather, to make it just like college. Completely optional, with total freedom to choose classes and schedules. I don’t see much reason to force older teenagers into seats when they would rather be elsewhere. Those who don’t feel like attending should be able to come back whenever they want and finish where they left off, regardless of age. There’s not much bullying in college – the bullies have no reason to show up.

  37. Hyman Rosen Says:

    Are we really doing socially maladjusted kids a favor by planting them into an environment with other such kids? How about getting them treatment or programs that will improve their emotional intelligence instead of focusing on their scientific intelligence?

    Shouldn’t your own [in]famous post about your terror at interacting in a normal way with women teach you that your schools desperately failed you? No matter how much math and science you learned? If you can’t make friends with regular kids, can’t talk about anything but some piece of science, and seek to escape into a place where you won’t have to, you have an illness. Escape may give relief, but it won’t give happiness. There’s a world full of ordinary people, and you have to deal with them eventually.

  38. Greg Price Says:

    Sad news. Like Jacob and Tim, in my experience at TJ the greatest thing about it was the other students — having a group of peers who were just as enthusiastic about things like math, and physics, and computer science as I was. We learned a tremendous amount from each other, and gave each other the confidence to push far beyond the curriculum or what anyone would normally expect high-school students could do.

    I do think that for most TJ students the baseline counterfactual experience, in a world without TJ (or with a diminished, lottery TJ), would be better than your high-school experience was, Scott. The area it draws from — Northern Virginia, primarily Fairfax County — is one with a high density of, well, the kinds of kids who go to TJ, and with the kinds of parents who tend to have them: highly educated parents, affluent parents, and parents who highly value their kids’ educations. For many TJ students in that counterfactual world, they’d have some of those peers of the kind I’m so grateful to have had, even if fewer of them; and the school at large would have respect for academic success and not contempt. It might be like some of my friends’ experiences in the high schools of… Palo Alto, say.

    One correction, I think: there’s no finality, yet, in this loss. AIUI, this is a proposal, which the school board will be considering at a meeting on Thursday. The Fairfax County school board has always, in the time I was paying attention, had members that would end TJ outright if they could, and others that sustained it. I haven’t seen any reporting that claimed to have counted up how the votes will fall on this; I hope they come out in favor of keeping TJ the uniquely valuable school it is.

    In the meantime, I guess those of us with first-hand experience there should be writing to the members of the school board — or perhaps better, asking our parents and friends who still live and vote in the area to do the same.

  39. Scott Says:

    Hyman Rosen #37: As soon as kids grow up and are free to move, usually the first thing they try to do is to surround themselves with others who are abnormal in the same ways they are! In my case, certainly, moving to a magnet school would’ve given me a better sampling of the sorts of people who I now deal with in my adult life.

    It’s true that, to be effective at life, one needs to learn how to interact with a wide variety of people, including ones wildly dissimilar from oneself. The problem here is that throwing nerds into the general American public school system, with no preparation, is liable to teach them exactly the wrong lessons: namely, how to “interact” by cowering or running away. It would be better even to homeschool the nerds, keep them totally isolated from the world, than to “teach” them the “lessons” that many schools have in store.

    If you want to teach nerds how to interact, then I think the key is this: you first need to put them in an environment where it’s taken for granted that the things nerds are good at are worthy of respect. Crucially, magnet schools and the real, grownup world are both much likelier to provide such an environment than the typical American high school. For an explanation of why, there’s no better source than Paul Graham’s Why Nerds Are Unpopular.

  40. Greg Price Says:

    For fun and details, here’s the actual proposal, in its current form, prepared for an upcoming Oct 6 school board meeting.

    It’s a bit revised from the version Jay Mathews was writing about (that’s here, but also detailed in the newer document), but with mostly the same effect.

  41. Sniffnoy Says:

    DS #3, David Karger #27, JDewey #29:

    In fact, as best as can be told from attempts to make systematic studies of them, the increase on SAT scores provided by test prep on average is pretty small. Now in this case we’re instead discussing the TJ entrance exam rather than the SAT, but I would expect the different test to not make too much of a difference here probably? So, the supposed test-prep bias seems to be pretty small. And note that here I’m implicitly granting the premise that test-prep is pure test preparation that only increases test scores and doesn’t at all actually make one better at what’s supposed to be tested!

    Alexandre Zani #5:

    I don’t think the word “merit” is usually necessarily used in the restrictive way you suggest? I think it’s often just a synonym for “skill” or “ability”, and I think that’s all that’s meant here. Point is I don’t think swapping out the word will change much of substance here…

    Disney #13:

    I think a key thing here to consider is, what do we mean by “success”? Sure, a smart kid might eventually do well for themselves regardless of where they go, but if they end up doing well for themselves as, I don’t know, a marketing executive or something, that’s kind of a civilizational loss, isn’t it? The point isn’t to get people to the point where they’ll make a lot of money and be happy, the point is to get people to the point where they can and will push the frontier! And my suspicion is that a good environment can matter quite a bit there.

    Similarly, getting together concentrations of smart people who can bounce ideas off one another seems to be pretty important as well, and sometimes they even turn into highly productive “schools”…

  42. Gil Kalai Says:

    The post starts with Carl Sagan comparing gifted children (in science) to gifted athletes. What is the situation (in the US) with gifted athletes? Are they send to special high schools? Or are there tools to encourage them in their ordinary high schools? Or are there out-of-school activities for them?

  43. jonas Says:

    And just two days ago I was hoping you’d write a post about mathematics, on occasion of “https://mycqstate.wordpress.com/2020/09/29/it-happens-to-everyonebut-its-not-fun/” the error found in a certain quantum information theory result.

  44. Scott Says:

    Gil #42: In the US, it would be fair to say that athletes don’t need special high schools, simply because the “normal” high schools are largely oriented around athletics anyway. At my school, for example, we listened to announcements on the intercom every morning about the exploits of the football and basketball teams against neighboring schools. The math team was never mentioned, even if we won a big championship. 🙂 School spirit meant rooting for the football team. And yes, there are extremely well-developed routes at most schools to scout out and develop unusual athletic talent, and funnel it to the college level. Here they don’t mess around!

  45. OhMyGoodness.. Says:

    Scott #43

    They truly don’t mess around and there are boarding and specific public high schools schools that specialize in basketball and play a national high school schedule. Since speaking about TJHS in Virginia the basketball analogue of that would be Oak Hill Academy in Mouth of Wilson VA. Cost for boarding and tuition is around $40k per year and they have 27 alumni that played in the NBA and are often the top rated high school basketball team in the nation.

    I was pleased you took the high road in your best wishes for POTUS since as we all know-

    https://youtu.be/KIiuTMzIHB4

  46. fred Says:

    This is yet another example of the socialist far left being so obsessed with correcting at all cost what’s supposedly wrong with America that it’s okay to wipe out all that’s right with America in the process.
    With equality of outcome, they first focus on skin color, before they realized they have to come after the ultimate thing in the lottery of life: the brain.

    Someone should explain to them that all the technological marvels they’re using 24/7 will not appear out of thin air just because they’re taking them for granted. That it takes very special gifts and sacrifices to make those things happen.

    This is why in a previous thread I was concerned that some people were saying things like “if Godel (or whatever exceptional person) didn’t discover/invent xyz, someone else would have eventually”. It is not true depending on the size of the human population. And this sort of argument is one step in the wrong way to think about all this.
    Exceptional brains are rare by definition, so we need a big enough pool of births to make them appear (they appear regardless of color and social status) and then the right circumstances to make them flourish (this is why we need equality of opportunity).

  47. Jon Tyson Says:

    To follow up on Gil’s question, at my school in Texas there were required pep rallies on Friday afternoons, where cheerleaders would put on a show to get people excited and cheering about the football game before it even started.

    Fortunately, my favorite math teacher arranged fictitious appointments to get me out of all of all the rallies. That said, I’m not sure anyone ever actually got in much trouble for not showing up.

  48. Nathan Williams Says:

    Class of 1994 here. If anyone calls the place TJHS, that’s new and different; in my experience it’s either TJHSS&T or TJ, locally.

    Even back then there was a notable chunk of the student population that wasn’t interested in (what we now call) STEM, but had been steered in TJ’s direction essentially as a college-prep school. Which may not have been wrong, tactically, but more than a few of them were resentful about the STEM focus of the curriculum they went through. I recall a lot of navel-gazing with classmates who *were* science-focused about how one might change that, to no avail.

    Given that and the fact of paid prep for the entrance exam – also not a thing back then, in the school’s first decade – I would very much wonder how much the tech mission is being accomplished, or if it’s already dominated by that crowd.

    (I am appalled at the fact that there is prep for the entrance exam. My reaction to that is to want to destroy the exam and salt the earth. The existence of paid test prep should be criminal.)

  49. B. Says:

    I see three distinct questions from what you (Scott) wrote and the comments:

    1. Are the reasons for these schools to close good reasons?
    2. Are those school the good way to “push” smart kids?
    3. Is it a good thing to push smart kids?

    For 1., I am quite unable to answer though I tend to think that the answer may well be negative.

    For 2., I’ve read the following by Elmer Fudd #18: “Kids with high aptitude need to be around other people with high aptitude. Being a smart kid in a dumb kid environment is very isolating.” I think that’s not right, and I agree with David Karger that one should honestly take the cons of such schools into account. My criticism of Elmer Fudd’s sentences, beyond the expression “dumb kids” that really is a problem to me, is that I consider much more important for school to teach kids to live in a society where not everybody has the same abilities, skills or even ideologies (to “make society” as we usually say in French) that to teach them some specific math or history. Thus I think that separating “high aptitude kids” from others is a wrong decision since it is necessary for kids (and older people as well!) to learn how to behave among different people. Though I absolutely trust and sympathize with Scott’s difficulties as a teenager, I really think it wouldn’t have been the best choice to never let him encounter kids with very different skills at school. I am pretty sure he handled the recent difficult situation much better with his experience.

    Now for 3., another solution for pushing smart kids could be to have out-of-school math (or whatever) activities, or even in-school special classes. It would avoid separating the smartest kids from other ones (really, reading “dumb kids” is shocking) while giving them enough food for the mind. Only remains the question whether pushing these so-called smart kids is necessary: They usually come from the more-privileged population (or at least more-educated), and this tends to favor social reproduction. And there is no need to learn complex variable calculus at 8 to become a good mathematician or scientist! I think that most, if not all, French Fields medalists followed a completely standard curriculum in high school before attending elitist higher-education.

  50. Vaarsuvius Says:

    Scott, I’m not sure your target market of “smart, underprivileged kids who perform very well on standardised tests but poorly otherwise” is as large as it may seem. I am not disputing that there exist large amounts of smart, underprivileged kids, but for most people (even smart people) preparing for a test requires you to know there’s a test and time and environmental availability to prepare, even if you are using cheap books available for free online. Just like how statistically there are probably Einstein-level intelligent people who died after a short life of dysentery or a long life as an unusally productive sustenance farmer, most of these underprivileged kids probably work part time jobs and have to take care of siblings at home. Doing extra exams are probably the last thing on their mind.

    An interview by qualified personnel would be much better than a test in my opinion, since any metric that can be viably skewed by stomach cramps at the wrong time is hardly a reliable benchmark. Of course, interviews are extremely resource intensive, so perhaps some sort of group selection/group interview process? You may argue that the exam is the best of a bad lot, but hinestly I’d much rather the examiners review what they cooked up after an hour of minecraft than how much they can perform under extremely unrealistic and pressure-heavy conditions that some bright individuals are very poorly weighted towards (and which can be skewed signficantly by prep, result-buying parents or not)

  51. Gerard Says:

    Scott #39

    > The problem here is that throwing nerds into the general American public school system, with no preparation, is liable to teach them exactly the wrong lessons: namely, how to “interact” by cowering or running away.

    I think a lot of the damage is done long before high school. As soon as I encountered “other kids” in kindergarten I realized that they were nothing like me. It wasn’t until high school that my peers started seeming like somewhat normal intelligent beings to me. The high school I attended was Catholic, all male, affordable to my working class parents and only somewhat selective (you had to score in the upper 50% on the entrance exam to get in).

    So if your main concern is with the social environment I’m skeptical that a hyper-selective, hyper-competitive school is necessary for making that tolerable.

  52. Scott Says:

    B. #48:

      I am pretty sure [Scott] handled the recent difficult situation much better with his experience.

    Which recent difficult situation are you talking about?

  53. Recent TJ Alum Says:

    Hi Scott, I’m a fairly recent TJHSST alum, and wanted to thank you for the quantum computing talk you gave when you visited! I remember being mesmerized by the image of the Bloch sphere you had on one of your slides, and went home to eagerly search for details online. I can’t say that I understood much back then, not having learned any linear algebra yet, but it was inspiring to me that advanced mathematics could and was being used to tackle cool technical problems. When I did eventually learn quantum mechanics in college, I had an ‘Aha!’ moment going back to your talk 🙂

    On the subject of the current debate, I had a tremendous time at TJ, as did several of my friends (similarly to some previous commenters, it was also the first time in my life that I even had more than 2 friends who were interested in talking about technical subjects). The difficult classes, extracurricular math/CS/science teams, and great peer group made high school paradise.

    I don’t agree with the proposal to completely get rid of the admissions test. However, to play devil’s advocate, there are two points I’d like to bring up.

    Firstly, the TJ admissions process has never been as selective as, say, elite college admissions. A few years ago, the admit rate was around 15%. For the most recent admitted class, it has risen to around 19% due to fewer applicants. I knew many brilliant people who would have fit in well at TJ who didn’t get in for one reason or another, and so I would guess that a substantially larger fraction of the applicant pool than those who are currently admitted could enjoy and do well at TJ.

    Secondly, as a few other commenters have pointed out, many applicants do currently go to TJ prep, with an estimated 50% of admits having gone to prep. I think the prep does significantly increase admissions odds. For instance, the front page of the website of one famous prep center advertises that 80% of the students who enroll in their 2 year prep program get in, as compared to the 15-20% odds of a random student. While there are many potential confounders here, like the parental wealth requisite to enroll in such a program leading to other educational opportunities earlier in life, the difference in admissions odds makes it seem nearly certain to me that there is some substantive advantage in prepping for the exam. The exam itself is not that difficult, and so I think is preppable– for instance, if I remember correctly, the math portion was comprised mainly of standard questions one might encounter in math class in middle school (nothing involving algebra or geometry, and nothing requiring difficult problem solving).

    I agree that an affirmative action policy is a better way to address problems with diversity than the removal of the test. And that the above two points could be better addressed by building more schools in the area similar to TJ and making the test less preppable. Indeed, Loudoun county (one of the counties sending people to TJ) has created an Academy of Science program for high-schoolers which seems somewhat similar. It would be great to see the other counties, especially Fairfax, do the same.

  54. bjuba Says:

    Unfortunately “TJ as we know it” has been gone a long time now. About a decade ago the admissions criteria were changed to, as I understand it, de-emphasize squishy things like recommendation letters (on the grounds that these might be biased) and instead focus on the test plus some kind of lottery. The results were measurably disastrous: the number of students requiring remedial math mushroomed from under 8% to over a third. (See: this WaPo editorial and this op-ed by Dr. Dell who taught AP physics at TJ for many years.) At the same time, the intended effect of those reforms, improving diversity, was not achieved. As far as I am aware, this is the same admissions policy that is currently in place. I don’t claim to be able to diagnose the root of the failure, but it does seem that preparing for the test as it exists today is really not the same thing as acquiring the mastery of subjects that the test is supposed to measure. Although the new proposal might be even worse, the current policy is not good and demonstrably better admissions policies were used in the past. In any case, the other lesson is that we should judge these policies based on their actual effects and not their intentions.

  55. Mike Says:

    Karger’s comments always take me back to the c*nt scene from the film “In Bruges”. Always good for a chuckle.

  56. Rahul Says:

    Aren’t we focusing on the wrong problem? i.e. Fix the bullying of the nerds in regular schools rather than just focus on sequestering the nerds.

    I mean, won’t this problem rear its ugly head with many other unique school kids? Basically kids like to pick on different kids. e.g. Will we establish schools for LGBTs etc.

    Historically, weren’t there separate schools for a lot of disabilities but over the decades we saw integration as a solution?

    Finally, isn’t there a lot to be said about exposing to stressors to create coping mechanisms? Post the gifted schools how would they be cocooned in college and thereafter too?

  57. Sandro Says:

    @Hyman Rosen #37:

    Are we really doing socially maladjusted kids a favor by planting them into an environment with other such kids?

    What makes you think all of these kids are socially maladjusted? Maybe some are, but certainly not all or probably even most, and the ones that aren’t can still bond over common interests with the ones that are, and so learn those skills.

    There’s also the possibility that bad high school experiences is what makes them maladjusted, and schools like this would prevent all of that suffering.

    As a final food for thought, babies are the most socially maladjusted of all humans. Are you suggesting there’s no value in letting babies play and socialize with each other? Of course there is, and in fact, babies actually learn these skills via interactive play. Kids that are “socially maladjusted” may have simply been denied this by harsh environment in which they were not permitted to participate in this interactive process, and so a more receptive environment might be exactly the right remedy.

  58. OhMyGoodness... Says:

    As I have written previously my entire educational experience from fifth grade to twelfth grade was a nightmare experience. Everyday at school I channeled Edvard Munch. The only thing that saved me from virtual hell in adult life was standardized testing. I personally know many others in similar circumstances. Some fought their way through it and some did not. The most mentally talented person that I have known personally did not. The ones that did survive their secondary education contributed very handsomely to society while the ones that did not are a sad loss to society and I mourn the result.

    As Dr Aaronson has noted it is a crime against reason to legislate against schools that strongly consider testing when so many schools are available that do not. Sometimes those with very high innate ability have unusual but innocent behaviors/attitudes with respect to US consensus standards. These differences are not valued but ignorantly attacked. The result of this can be a terrible loss society considering how rarely these abilities arise and how brutal social attacks can be on an adolescent personality.

  59. Greg Price Says:

    > Unfortunately “TJ as we know it” has been gone a long time now.

    bjuba #53: I remember that Dr. Dell op-ed, and I don’t doubt him that the effects of that change were bad. But I don’t think it helps to catastrophize. As far as I’m concerned, the sine qua non of “TJ as we knew it” was the number of other students around that I could really deeply engage with on math and science and tech. Even if there had been other students that needed remedial math, it would still very much have been TJ as I knew it, and something it’d be sad to lose.

    It’s hard to get a deep sense quickly from afar of whether that density of seriously enthusiastic math etc. students is still there, but here’s a quick check: I looked to see how TJ has done recently in ARML, a national math competition. Turns out they won in both 2018 and 2019.

    That means they had a team of 15 students that solved math problems better than teams of 15 drawn from the whole SF Bay Area, the whole metro Boston area, the whole of NYC, and many whole states. Moreover they didn’t get there without practice; if it’s like it was in my era, that meant about 40 students showing up every Thursday evening all spring, spending a couple of hours solving problems and then talking about the solutions, and most of them an hour or more before that hanging out and talking about more math. That alone is a pretty big chunk of what made TJ the TJ I knew. So I think they’re doing all right.

  60. anon Says:

    Rahul #55:

    Aren’t we focusing on the wrong problem? i.e. Fix the bullying of the nerds in regular schools rather than just focus on sequestering the nerds.

    I’m a fully grown adult but I’m too shy to write this under my real name. I’m a nerd and I wasn’t bullied and yet I suffered a lot during my teen years because I felt like a completely different species, but not in the “misunderstood teen phase” way.

    * I could get invited to play basketball but couldn’t do a layup to save my life.
    * During a teen music event, out of 30 people only I passed out from the flashing lights.
    * Despite understanding the social games, I couldn’t play them to save my life. I could engage with others, but I couldn’t force myself to like the activities, the pop culture and celebrities others use. The only exception was computer games.

    And then as I grew up and had the freedom to go to nerd events I found people like me. You can simplify this as discovering people that “share the same interests” or “having similar anxiety symptoms” but it wasn’t that. I could notice the same body tics, the same speech patterns, the same sensitivity to odors, lights and sounds. It’s a subconscious feeling of safety, being part of a group like you. Other eccentric subcultures, like groups of shy theatre and art people gave me that feeling as well, to a certain point.

    Now as an adult I have confidence not because I go to the gym (although that helps) or reading some shallow motivational quotes, but due to the experience with mimicking the non-nerdy people around me. Googling those feelings leads to autistic spectrum forums. Are me and all my nerd friends on the spectrum? I don’t know. What I know is that we all became well adjusted but it could’ve been done much faster if we weren’t forced to be something we were not. And me and my friends had relatively open-minded parents who did fight the school administration sometimes. God knows how bad nerds in harsher environments felt.

  61. Ethan Says:

    I have to agree with Rahul #56

    As it is probably obvious from reading my comments, I identify with the geek/nerd personality and yes, I was also bullied/tormented in high school.

    The first time I encountered people of the “geek/nerd tribe” was in college studying engineering. I haven’t looked back. I have very few memories from my K-12 years other than the great teachers I had that helped me along the way.

    Doweling in those years would be giving my tormentors too much power over me. I have developed my professional career far away from where I went to high school. I have reconnected with some of my former classmates and I don’t envy them.

    The best revenge in situations like this is success in life (understanding that the definition of success is very personal) and paying it forward when you run into fellow geek/nerds, particularly those who are early in their careers, who can benefit from your advice.

  62. Jason GL Says:

    Can anyone articulate a principle that explains how much meritocratic competition we should have in high schools? It seems obvious that “none” is the wrong answer, because it’s good to be able to sort students by effort and ability levels, so that they can connect with their intellectual peers, enjoy a curriculum that matches their abilities, and face appropriate rewards and punishments that will incentivize and appropriate level of effort. It also seems obvious that “all of it” is the wrong answer, because then you get a phony hellscape where teenagers spend 110% of their available energy on pretending to be well-adjusted, passionate, considerate uber-geniuses who are also lacrosse champions and first-chair bassoon players and wind up miserable and twisted.

    But I honestly don’t know how to identify the proper middle ground. How much competition is the right amount of competition? How would we know?

  63. Ethan Says:

    OhMyGoodness #58

    I recall that conversation. Each of us see the world based on our own experiences. I don’t do particularly well in standardized tests without preparation (ie, when I coach for them, I tend to do quite well) precisely because they are standard. and I struggle in questions that don’t have an obvious answer. Obviously 1+1 only admits one correct answer (2). However many of the trickiest multiple choice questions have official answers that tend to come naturally to people who are used to think in a very narrow and particular way (a way that is typically absorbed by those who are brought up in brainy upper middle class backgrounds; this is not to say that there aren’t people who think naturally that way being brought up differently, but the College Board’s own statistics are very clear that high scores in the SAT are correlated with high income of the family of the test taker).

    I am reflective and tend to think things carefully. I typically do well in exams that test hard abstract (not tricky!) mathematical and linguistic thinking. Over the years I have met people who do better than me without preparation in standardized testing and a pattern that I have observed is that these people generally do well in traditional careers that follow a rat race model (investment banking, management consulting, large corporations). They tend to do less well in (in fact the tend to shun) careers that involve high originality and risk taking to get ahead.

    So I would say that there is no universal way to detect “talent”, because talent comes in all shapes and forms. The best society can do -and American society does this much better than other societies I know of- is to offer as many opportunities as possible for people to shine at different stages of their personal journey. The notion of giving a test to people at a certain age that will “detect” whether these people “have it” as to focus society’s resources on them is what France does and what the Soviet Union used to do. I have never lived in the Soviet Union but given their ultimate collapse, I don’t think it’s a place we want to go. As the NY Times mentions in the case of France, the end result over there of their early selection methods is the creation of a de facto aristocracy that passes their privileges from one generation to the next. Hardly a model to imitate given France’s decades long economic stagnation.

  64. Ethan Says:

    Jason GL #62

    One of my intellectual contemporary heroes, Peter Thiel, has coined the mantra “competition is for losers”,

    https://www.amazon.com/Zero-to-One-audiobook/dp/B00M284NY2/

    https://www.youtube.com/watch?v=3Fx5Q8xGU8k

    His actual message is more nuanced than the mantra, but the mantra is a very catchy one. Given that he has done pretty well financially and otherwise in life, I think he knows something that policy makers should listen to!

  65. 90s TJ alum Says:

    Many of the things I was going to say have already been covered by JDewey #29, Recent TJ Alum #53, and bjuba #54, but I wanted to provide more information about the current state of the admissions discussion, as well as correct some misinformation that Scott seems to be working from.

    I’m a 90s alum who works in gifted education and has stayed connected with the TJ community. I’ve worked in test prep in Fairfax County and am part of the TJ Alumni Action Group despite disagreeing with several aspects of their approach (I’m not a fan of selecting all the seats via merit lottery).

    As others have mentioned, Scott’s 2012 visit was already during a time when TJ’s admissions were problematic, so it wasn’t like he was seeing the result of some perfectly meritocratic system. The classes of the early/mid 10s may actually have been during the worst stretch of time as far as admitting the most brilliant candidates goes, as some kids who were in the top 10 at Virginia’s state MathCounts competition were rejected. Despite that, Scott still found a core of kids who were enthusiastic and talented. As Greg Price said in #59, TJ won ARML recently. Clearly, TJ’s best kids are alright.

    From what I’ve observed, about 75% of the kids at TJ are pretty indistinguishable from the usual top-tier kid at the typical Fairfax County high school, and the other 25% of the kids you really couldn’t imagine being anywhere but TJ. This opinion was shared by many TJ teachers. Additionally, the teachers knew that when evaluating candidates, there was essentially no difference between the kid ranked #450 and kid #451, but there would be a significant difference between kid #450 and kid #50, and often a huge difference between kid #50 and kid #5 (and sometimes from kid #5 to kid #1!). So long as we get all of each year’s top 50 kids (and to a lesser extent all of the top 100-125 kids) who do things like win ARML, the school will remain great.

    The new proposal is that 100 kids will be selected for exceptional ability, while the other 400 slots will be filled via the lottery. I have some concerns whether the admissions office will be able to properly select those 100, but if they do a good job with that, the core of the school will remain and TJ will continue sending kids to MOP and winning ARMLs. In this setup, there would ideally be 400 candidates selected by lottery first, then the best 100 remaining candidates selected. I would prefer a 200/300 split instead of a 100/400 split, but 100/400 still solves most of the worries that the core of the school will be ripped out.

    Test prep is an even bigger problem than JDewey #29 implies. The poll in the alumni group is actually undercounting the percentage of recent TJ kids who did test prep. With the level of vitriol being poured upon test prep, there are a lot of kids who wouldn’t want to admit it in a FB group with thousands of members. As someone who briefly worked in the industry and was in frequent contact with TJ STEM teachers, the actual percentage of kids who did test prep in TJ’s classes of the 2010s is definitely above 75% and most likely above 80%. This problem was exacerbated by the low ceiling of the admissions test, making it both more difficult for the top kids to stand out and more gameable via test prep.

    I am also one who would prefer that the diversity issue be solved via the use of affirmative action for race, income, and geography, but since that is not a feasible solution, the mix of ability and lottery selection is likely the best we can get.

    Finally, be aware that Asra Nomani is not a reliable source. She’s leveraging her past WSJ experience to give herself an air of credibility. She has selectively misquoted TJAAG conversations at the level of “Aaronson, his mask finally off, conveys well-wishes to Donald Trump” on repeated occasions. A quick search for her interactions involving Georgetown professor Christine Fair will give you some idea of how she likes to operate.

  66. Fier Says:

    Ethan #33:

    Do you have any references on that Feynman number? Last I tried to chase it up I couldn’t find any good evidence.

    We do know that Feynman got the highest score in the country on the Putnam exam—while joining the team on short notice and not preparing. Seems fairly likely that other Feynmans could be captured by something like the SMPY process.

  67. Scott Says:

    90s TJ alum #65: Err, I guess it’s good that the “new” proposal throws a tiny bone in the direction of “exceptional ability,” but why such an utterly lopsided distribution? To maintain the character of the school, at the very least, why not do 250 and 250 rather than 100 and 400?

  68. Ethan Says:

    Fier 66

    Wikipedia has it https://en.wikipedia.org/wiki/Richard_Feynman , citing a book by https://en.wikipedia.org/wiki/James_Gleick .

    In general Richard Feynman was known for being “anti authoritarian” and “anti honors”. He explains himself here https://www.youtube.com/watch?v=f61KMw5zVhg . You can tell that if he became one of the top 10 physicists of all time according to living physicists https://www.caltech.edu/about/news/physics-world-poll-names-richard-feynman-one-10-greatest-physicists-all-time-368 it’s because of his idiosyncratic personality and his passion for understanding nature, something that is the antithesis of a personality that conforms to a “standard” as defined by others. How many people you have heard about to have renounced membership in the National Academy of Sciences ?

    As I have said, I have studied the life of several geniuses in detail and the pattern that they are not easily identified early in life as “genius” is there all the way back to Isaac Newton.

    In the modern business world, few would have said that Steve Jobs, given his mediocre academic performance in high school, would go on to revolutionize several industries (personal computer, music industry, the phone industry).

    Ditto of Elon Musk and the automotive / space industries. Despite having already been a successful entrepreneur with PayPal, he had to put up with people saying that SpaceX would be a failure, including famous astronauts from a previous era criticizing his efforts. Here you can watch him almost crying https://www.youtube.com/watch?v=8P8UKBAOfGo .

    The notion that you can use some sort of proxy exam/text to detect who “has it” and who “doesn’t have it” to focus all of society’s resources on those people who “have it” is plain social Darwinism; one of the remnants of eugenics in modern day America. Unfortunately bad ideas die hard and this one is one of them.

    What society should offer is opportunity to develop one’s talents -whatever they are-, not “denial of opportunity” early in life which is one of the side effects of the mindset behind talent searches.

    The first of such talent searches I am aware of (Lewis Terman’s) already proved that there is no formula to detect future Nobel Prize winners early in life. And if there is something that people should have learned from that whole experiment is that telling people “you don’t have it”, can create creatures like William Shockley who did have it but who was also so fond of proving everyone that he “had it” -after being told “you don’t have it”- that towards the end of his life he went on to revive the same discredited eugenics ideas behind the Holocaust only targeting Black Americans instead of German Jews.

  69. Allemaraiccire Says:

    @ 90s TJ alum #65:
    The school should exist for the students, not the other way around. I find your approach sickening.

    This country needs to get far more ruthlessly meritocratic (and yes, the merit measurement problem is a real problem, but you have to try) and provide all students with an education that fits their abilities, achievements and motivation, within bounds of practicality and economy. Other countries do this, and if the US fails to, it will eventually be left in the dust.

    All the extracurricular and athletic stuff should just be for fun. Let people do what they want, but totally disregard all that stuff for any admissions (like most countries, who are aghast at the way it’s done in the US).

    And for crying out loud, ditch all the demographic quota goals. Sure study the statistics, but for decisions, stick to pure merit, and let the chips fall where they may.

  70. Boaz Barak Says:

    I am struck by how thoughtful and nuanced are the comments from actual TJ alum here and in hacker news. It is unfortunate if this becomes another example in some national “culture wars”. I hope that the parents, alum, and educators come to a system that maintains the many positive aspects of the school and only improved on it.

    Honestly, ultra selective schools like that can be great in some aspects, but they are not a solution for the problem of bullying. There are plenty of “nerdy” people that could suffer from bullying but are not in the top percentile of such tests, and maybe even not academically strong enough for TJ regardless of the admission. Bullies don’t give you an SAT test.

  71. Scott Says:

    Boaz #70: I’m happy to serve as the radical meritocratic firebrand who holds open the Overton Window to make room for the actual TJ students and alums to try and hold the line for the same values I’m advocating in their more thoughtful and nuanced way. 😀

  72. Ethan Says:

    Fier #66

    Something else,

    “Seems fairly likely that other Feynmans could be captured by something like the SMPY process.”

    By my last count, nobody that society identifies as “genius” such as Einstein, Feynman, John von Neumann, Turing, Claude Shannon -and the list goes- has ever produced a standard test that would help identify people like them. From the few we have testimony, they didn’t seem to like exams much even when they did well on them.

    Every single form of “standard” test that allegedly tests someone’s mental abilities was designed by a psychologist (Lewis Terman in the case of IQ, Carl Brigham in the case of the SAT) claiming to know how to detect “mental ability”.

    Conflating “scoring high in standard tests” with “genius” is a talking point used by the people who benefit (economically or otherwise) from the existence of these tests. The genius themselves don’t seem to think highly of them. Terry Tao, who is someone the field of mathematics recognizes as “genius” and who was identified by the SMPY program, has very nuanced public views about these things here https://terrytao.wordpress.com/career-advice/does-one-have-to-be-a-genius-to-do-maths/ and here https://terrytao.wordpress.com/career-advice/advice-on-gifted-education/ (check also his comments).

    In Terry Tao’s case, his biggest gift was being born in a family -particularly his father- where his gifts were recognized early on, nourished and developed. Being identified by the SMPY program was a consequence of this, not a cause.

    So if you want to make your mind think along the lines of what Lewis Terman or Carl Brigham thought a “great mind” is, go for it. In my case, reading first person accounts of those who are “true geniuses” does the trick even if in many cases -William Shockley’s prominently but also others- I disagree with many of their proposals. Learning from the best minds necessarily means not giving standardized tests too much value when it comes to detecting who “has it”.

  73. Nilima Nigam Says:

    This shall be an experiment in optimizing some function of students, with active constraints involving a brand, scarcity (=access), and academic standards.

    A similar experiment has been conducted over the years in India, with the IITs. In the beginning, there was one (in Kharagpur, my alma mater). Over the decade, another 4 were created. Admission was through the famous joint entrance exam (JEE).

    My dad grew up in an impoverished part of the country, no electricity in the house, a school where the medium of instruction was in Hindi. His family knew nothing about the IITs or the exam. He filled out a form discarded by a friend, took the exam ‘cold’, and went to IIT. Fast forward 23 years, the number of students vying for entry was even bigger. Coaching classes had sprung up, but it was still possible for kids to get in without expensive coaching. My family didn’t know I intended to go, I didn’t attend any coaching classes, and I was just nerdy enough to find the exam fun. Of course growing up in a household where education was valued helped!

    When I graduated, there were a total of 5 IITs, and just around 2000 students entering per year.

    The brand was ‘world class’ – who could name any other engineering schools in India apart from the IITs? –
    and driven both by scarcity, and the ferocity of the JEE. The rank instantly became an ‘identifier’ in your generation. So, of course, graduates got recruited by well-paying firms, further establishing the brand.

    The country saw the problem of a strong brand and not enough access. To fix the problem, first the government rapidly converted (kind of overnight) many existing colleges into IITs, through the expedient of renaming them. But this didn’t fix access enough, and coaching was adding to the problem, so then the JEE was ‘reformed’ entirely.

    Since people couldn’t tell if the brand was about the student, the admission process, or the education at the IIT, it was decided to kind of split the difference. So now to get into _any_ engineering school, you take the JEE. To get into one of the (23!) IITs, you take the JEE-‘Advanced’. The history of the exam is convoluted beyond belief, and I’m glad to have graduated before this period. (Wikipedia has a reasonable timeline).
    n.

    The scarcity problem remains. Acceptance to the IITs is at under 1%. The brand is losing its sheen- IIT graduates are no longer (on average) being recruited into high-paying jobs/programs at the same rate as when my dad or I went.

    So – as a friendly suggestion to whoever is running this experiment in TJ – maybe start by renaming a whole bunch of highschools ‘TJ’. That would address the first part of the problem (scarcity). Then decide what it is that makes the TJ brand. The exam? The coaching? The educational standard?

    What are various well-meaning people optimizing here?

  74. Nilima Nigam Says:

    (Oh heavens, there are many formatting and other errors in my previous post. I apologize, and thank you again for hosting thoughtful discussions on many issues!)

  75. Douglas Knight Says:

    Fier 66:

    The source for Feynman’s IQ of 125 measured in high school is a speech he gave at his high school in 1965, the year he received the Nobel Prize. He says that that very day, visiting the school, he looked at his old records and found the measurement. Gleick claims to have seen the transcript of the speech from that year, although not the original records.

  76. Greg Price Says:

    > (serious question, not rhetorical) if there is any evidence at all (beyond the anecdotal) that selective high schools like this offer benefit to society at large that justify the inequality they create. It’s clear that they offer benefit to those who attend—but do those benefits spread?

    David Karger #25: I think this question deserves a serious answer, and it seems nobody’s yet tried to offer one. So here’s mine. I don’t have research studies to point to — I suspect that, like many important questions in public policy, it’d be hard to effectively answer that way and few people have tried — but perhaps I can give some useful reasoning.

    I think the benefits of a TJ education spread in the same way as the benefits of an education generally. A major reason we have public schools is that we all benefit from living in a society where most people around us have a level of skills gained through education. In a highly technological society like our own, where we’re surrounded by complex inventions and products of engineering, we similarly all benefit from the people who are designing and engineering those things having a high level of skill in math, science, and technology.

    For my own field, software: there is quite a lot of terrible software out there, and there’s also a lot of well-made software. The well-made stuff tends to be a lot more pleasant to use, or to have to rely on, than the bad stuff. From my experience seeing how it’s made, I know that very often the terrible engineering happens because there just aren’t enough people around with the skills to do it better. For the growing chunk of all our lives that is interactions with software, that means we’re better off if more of the people behind that software had a better education where they learned more. I think the same is true of electronics, materials, medical devices, and many other fields.

    For a comparison, you might consider: do colleges and universities offer benefit to society at large that justify the inequality they create? Would the world be better off without MIT — or perhaps without its undergraduate program, setting aside the research work done there? I think it would not; we’d lose the contributions of some highly skilled scientists and engineers, or lose their best work because they didn’t learn quite as much in isolation from each other as they could there.

  77. Almaca Says:

    Scott,

    Just want to say thank you for your post.

    I spent a decent chunk of my TJ experience reading Jacob’s TJIMO notes, hearing from people who went to your talk in ’12, and learning there’s infinitely more to learn.

    Though I also was bullied (even at TJ) – I’m still forever grateful for the experience and people I met there who taught me to learn with humility and the experience that guided me to niches at CMU/Berkeley.

    As such I greatly appreciate you bringing light to a bridge I hope future students can also walk over.

    Thanks

  78. Vampyricon Says:

    Not sure if I’m doing something wrong, but the follow button isn’t working for me.

  79. Shmi Says:

    Scott,

    This post reminded me of my high school, the same one Grigory Perelman attended, one of three in a city of 5 million with math and science enrichment, and how it felt like a breath of fresh air after the regular Soviet middle school educational mediocrity and ethnic bullying. As my grades were not perfect, I had to write an entrance exam to get one of the 10 remaining spaces, but that was the only obstacle. The high school years were some of the best times of my life, struggles with non-STEM subjects notwithstanding. How many nerds anywhere can say that about their high school? Our graduating class, now spread across three continents, and our home room teacher are still in touch, decades later. It’s sad to know that, because of misguided political and ideological reasons, so many years later and on the other end of the globe, many young and bright misfits will likely be deprived of a similar experience.

  80. Ethan Says:

    Nilima Nigam #73

    I think that the lesson from what you describe (while I didn’t know the full details, I am full aware of how competitive is to get a quality education in India from several Indian friends, one of them who also went to Kharagpur’s IIT) is that we should avoid at all cost creating a system like India’s that favors Darwinian competition over quality of the educational experience targeted at developing the students’ talents.

    While India’s uber competitive system necessarily produces graduates that have high “raw talent” and who are very competitive, on important metrics such as quality of original research and scholarship -which is the alleged reason for TJHS’ existence-, India’s institutions do not compete with the best institutions in the world that are less competitive in their admission procedures as measured by the admit rate. Check here http://www.shanghairanking.com/World-University-Rankings-2020/India.html . The so called “Shanghai ranking” is taken seriously by governments around the world as an indicator of world class scholarship, so much so that France created a brand new university federating some of its most prestigious schools, https://en.wikipedia.org/wiki/Paris-Saclay_University , to ensure it had one such institution in the top 20 (in this year’s edition, 15 of them are American, 3 are British, 1 is French and 1 is Swiss).

    America’s most prestigious schools “holistic admission” method had racist origins (it was born as an attempt to exclude Jews from having access to Harvard and other top American schools) but it’s hard to argue with the results in terms of the quality of the education their graduates receive. I know because I went to one of them for graduate school. Not to mention the production of world class scholarship as measured by Nobel, Fields Medals and other internationally recognized awards.

    If anything, I see the Indian system (and Iran’s, which follows a similar logic) as a refutation of blind reliance on results in a standardized test as a proxy for identifying top talent of the kind that will go on to produce world class scholarship.

  81. Greg Price Says:

    Ethan #33, #36, #72: You’ve cited Feynman several times here as if he stands against the value of having a selective high school. Did you know he went for college to MIT? Feynman very much got the benefit of spending school years with lots of other students who were serious about math and physics. To get there, he successfully got admitted at an institution that is more selective than TJ has ever been.

    I don’t know if anyone tried to identify the young Feynman as a future famous super-genius — but what’s much more to the point is that the MIT admissions office sure succeeded at identifying him as someone who would benefit from being around the other students at MIT and vice versa.

    Feynman’s disregard for authority would have fit right in at MIT in the period I knew it, and I rather suspect it fit in just as well when he was there too. Ditto his “idiosyncratic personality” — there are a lot of things that could mean, but pretty much all of them describe personalities found in abundance at MIT. The same is true of TJ and, I suspect, of the few high schools like it. With all of those and most of all with his “passion for understanding nature” (!), you’re really just making Scott’s case for him: Feynman was exactly the type of student who gets the most out of a place like TJ, or like MIT thereafter.

    But: I also want to push back on the premise of this whole subthread (which I know Scott started). I don’t think the value of places like TJ is primarily about future world-famous “geniuses” like Feynman and Turing and von Neumann. They don’t come along very often, not in the small geographic territory of a given magnet high school; and some of them, perhaps, can even take care of themselves. (Given how much Ramanujan figured out in near-total isolation from mathematical knowledge, imagine what he’d have done just with access to the Internet!)

    For the rest of us, mere mortals who are enthusiastically serious about math and science, to get to spend time with lots of other people with the same interests is life-changing — far more educational than any class. And as Terry Tao says in a blog post you linked, it’s us mortals, not a handful of “geniuses”, who primarily drive math forward. The same is true of science and engineering.

  82. Rahul Says:

    Scott #44

    “And yes, there are extremely well-developed routes at most schools to scout out and develop unusual athletic talent, and funnel it to the college level. Here they don’t mess around!”

    Don’t colleges operate a large portion of these “funnels”?

    Ergo what could universities be doing to run similar funnels for the nerd-talents (assuming we envy the athletic-funnels)

    Or is it that the mainstream college system is itself a nerd-talent funnel and the attention drawn by the athletic-funnels is only because they have to work outside the primary college intake system which is slanted more towards nerdy pursuits?

  83. B. Says:

    Scott #52: I was talking about the famous “comment #171”. Sorry for being unclear!

  84. Scott Says:

    B. #83: I mean, if I’d been able to spend my teenage years surrounded by others the same age who were as into math and CS as I was, that whole period of my life would have been completely different, so comment 171 would presumably never have been written or would’ve said different things if it had been, no?

  85. Doug Says:

    > (serious question, not rhetorical) if there is any evidence at all (beyond the anecdotal) that selective high schools like this offer benefit to society at large that justify the inequality they create. It’s clear that they offer benefit to those who attend—but do those benefits spread?

    That claim is totally backwards. The board is threatening the school because it is *too successful* at giving opportunities to Asian immigrants. Very little of the inequality in our society is due to high *income* – it’s true that elite educations do provide mobility into higher income brackets, but they don’t create the existence of those brackets (it certainly isn’t a school’s idea that bankers get paid more than teachers!) The more significant source of inequality though, I would say by far, is entrenched wealth, and sticky privilege. Having your elite schools based on zip code rather than an exam is exactly what perpetuates that.

  86. krishna sampath Says:

    Hi Scott,
    The schools is dismantled because of political reasons. The same party which you and I respect are doing this which is very unfortunate and tragic. I don’t see how dismantling the magnet schools like TJHS would change the race relations or uplifting the under presented minorities to a difference level. It’s like let us all bring down the middle class and rich to the poor class and achieve a class less society where in the end everyone will kill each other for survival and in the end only the capable will survive. My wife had an argument that not all magnet high school kids end up in good universities or doing ground breaking work. It’s true but it’s same for nurturing the talent in sports as well not all end up as NBA or NFL stars but it’s popular among the mass.
    the general complaint from the public is that one particular race manipulating the examination system(by coaching) is unfair but how about people failed at the entrance exam in spite of the coaching. The unknown benefits of putting smart people in one basket are many as some special skills will turn out and will serve the society. Few or more intellectuals can inspire a generation of talented or curious kids to do well in life. One of the few ways to identify such talents is bring them closer and let them butt themselves and lets watch some spectacular fire works. Govt should frame policies to identify and nurture the talented kids from under represented minorities and elevate them. Watering down an existing elite system is not the way to end racism. The TJHS alumni data will prove the importance of TJHS so as for other similar schools. I don’t want to get political but democrats are going very far to appease the minorities which will end in disappointment. They should learn when to stop being liberal. A line has to be drawn between crazy and liberal if such line exist.

  87. Ethan Says:

    Greg Price #81

    “You’ve cited Feynman several times here as if he stands against the value of having a selective high school.”

    Not at all! I am citing him more as an example of this,

    “I don’t think the value of places like TJ is primarily about future world-famous “geniuses” like Feynman and Turing and von Neumann”

    Meaning, that the whole premise for Scott’s thread is that changing TJHS’ admission procedures would imply that people like Feynman will be lost whereas I totally agree with what you seem to say that people like him find their way to raise to the top of their fields. For someone like Feynman, being born in the US is all that was needed for him to become Feynman. Had he been born in a different country that doesn’t value education and intellectual achievement as much as the United States, his chances of becoming “Feynman” would have been much lower. It’s no accident either that Einstein was born in Germany at a time when Germany was the world center in physics and working in the patent’s office was not an obstacle for him to develop the work for which he became Einstain. Going back to India’s IIT system. It’s ruthless in their admissions mechanism but it is not known for producing Nobel Prize or Fields Medals winners, let alone people like Feynman.

    The reason I have cited him often, instead of having quoted other geniuses, is that there is plenty of testimony of him in youtube. There isn’t as much from other geniuses whose work has stood the test of time as measured in decades.

    So if schools like TJHS are not about finding future Feynmans, then what are they all about? And what is this business of blind trust in standardized tests scores which we know are not the prime tool to find future Feynmans?

    I have offered a few ideas that I am happy to reiterate:

    – One issue is about how best allocate taxpayer educational dollars. While I am naturally sympathetic to the notion of spending in them in STEM education, one has to take the wider view that not everybody wants to pursue STEM careers. I never got to understand what’s so great about watching (or participating in) sports like NFL, MBA or MLB but it turns out that a lot of people disagree. Their views also matter even though geeks/nerds find them irrational. I certainly find the whole “mystique” around sports irrational. I am happy with ignoring the whole thing to focus on my own interests but that doesn’t mean that I fail to acknowledge that other people feel differently.

    – Another is some other point already discussed (related to the above but a bit different): is there a clear benefit of spending public dollars in a system that gives people who already have an upper hand in society (being born in families that value education to such extent that they spend their resources in living in a very expensive place that make their kids eligible to attend these schools)? Do we want our education system to become like India’s or Iran’s? And who would benefit from such a system. Would our society become poorer as a result of the extreme inequality in educational opportunities?

    – Then there is the conversation about the value of standardized tests as a proxy to detect great talent. The real world evidence doesn’t support the claim that they are a great tool to identify genius talent -something that was evident in the first use of the same by Lewis Terman. Why is that some people still claim that standardized tests measure something beyond “ability to do well in standardized tests”? Who benefits from this? To me it is very clear that the people who benefit from perpetuating the myth are the people who know how to game the system by coaching their kids to do well in the tests. In extreme cases you have rotten people like William Shockley who want to use these tests to justify why Black Americans deserve to suffer discrimination (in his view they were inherently inferior because IQ tests said so!).

    – And last but not least there is the whole theory of “early selection” as the best way to identify and develop great talent. I don’t believe in this for several reasons. First, what we know from the experiences of France and the Soviet Union is that these systems tend to create a de facto aristocracy that passes the lessons of how to get into and game the system from one generation to the next. These systems end up behaving more as “exclusion systems” not as systems that offer opportunity to people. One of the great distinctive features of the United States as a country is that it offers many opportunities to develop one’s talents at different stages of a person’s life. It understands that not to lives are the same and that therefore, using a “one size fits all” approach is antithetical to the country’s values. In the United States, it is possible to waste one’s childhood and teen years and then end up becoming https://en.wikipedia.org/wiki/Craig_Venter . It seems that some want to create a “Logan’s Run” world (that’s a reference to this 1976 movie https://en.wikipedia.org/wiki/Logan%27s_Run_(film) ). Without giving details for anonymity reasons, I have personally benefited from this. I was 30 when I first set foot in the top American school I ended up getting a STEM PhD from.

    People who feel that modifying TJHS’ admission procedures should relax. I would be way more worried if the lawsuit by Students for Fair Admissions against Harvard were to be successful in the end, turning the admission to America’s best and most prestigious schools into a formulaic admission like the one they have in India or Iran. Assuming this -and similar- lawsuits don’t alter the fundamental system of “holistic admissions” at America’s best schools, these kids, if they are truly meant to become STEM stars, have already won the biggest lottery there is on the planet: being born in the USA!

  88. Ethan Says:

    Greg Price #81

    Are you this Greg Price http://web.mit.edu/price/ ? If the answer is yes, I would love to emal you to give you the fuller picture.

  89. krishna sampath Says:

    I am also in support of more varying levels of magnet schools to attract different level of talents after a bench mark. This would ensure most of them end up in right school. If needed the system should consider other technical work like publications or math count score other than the scores of entrance examination.

  90. Scott Says:

    Ethan #87: Given the way the conversation developed, I regret ever having mentioned Turing and Feynman and von Neumann. Let’s grant that a few people are likely to become great scientists even if their “schooling” consists entirely of learning to forage for scraps in a homeless encampment. I don’t actually know if that’s true (as others pointed out, T, F, and vN all got fairly world-class educations), but suppose for the sake of argument that it was. Even then, it seems to me that there would still be thousands upon thousands of “triage cases”: kids who will become creative scientists with the right nurturing, but rich, frustrated corporate lawyers without it.

  91. Ethan Says:

    Scott #90

    “it seems to me that there would still be thousands upon thousands of “triage cases”: kids who will become creative scientists with the right nurturing, but rich, frustrated corporate lawyers without it”

    That’s an argument I can support, but these potential great scientists maybe come from a family background that doesn’t value education, let alone STEM education. Thus, a kid that has a GPA of 3.5 who comes from one such background is showing more promise than someone who comes from an upper middle class background who aces the admissions tests by coaching for it. In my view, these are the dynamics that TJHS is trying to address.

    And we should all be humble that no matter what mechanism you use to to do the selection, winners and losers are created and that we should keep the open mind of not denying future opportunities to those who, for whatever reason, lost in the first round (namely, admission to a magnet high school). We might be denying someone like Craig Venter opportunities otherwise.

  92. Wolf Says:

    “Score high enough on an entrance exam—something hard but totally within your control—and you could attend a school where, instead of the other kids either tormenting or ignoring you, they might teach you Lisp or the surreal number system.”

    High IQ people notoriously don’t get what it means to *not* have such a brain. The above certainly isn’t something within even the wider sense of “personal control” for many people.

    “The trouble is, TJHS without an entrance exam is no longer TJHS. More likely than not, such a place would simply converge to become another of the thousands of schools across the US where success is based on sports, networking, and popularity. And if by some miracle it avoided that fate, still it would no longer be available to most of the kids who‘d most need it.”

    This is written like only people who have special talents setting them apart from ordinary society can be non-jocks/non-social butterflies.

  93. a Says:

    I am asian but I greatly propound the idea all races should be equirepresented and that includes for immigration purposes. Right now the only mercy Trump has in my eyes is that he propounds greatly leveling the field in immigration bias. Or else no struggling legal immigrants would support Trump.

  94. Ethan Says:

    Wow, just announced that Roger Penrose won the Nobel Prize in Physics for his work on Black holes!!!!

    https://www.nobelprize.org/prizes/physics/2020/press-release/

  95. Scott Says:

    Ethan #94: Wonderful to see the prize go to such a promising up-and-comer in the theoretical physics world! 😀

    (Seriously, the singularity theorems are a cornerstone of the modern understanding of GR. So if our inability ever to observe a black-hole singularity isn’t disqualifying, as we learned today that it isn’t, then Penrose would’ve been a good pick at any point in the past half-century.)

  96. Ethan Says:

    Scott #95

    “Wonderful to see the prize go to such a promising up-and-comer in the theoretical physics world!”

    Indeed! But, related to the topic at hand, according to the press release,

    “In January 1965, ten years after Einstein’s death, Roger Penrose proved that black holes really can form and described them in detail; at their heart, black holes hide a singularity in which all the known laws of nature cease. His groundbreaking article is still regarded as the most important contribution to the general theory of relativity since Einstein.”

    By 1965, he was 34, an age considered “relatively old” by the believers in a “Logan’s Run” worldview!

    By my count, and I would love to know if other people know about this with certainty, this is the first Nobel Prize given to a theoretical physics result (even though it is supplemented with the people who provided empirical evidence for their existence), kind of the Nobel Prize that Einstein never got for his contributions for theoretical physics. Am I wrong?

  97. Ethan Says:

    Here is commentary by Peter Woit -for those who are unfamiliar with the name, he is a notorious critic of https://en.wikipedia.org/wiki/String_theory as a theory with the potential to unify quantum mechanics and general relativity due to the lack of testable predictions specific to string theory,

    https://www.math.columbia.edu/~woit/wordpress/?p=12009

    It would seem that indeed, it’s the first Nobel Prize to work in theoretical physics.

  98. Doug Says:

    > Penrose would’ve been a good pick at any point in the past half-century.

    Maybe they could have even made the leap while Hawking was alive to share it…

  99. Scott Says:

    Ethan #97:

      It would seem that indeed, it’s the first Nobel Prize to work in theoretical physics.

    Of course it isn’t! Planck, Einstein, Bohr, Heisenberg, Schrödinger, Dirac, Born, Landau, Feynman, Schwinger, Tomonaga, Gell-Mann, Glashow, Salam, Weinberg, Wilson, ‘t Hooft, Gross, Politzer, Wilczek, Nambu, Englert, Higgs, others … all won for theoretical work. (In Einstein’s case, famously, not for his greatest work, but for other theoretical work that also deserved the prize.)

    What might be true, though, is that no other physics Nobel Prize has ever been awarded for anything as inherently remote from “direct experimental confirmation” as the formation of a black hole singularity. On the other hand,

    (1) I always found that an artificial restriction,

    (2) black holes are now empirically known to exist to exceedingly high confidence, and

    (3) while the most astonishing features of black holes aren’t directly observable, many of those features (with thanks to Penrose) are theorems of GR, meaning that either they’re there or else GR is badly wrong … clearly a Nobel-worthy disjunction! 🙂

  100. Ethan Says:

    Scott #99

    Thank you for the clarification!

    Still it’s very exciting to see a Nobel Prize in Physics to Roger Penrose for reasons people who have read my comments in other threads I am sure understand : – ). It has made my day!

  101. Jo Says:

    First of all, let me say that I find the general level of the messages in this discussion quite high, thanks everyone.

    Let me add something about bullying and/or social isolation. I grew up in France so the system might be quite different from the US. I was a “nerd” and top of the class, not bullied but felt quite isolated until approx. 16yo. At this time, I found common interest with people around me, namely around videogames. Common interest in science would be found much later. Anyway.
    What I realized much later was how crushingly difficult must school have been for most of my classmates, not because of the social interactions but because of the system they were in. 30 hours per week they were told how they were worthless failures (I’m exaggerating but it boils down to this). And then 2 hours per week (in sports) they were valued and told they were adequate or even good. No wonder they liked sports!
    Unfortunately for nerds, it’s during sports (and recess but here the dynamics are not the same, although at some age social status at recess is correlated to physical abilities) … it’s during sports/phys ed that the social interactions are the most developed. The 2 hours per week when the nerd feels inadequate.
    It may be good to have math/science education organized more around social interactions (such as team projects) … Reminiscing, the few group projects that we did were a very good experience, both for “the nerd” and for other non-nerdy types.
    With group projects or other means, the “nerds” would learn to interact socially, learn to lead, to respect different abilities, the “muscles” would learn to value and respect the nerds and see what everyone can bring apart from “A pluses” and “Fs”. More socializing, more understanding and valuing of differences, less bullying, everybody’s happy 😉

  102. asdf Says:

    Ethan #96: iirc, Paul Dirac won the Nobel in the 1930’s for predicting the existence of the positron.

    On the other front: it looks more and more like President Trump’s supporters who want him to be president-for-life, may actually get their wish :(.

  103. PTT Says:

    Dear Scott,

    can you confirm our hunch that despite authoring the above, you still believe that STEM will fare better under Biden than under Trump?

    Cheers,
    -PTT

  104. Raoul Ohio Says:

    Pardon for switching from US to Finnish Highschools. Does anyone have an opinion on Lindgren and Liukkonen’s view of Quantum Mechanics:

    https://phys.org/news/2020-10-quantum-mechanics-reality-person.html

    If this theory holds water, ….

  105. Scott Says:

    PTT #103: Of course science will fare better under Biden! If nothing else, we’ll hopefully be able to recruit foreign students, postdocs, and faculty again, without the federal government constantly dreaming up new ways to revoke their visas.

    More to the point, though, what the hell has Trump done to protect magnet schools?!? I mean, think about it. As I’m far from the first to notice, not only did wokeism in the US not recede under Trump, it massively accelerated. A central reason, it seems to me, is that Trump conveniently offers himself as the living proof and embodiment of straight white male villainy and everything else that the woke believe about the world. The Trumpists and the woke are ironic allies, with each providing the other’s ultimate justification. Which doesn’t mean that they’re equivalent in their power, their ill intentions, or anything else—but as for me, I’d rather put a pox on both houses and go back to Enlightenment liberalism! And while Biden is not the ideal candidate, compared to Trump he’s Spinoza, Franklin, Paine, and Mill all rolled into one.

  106. PTT Says:

    Did I call that one or what? Don’t know if you read my linked post, but I had predicted your answer would have something to do with immigration. Also, your point that wokeism increased under Trump is a Kafkaesque. It’ right up there with “Your event can’t be held due to threats of violence.” “Violence by whom?” “Violence by us.”

    You ask, “what the hell has Trump done to protect magnet schools?” Well, nothing. Magnet schools didn’t need protecting, they needed *being left alone* — which Trump was doing a fine job of. Biden-Harris & co sure don’t look like the kind of folks willing to leave much of anything alone…

  107. PTT Says:

    Oh, and Enlightenment liberalism is very much off the table for any foreseeable future — I hope you don’t have any illusions at least about that!

  108. Gerard Says:

    Scott #105

    > The Trumpists and the woke are ironic allies, with each providing the other’s ultimate justification.

    I suspect your observation is of more general validity than that which you are claiming for it. It seems like the best way to advance one extreme is often for its opposite to gain ground. I wonder if the Nazis would have succeeded in coming to power if it weren’t for the prevalence of communist sympathizers in the Weimar Republic.

  109. MJ Says:

    Scott #90: I regret having mentioned Turing and Feynman and von Neumann.”
    I agree that von Neumann wouldn’t have needed a magnet school to learn mathematics but in modern times he would be cancelled for having ultraconservative political beliefs. The same goes for his fellow formulators of quantum mechanics Heisenberg and Jordan.

  110. fltc Says:

    I wonder how many Indian students at this school come from Brahmin caste families whose admonitions for fair play and against hereditary white “privilege” might be more than a little ironic. Since you favor quotas would you support reserving some of their spots for untouchables?

  111. Scott Says:

    fltc #110: They do do affirmative action for untouchables in India. In the US, though, knowledge of this stuff (and ability to verify it) is minimal enough that, if you tried such a thing, I fear thousands of Brahmins would suddenly discover their untouchable ancestry! 😀

  112. Ethan Says:

    fltc #110

    That’s a good point. I read a few months ago about this case https://www.cnn.com/2020/07/01/tech/cisco-lawsuit-caste-discrimination/index.html at Cisco dealing with caste discrimination in the US. Aiming at lower the percentage of Asian students -if a majority of those 75% Asian students are of Indian heritage- might be good for the Asian students themselves!

    I think it’s great for the US to have so many brilliant people of Indian ancestry come to the US -although not all Indians coming to the US as H1B immigrants belong in the “uber bright” group as it is well documented http://heather.cs.ucdavis.edu/h1b.html .

    At the same time, as I think it is clear from my comments, I don’t think we should adopt the Darwinian selection methods that are normal in higher education in India. The US higher education system plays in a league of its own and I think that its main virtue is its flexibility both in terms of options available (with different colleges using different admission methodologies aligned with their respective missions) as well as the fact that it’s open to take brilliant people in at different stages of their personal journeys. In the US, cases like Craig Venter’s at the graduate level are the norm, not the exception, based on what I have seen in my own experience. This creates the impression in society that with hard work and motivation, anyone can get into a top school even if they didn’t get in as undergraduates right after high school which is a very good thing for American society compared to the society that results from telling people at the age of 18 “you don’t have it, and there is nothing you can do to improve your chances in life”, which is essentially what the Soviet system was.

    As we know from the information released as part of the lawsuit against Harvard on the latter’s affirmative action policies, standardized testing plays some role but it’s not the most important role https://blog.prepscholar.com/harvard-asian-admissions-lawsuit-application-strategy

    “This confirms what we already know – getting perfect grades and test scores is not impressive enough to be world-class in academics.”

    It’s surely expensive to maintain such a system, but it’s difficult to argue with its results!

  113. JimV Says:

    Raoul Ohio at #104,

    My reaction is that I don’t personally know of anyone but Deepak Chopra (who has no standing) who believes that quantum effects are due to specifically-conscious observation. There have been a number of experiments which give evidence against this belief, such that it is typically given the disparaging name “quantum woo”.

    For example the two-slit experiment has been done with C60 molecules, over a range of temperatures. At temperatures high enough for the molecules to emit thermal photons, interference patterns gradually disappear with increasing temperature. The photons give information which could be used to determine the molecules trajectories, but that information was not measured by the experimenters, and therefore not known by them or any other known conscious entity.

    Perhaps there is something new in the analysis that was referred to in the link you gave, but what it was was not clear to me in the article. The lack of impact of consciousness on quantum results was old news to me.

    On the topic of this post, I don’t feel I know enough to have a good opinion. I went to a very small rural school (19 students in my graduating class, one of them held over from the previous year). Our chemistry “lab” had one bunsen burner and I don’t recall ever seeing it lit. I got good scores on on the SAT and National Merit tests (with no test prep other than ordinary school work). I managed to avoid most of the bullies and only got beaten up a few times. Sometimes another student would come to my aid–there were some good people there as well as some bullies. I usually didn’t go to social events such as school dances, but did try out for sports. In hindsight I don’t think the sports coach ever taught much in the way of tactics or strategy. The one time he did (if you’re the only one back guarding the basket against a fast break, don’t try to pick up the oncoming guy with ball out at the foul line where he can go around you, stay under the basket) I did exactly what he said and I blocked the layup, a few plays later. That was one of my best on-court moments. If he had given me more specific instructions like that I might have done better. The attitude seemed to be that by high school everybody should know the rules and moves. I never played early-youth sports and didn’t.

    The only rule I have that might apply to the post topic is trial-and-error. That’s how everything progresses, but for progress to be made you have to keep track of how well the current system is working and be prepared to tweak it for improvements. Some things you try aren’t going to work, but you should keep trying as long as improvement seems possible.

  114. JimV Says:

    P.S. A side anecdote on high school bullies. I was a passenger in a car in Schenectady, New York, driven by a friend who grew up there. As we passed a parked police car he noticed the driver, cursed, and exclaimed, “Every bully I knew in high school is now in the Schenectady Police Force!” (For the ring-wing trolls here, the friend was not a left-wing pinko, but a devoted listener to Rush Limbaugh’s radio show. Still a good guy, though. Haven’t seen him in years, hope he isn’t fooled by Trump.)

    There should be some sort of psychological test for bullies to keep them off police forces, in my opinion. Probably it won’t be perfect and have errors of the first and second kinds, but would be worth a try, I think.

  115. Elizabeth Says:

    About a decade ago, Warren Siegel joked about a Nobel prize singularity in theoretical physics;  Although prizes are regularly awarded for theory, the timing of the work is only allowed to asymptotically approach 1973.   Since then we’ve seen prizes for the Higgs boson (work done in 1964) , KT topological phase transitions (work done in 1972), Gravitational Waves (Ligo started in the 60s, Misner/Thorne/Wheeler textbook published in 1973), and now Penrose for work done in the 60s.  But Nobels are not awarded posthumously, so how will they escape the singularity? Maybe by using diversity quotas and gradually changing the meaning of the award (ironically in line with the main topic of this thread).

  116. yme Says:

    Ethan #100: “Still it’s very exciting to see a Nobel Prize in Physics to Roger Penrose for reasons people who have read my comments in other threads I am sure understand : – ).”

    Well, now, he didn’t get the prize for that. 🙂

  117. Ethan Says:

    yme #116

    I know :-).

    And to be clear, Roger Pernrose’s Nobel Prize for his work on black holes has no bearing on the other stuff, it’s more like until today the other side -particularly low information defenders of the other point of view who hold dystopian views of AI- could say “we have Elon Musk and his billions on our side, who do you have on your side?”. Now we can say “well, we have Mr Penrose, Nobel Prize winner in physics!”

  118. 90s TJ alum Says:

    Scott #67:

    It’s more than a tiny bone. As I wrote earlier, it will largely preserve the benefits reaped by the most talented and interested kids like Greg and Nathan. I’d be more comfortable if it were 200:300, or even 250:250, but politically, 100:400 is much more feasible and I’d settle for taking the first 400 via lottery then selecting the 100 best remaining candidates rather than risk the all-lottery proponents winning (which is likely if the only alternatives were “lottery” vs. “non-lottery”).

    Another thing I’d like to emphasize is that over the classes of the late-2000s and early/mid-2010s, the test did not do a very good job of sorting students by STEM ability and talent. Because of this, many of the cries of “meritocracy” via the test really don’t carry much weight. A poorly designed low-ceiling test is barely better than no test at all- not only does it lead to a very poor sorting, it lets many people fool themselves into thinking that the test is in fact doing a good job sorting. Again, as I mentioned previously, it’s arguable that the student population Scott saw in 2012 was already the result of what a fan of “merit” would consider a poor admissions process. Some of the old tests failed to discriminate between students in the top 5%, leading to what was pretty close to a lottery (though with a higher cutoff than what’s currently proposed). The more recent tests have been slightly better, but not great. That’s why we need a very difficult test, preferably one that tests ingenuity and problem solving in addition to just the mechanics of Algebra 1 and Prealgebra.

    There’s actually one area of extracurricular that correlates highly with doing well at TJ. I’d be very comfortable auto-accepting any kid who qualified for state MathCounts or scored top 1% in the AMC 8. Though the best math team kids are no doubt talented and good problem solvers, they’re also used to failing productively. Even the top competitors have to learn to deal with failure and to improve themselves afterwards. It builds resiliency from an early age.

    Finally, people have generally been talking about some abstract “merit” as if there was a single definition and that we could quantify it to sufficient granularity to determine the 450 or 500 “best” candidates. We have neither sufficient rigor for defining “merit” in a universal way, nor a method of measuring merit that can reliably distinguish between kid #500 and kid #501 in any reasonable amount of time. Even the differences between kid #400 and kid #600 are very small. It’s as if every basketball player in the world was suddenly unattached and the 30 NBA teams got to redraft their 15-man rosters from scratch. The top players like Giannis, Lebron, Steph- those are obvious early picks. The solid starters in the 75-125 range like Tristan Thompson and Goran Dragic will certainly be picked as well, regardless of what kind of “merit” the front offices are evaluating on. But by the time teams are filling out their last 4-5 roster slots, they’ll be drafting based on specific role and who goes at #450 vs. who gets left out at #451 will be based on whether that team prefers a rim protector with deficient offensive skills or an undersized shooter that’s a defensive liability.

  119. yme Says:

    Here’s an interview of Penrose by Lex Fridman, which I guess YouTube recommended to me because I watched an interview by him of Donald Knuth that Ethan suggested in another thread. I enjoyed Knuth’s and now I’m enjoying Penrose’s—I’m still in the middle of his—so, thanks, Ethan.

  120. Nilima Nigam Says:

    Ethan (#80),
    I didn’t assert that optimizing research excellence was part of the objective at IIT. It wasn’t, at least not in its original conception.

    The IITs were explicitly conceived as a way to train engineers in a very young country, to a high level. IIT Kharagpur’s initial classes were in the what was originally the Hijli detention camp – a facility for loud-mouthed Indians who were protesting a British colonial government. So, in a truly profound way, the IITs were born of the aspirations of a people who were oppressed in rather hideous ways.

    Even today, while research is something that’s important at the IITs, it is resolutely not the raison d’etre. So yes, it’s not in the top-ranked lists. The country is not a wealthy one. There aren’t large granting agencies. Maybe a good question is: why have the IITs *dropped* in the rankings since the vast expansion? The original IITs would manage to have at least one in the top 100 in the ’90s and 2000s (this predates the ARWU rankings).

    What you describe as Darwinian competition is thus because of sheer numbers – reflecting in many ways a success story. When the Brits exited, the population of India was 361 million. By 2017, thanks to dramatic decreases in infant mortality and increases in food supply (uneven, but still), the population was 1.3 billion. Imagine what that looks like, and try to conceive of how challenging it is to get jobs, admissions, heck – even a taxi in Mumbai rush hour. ‘Darwinian’ is a feature, not a design choice. And for me (and others like me), conceiving of a career in science in India seemed the height of ‘mooching’ in a way. There’s nothing quite like walking by slums every day, to realise that the government should really be devoting their money there, not to your own daydreams of applied mathematics. Daydreams of applied mathematics, by geeky immigrant brown Indian girls, are best supported in the USA. (Were, and thank you, and what the hell did you guys do down South?!)

    I get it – I totally get trying to fix something that seems wrong. The reason to bring up the IITs is to provide, if you will, an example of why it may be beneficial to figure out precisely what is the desired outcome. Otherwise one is putzing around with experiments for years, generating confusion. These experiments involve real humans. Take it from someone who has watched a few such experiments.

    What is the objective function? Is it to have lots of kids have the TJ brand? There are easy ways to achieve that. Is it to have the ‘credential signalling’ that comes with a tough admission exam? That’s a different optimization. Is it to have a really tough curriculum which demands a lot? That’s yet a different thing. Some combination of these? Is it to have, as some describe, a place where self-identified nerds go to escape the tedious stuff of other high schools? Is it a place where immigrants with aspirations for their kids dream of sending them?

    What are the constraints? Why isn’t there a move, say, to create a program as academically challenging as TJ’s in another existing public school, thereby easing the issue of access? Why isn’t there a call to create this as a default curriculum (or at least some track) in the entire country? Is this a matter of funding?

  121. Rahul Says:

    Scott#111 “They do do affirmative action for untouchables in India.”

    That’s a gross understatement nowhere close to the American version of affirmative action. What India has is a stupid system where 30% to 50% of the available positions are reserved for particular groups (castes, but also other groups such as women, religious minorities etc.)

    This applies to universities, government jobs, medical hospitals and almost all other governmental institutions. It’s a horrible system, perhaps the perfect antithesis of a meritocracy and something that I would advise no other nation should replicate.

    As compared to the Indian monstrosity of reservations the American affirmative action system is docile and totally innocuous.

  122. Arul Says:

    Nobel included the clause ‘greatest benefit on mankind’ which precluded mathematics. I do not see how blackholes benefit humanity. If it indeed did in some way then why not include other areas such as art and of course mathematics?

  123. Anon Says:

    I lean towards an entrance-exam specific policy, with as much possible done to level the playing field prior to the exam. However, if there are quotas at TJ, there is a case for it to be biased *against* kids who enter solely by merit. I have watched how the minority of kids at IIT Bombay who entered by quota were teased by sections of the majority who entered through the entrance exam. Although I never perpetrated it (and things may have changed for the better now), I walked away silently when I noticed it (to my shame). Subjecting even younger kids to that at TJ would be worse.

  124. Ethan Says:

    yme #119

    Thank you for you kind comment. Yes, Lex Fridman has interviewed quite a few interesting people over the last couple of years. I watched the one to Donald Knuth entirely and I watched excerpts of Roger Penrose’s (mostly because I had already watched Penrose’s material before whereas I haven’t watched as much from Donald Knuth’s). The first time I recall watching Penrose making the argument -I was somehow familiar with it earlier from The Emperor’s New Mind but I had not watched him make it personally-was watching this talk in youtube around 10 years ago shortly after it was published https://www.youtube.com/watch?v=f477FnTe1M0 . Someone in the audience asked him during the Q&A about https://www.scientificamerican.com/article/why-the-human-brain-project-went-wrong-and-how-to-fix-it/ saying that he was targeting building a computer that replicated the human brain in 10 years (I think the technical name for this kind of computer is a “cognitive computer”). Penrose replied sarcastically something along the lines “it’s always 10 years, isn’t it”.

    One of the great things of our current era -despite all the problems created by social media- is that services like youtube allow you to listen to great minds unfiltered and unedited. It has never been easier to be exposed to them and to form your own opinions as opposed to absorbing the opinions of the middle man.

  125. Gerard Says:

    @Arul

    There is a Nobel Prize in Literature so the arts are not entirely excluded.

    I’m not sure how the initial list of prizes was decided but my guess is that it reflected the biases either of Alfred Nobel or of those he left with the task of organizing them. As I understand it the Abel prize was created fairly recently to make up for the absence of a Nobel in mathematics.

  126. Ethan Says:

    Nilima Nigam #120

    You are covering two distinct topics: the issue of IIT’s mission/objective function and the matter of TJHS that Scott brought to our attention.

    I won’t say much about the IIT mission because, believe it or not, although I have several good friends who are originally from India and who have attended IITs, I am not from there myself :-). All I will say is that “being good for India” and “being good for the United States” are two very different things and I reiterate that the Darwinian approach with a government defined objective function is antithetical to the United States’ values of giving everyone a chance to achieve his/her personal dream, whichever that personal dream might be. And certainly, don’t let government have an input in or interfere with that personal dream!

    On the issues you bring about TJHS, I think that these are very good questions,

    “What is the objective function? Is it to have lots of kids have the TJ brand? There are easy ways to achieve that. Is it to have the ‘credential signalling’ that comes with a tough admission exam? That’s a different optimization. Is it to have a really tough curriculum which demands a lot? That’s yet a different thing. Some combination of these? Is it to have, as some describe, a place where self-identified nerds go to escape the tedious stuff of other high schools? Is it a place where immigrants with aspirations for their kids dream of sending them? ”

    I asked similar questions earlier. And if I am blunt, based on the thoughts expressed by several alumni, the defenders of the current system seem to be defending the subsidy by the taxpayer of a credential signaling experience that allows people to get into some of America’s best schools. While undoubtedly that’s great for the people who go to TJHS thanks to their parents vision and payed by Fairfax County’s tax payers, is that what is best for these same taxpayers as a whole? Is this the best for American society as a whole? As I said, is someone is truly a STEM star, the opportunities in the USA to develop one’s talents are so immense that changing TJHS admission criteria won’t be an obstacle for that person to achieve those dreams.

    Now if the goal is to give people opportunities from underrepresented backgrounds, it seems the new proposal is reasonable. Let me give some additional information about me. Both my parents were uneducated. While both understood that going to college was important, and invested the little they had in ensuring that me and my siblings could go to college, they lacked the sophistication to understand the different options when it came to high school options or even college options. For them “doing well in school” had a different meaning from parents who are educated. As a result, while I was a good student and I had decent grades, I wasn’t taught all the tricks about maximizing one’s GPA and ways to game the system. It seems to me that the new proposal has people like my younger self in mind all while avoiding the stigma associated with being a beneficiary of affirmative action.

    I enjoy working on challenging, real life stuff not on proxies of the same. To this day I hate exams precisely because exams are someone else’s view of what’s important, not mine. To do well in life I had to learn to “game the system” but it’s not something I enjoy doing whereas some people have been brought up with the notion that “life is a game” since they were little and thus they tend to do well “gaming systems”. In a world with unlimited resources, there would be schools for both types of people payed by the public. We don’t live in such a world and therefore the notion that we need to build a public system for the benefit of “gamers” who see attendance to schools like TJHS as a checkbox in building their MIT admission packets seems a bit preposterous to me. It’s giving even more advantages to people who have already won the birth lottery and doing so payed by society as a whole!

  127. fred Says:

    Scott #111

    https://www.marketwatch.com/story/california-will-require-public-companies-to-have-diverse-boards-11601504807

    “California will require publicly traded companies headquartered in the state to add minority board members by the end of 2021 under a first-in-the-nation bill signed into law by Gov. Gavin Newsom on Wednesday.”

  128. fred Says:

    fred #127

    Of course the “loophole” is that *anyone* can literally claim to be “bisexual”…

  129. Scott Says:

    fred #128: “Bisexual under the right circumstances, but only ever acted on one of them”?

  130. Arul Says:

    Actually that is right indeed. How does Nobel prize in literature provide greatest benefit to humankind when in fact reaching 0.1% of humankind with any kind of literary work is a dire difficulty? Perhaps Hollywood scripts?

  131. Scott Says:

    Arul #122:

      Nobel included the clause ‘greatest benefit on mankind’ which precluded mathematics. I do not see how blackholes benefit humanity.

    Whatever else one thinks, that horse bolted from the barn almost a century ago. Nobel Prizes have regularly been awarded for high-energy particle physics and other topics very far removed from any known application (and this isn’t even the first Nobel for black holes, or related objects like neutron stars). Not only that, but today there’s mathematical work that seems enormously closer than particle physics to Nobel’s original practical ideal (to take just one obvious example, public-key crypto), which can’t win a Nobel because it’s outside the scope! Much like with Constitutional law, one needs to look at the interaction between the “original intent,” what was actually written down, changed circumstances that might make the original intent fail to match what was written, and how the rules have actually been interpreted over the generations…

  132. Gerard Says:

    @Ethan

    You’ve mentioned France and the former Soviet Union a few times as having meritocratic systems that underperform the US. But in the specific case of Fields Medals I don’t think the data support your claims.

    France and Russia/former Soviet Union are in second and third place in the total number of Fields Medals won and France is first in medals per capita, except for a couple of countries with very small populations and only one award, making their statistics dubious.

    https://stats.areppim.com/stats/stats_fieldsxcapita.htm

  133. Ethan Says:

    Gerard #132

    You are cherry picking a particular metric that doesn’t contradict my claims.

    Besides, in the French case you have no other than a French Fields Medal winner leading teh charge the charge, Laurent Lafforgue, against the French education system as a system that offers equal opportunity https://translate.google.com/translate?sl=fr&tl=en&u=http%3A%2F%2Fwww.en-aparte.com%2F2013%2F06%2F28%2Flaurent-lafforgue-mathematicien-leducation-nationale-est-devenue-un-vaste-mensonge%2F

    “Laurent Lafforgue, mathematician: “National Education has become a vast lie” ”

    “It sounds silly but you should know that at university, it is not uncommon for students not even know how to add two fractions. There is a very big contrast between the bulk of the students and a very small elite who benefit from French mathematical research which is of a very good level. Among today’s young mathematicians, a significant proportion are the sons or daughters of mathematicians. Unless I am mistaken, when I was at the ENS, there was none. Why ? Because, with school deteriorating, the family environment has become essential for learning!”

    That’s the very definition of an aristocracy even if the French system is good at producing Fields Medals winners. The two things are not incompatible. Ditto of the former Soviet Union.

  134. fred Says:

    Scott #129

    Did I ever mention that I find you kinda cute?

  135. Gerard Says:

    Ethan #133

    A priori the number of Fields Medal winners is no more a cherry picked metric with regard to an education system than is economic productivity. Arguably it is less so because while there are a great many political and social choices that can affect a country’s economic performance, one would expect its education system to be more directly related to how well it performs in academic matters, such as the Fields Medal.

    As for your quote from Laurent Lafforgue, you are attributing a position to him that is nearly the opposite of what he is stating. The entire interview consists of him deploring the degraded state of French education today relative to what it was when he was in school. But this rant is almost entirely about teaching methods and philosophy rather than exams or how students are evaluated. Furthermore the majority of his complaints concern elementary education, not secondary/post-secondary. His point about many ENS students now being children of mathematicians is that he believes that mathematics is now so poorly taught in French schools, starting at the elementary level, that one must now have a mathematician parent to have a good chance of actually learning the subject. That is his explanation for the loss of equal opportunity in France, not the existence of elite schools or competitive entrance examinations (one of which he attended).

  136. Ethan Says:

    Gerard #135

    First of all, you are the one who brought the metric of Fields Medals, not me. All I am saying is that one cannot conflate production of Fields Medals with having an egalitarian higher education system.

    France clearly doesn’t have one such egalitarian system. It’s well documented that it has a two tier system: one for and gamed by the elite made of schools like ENS, Ecole Polytecnique, ENA (most French presidents come from here), CentraleSupélec (the union of two former schools Centrale and Supélec), Arts et Métiers ParisTech, HEC, etc and then the general higher education system made of universities open to anyone who passes an exam known as https://en.wikipedia.org/wiki/Baccalaur%C3%A9at that has a very high passing rate. Interested readers can read more about this topic in references like this https://www.nytimes.com/2010/10/11/education/11iht-educLede11.html .

    Studies show that not only at ENS, but France’s most prestigious schools, are de facto mechanisms by which the French elite passes its privileges from one generation to the next. So much so that French president Macron, a product of ENA, wants to abolish the school https://www.timeshighereducation.com/features/will-macrons-move-against-his-alma-mater-make-frances-he-system-fairer . Good luck with that! As it it happens with any powerful incumbent elite, they won’t go away quietly into the night without a fight!

    I quoted Laurent Lafforgue only to state a qualified opinion that ENS has become a de facto aristocracy. He gives his reasons for why that’s the case but I disagree with those reasons. Fundamentally what has happened in France with their Grandes Ecoles system is what in the process of happening in the US with undergraduate admissions to Ivy League and similar schools for similar reasons https://www.nytimes.com/2019/09/09/arts/meritocracy-trap-daniel-markovits.html . Only, and this is the relevant part for this conversation, in France, the aristocracy has convinced the government to cover the entire bill for this system. In the US, attending an Ivy League school or MIT for undergraduate is not free. In France, to my last count, there was a time when the French government paid not only the fees of those who attended ENS or Polytechnique, but it also paid their students a salary. I don’t know if that’s still the case.

    It seems to me that at the core of the debate on admission procedures to TJHS there is the question of how best allocate public dollars and who benefits from the old system vs the new system being proposed. It’s clear that those Americans who know how to game the system, in the same way some French mathematicians game the French system to have their kids attend ENS, benefit from having a free public school that increases the chances of having their kids admitted to MIT and similar schools. It isn’t that clear that the taxpayers of Fairfax County or American society at large benefits from the way TJHS currently admits its students.

    As I said, I am not taking sides, but at the same time this is not an innocent debate and the French system provides a view of what the American higher education could become if the incumbent elites have their way. I see the lawsuit against Harvard as a way for these elites to make these institutions less able to admit their students according to the method of holistic admissions, something that would inevitably benefit the incumbents who know how to game the American higher system in the same way the French elite games France’s higher education system.

  137. Nilima Nigam Says:

    Ethan,
    thanks for your thoughtful responses.

    To be clear: yes, absolutely I’m bringing in the issue of the IITs regardless of its immediate connection to the TJHS issue. I’m bringing it in to highlight an interesting example concerning resource allocation/optimization in a highly constrained system, and how this may proceed in nonlinear steps. The objective function (with constraints) is not strictly convex, and maybe we should expect rapid convergence to an optimum. As with any search in a tremendously noisy landscape, it is pretty unlikely you’ll hit the global optimum starting at random. And even if your local search direction seems optimal, it may only be a locally optimal search direction. You may find the path of best intentions to not lead to the desired results. That is all you can infer from the stated example.

    ” We don’t live in such a world and therefore the notion that we need to build a public system for the benefit of “gamers” who see attendance to schools like TJHS as a checkbox in building their MIT admission packets seems a bit preposterous to me. It’s giving even more advantages to people who have already won the birth lottery and doing so payed by society as a whole!”

    I read this as saying your concern regarding the present intake system would be alleviated if TJHS was not a public school, funded by taxpayers. In other words, you’re not objecting to some people wanting this particular credential, nor to the fact that it apparently confers some advantage to them. You’ve objecting to taxpayer dollars supporting this. This seems reasonable. I guess the same logic would apply to *any* public school program which was rigorous, and for which admission was highly competitive in the manner you dislike.

    [It would be an interesting experiment in the market to see how much parents would be willing to pay in tuition to get their kids to take the admission exam, and then enter this program, if the TJ advantage at the entrypoint to college is indeed significant. ]

  138. Ethan Says:

    Nilima Nigam #137

    “I read this as saying your concern regarding the present intake system would be alleviated if TJHS was not a public school, funded by taxpayers. In other words, you’re not objecting to some people wanting this particular credential, nor to the fact that it apparently confers some advantage to them. ”

    Yes! I elaborate more in my comment #136. One of the many people I know who went to an IIT (not a close friend) once made a comment boasting that he never paid a dime for his education (ie, that he went to Indian public schools all they way through). Since India’s uber constrained system that you describe means that people game it, this is de facto Indian tax payers paying for the Indian elite who knows how to game the system.

    The United States’ higher education system strength, at least for now, is its diversity and its lack of “one size fits all” coupled with it being open to admit students at different stages of their life journeys. The more we move towards a model like the Indian or French models, and the more it is paid by the taxpayer, the less fair it will be because humans are humans everywhere: incumbent elites will game the system for their children. That’s the core of the TJHS issue, namely, people move to the district to have Fairfax County taxpayers provide an elite “free” education for their children, to what the County seems to be saying “what a minute, this doesn’t look fair, is there any way we can make it fairer for underrepresented groups?”.

  139. Christopher Chang Says:

    This discussion has become thoroughly detached from reality.

    Top-tier private schools will continue to exist regardless of what happens to TJ. “Elites passing privileges from one generation to the next” is a real problem, but at least in the US, it’s more accurate to say that schools like TJ and Stuyvesant today, and CCNY in the 1930s-60s, are more on the “solution” side of that than the “problem” side. All of these elite public schools have been flooded with lower-social-status minority groups (primarily Asians in the case of TJ/Stuyvesant, and Jews in the case of old CCNY) which weren’t able to send the lion’s share of their best kids to private schools.

    The main question here is whether you consider it a social good for very smart and hardworking middle-class minority kids to have access to top-quality education, or if you think it should be fully privatized and thus reserved for the upper class.

    (A secondary question is what and how much should be done about the test-prep arms race.)

  140. Ethan Says:

    Christopher Chang #139

    Obviously I strongly believe that education is a huge asset for people -particularly people who come from the lower end of the chain food since they are the ones statistics say benefit the most from education as a tool to increase social mobility- and that maximizing access to education to anyone -irrespective of someone’s socioeconomic background- is a great thing for America.

    However, I dispute this notion that keeping the current system at TJHS is about increasing access. The very way they recruited before the current reform was discussed is indicative of the opposite: that’s precisely the reason they are considering a change. The issue cannot be decoupled from the test prep arms race because it seems there was a test that was used primarily to restrict access to underrepresented groups such as Black Americans. Given that Fairfax County median household income in 2018 was $118K https://www.fairfaxcounty.gov/news2/who-we-are-in-fairfax-county-in-2018-annual-demographics-report/ , the notion that the system of selecting people for the school via an entrance test is about “increasing access” is not believable. I see it more plausible that the system that some want reformed is about entitled upper middle class Americans and immigrants who believe that that they have a right to have their children offered a top high school education at the taxpayer expense. When the College Board announced that they were going to add a “social adversity score” to the SAT -a plan now abandoned- I heard someone lament that it’s a pity the plan didn’t go through because it was about the only thing that could motivate wealthy Americans to move to poorer neighborhoods.

    As I said, this is a very complex issue and none is denying (at least I am not denying it) that the students offered a spot at TJHS benefit from their educations there. The question is one of fairness given that there are public dollars involved. Who would benefit most from attending one such school, a promising Black American raised by a single mother who through his own efforts and little help gets a 3.5 GPA in scientific subjects or the child of two math professors with a perfect GPA? I am sure that the latter would probably do better in the entrance exam but there is no question in my mind that it is likely that getting a spot at TJHS is more beneficial for the former given that the child of the two math professors will do great in high school no matter what by the mere fact of the home he is being raised.

  141. Recent TJ Alum Says:

    Ethan #126, Nilima Nigam #137

    I just wanted to add a little to the discussion of elite college admissions, and whether going to TJ helps. I think the answer is ‘maybe slightly’. If I remember correctly, in my graduating class, approximately 10 people were admitted to MIT. Around 100 people apply to MIT from TJ per year. The admit rate from TJ is then higher than MIT’s overall acceptance rate of 7%, but if one adjusts for the fact that many of TJ’s applicants are very strong academically, the admit rate is not as high as one might expect.

    The reason for this is that MIT consciously tries to admit a diverse class of people. There’s a great post on this by the assistant director of MIT admissions here: https://mitadmissions.org/blogs/entry/diversity-or-merit/ . In practice, this implies that in order to be admitted to MIT from TJ, one has to typically demonstrate much more academic success than applicants from most public high schools, because the MIT admissions committee expects more given that one had access to more opportunities.

    This still doesn’t address the question of whether going to TJ helps one get into elite colleges. It is hard to say for sure, because we don’t have access to the counterfactual world in which TJ students instead went to non-magnet public high schools (many of which are quite good in northern Virginia). In my case, I would have never been able to accomplish all that I had in high school if I hadn’t gone to TJ. However, had I gone to a non-magnet high school, I would have likely gotten better grades comparative to my classmates, and been more interested in science and math than a greater percentage of my class. I then think that it’s likely that I would have been admitted to an ‘elite’ college even had I not gone to TJ, but possibly a different one than the one I went to. On the other hand, I know people who regretted their decisions to go to TJ, as they felt that they could not stand out academically for college admissions, while they would have stood out elsewhere.

    Regardless of whether the net effect on college admission rates is positive or negative, I think the increased competitiveness makes it unwise to go to TJ for anything other than a challenging high school experience. I think something like 40% of my class went to public universities in Virginia (which are excellent, and far less costly), some by choice, and some not.

    Whether or not elite colleges are worth going to is a completely different (and fair) question…

  142. Ethan Says:

    Recent TJ Alum,

    Thank you for great insights on your two comments. I think that this is very significant,

    “If I remember correctly, in my graduating class, approximately 10 people were admitted to MIT. Around 100 people apply to MIT from TJ per year. The admit rate from TJ is then higher than MIT’s overall acceptance rate of 7%, but if one adjusts for the fact that many of TJ’s applicants are very strong academically, the admit rate is not as high as one might expect.”

    I bet that most high schools in the US don’t send anyone from their graduating class to MIT or any of America’s top schools (group where I include roughly the top 50 schools as ranked by US News in their national universities section). So having 10 people from a single graduating class going to MIT is like WOW! I am assuming that other graduating seniors got into other good schools, including the 40% you mentioned who went to Virginia public schools. I think that right there you have why some people go to extreme lengths to game the system in favor of getting their children attend TJ and why the issue of how to select who can benefit from such an experience is not a minor one. This is compatible with your other observation that some people regret going there. It’s clear that TJ seems a very competitive environment and it’s therefore not suited for everyone.

    As an aside, I have met people who have gone to most of America’s top schools and MIT people for one reason or another are among my favorite people (for the record, I didn’t go to MIT myself other than as a visitor 🙂 ).

    Then,

    “Whether or not elite colleges are worth going to is a completely different (and fair) question…”

    Assuming you are the kind of person motivated by intellectual challenges and with a healthy intellectual curiosity, I think that the answer is a resounding yes. Less clear to me is whether going as an undergraduate is the best option, particularly if you are a student who comes from humble origins unfamiliar with the world of privilege, or perhaps it is better for you to attend as a graduate student (either to complete a terminal masters degree or a PhD degree) once you have developed a certain level of maturity and have a good undergraduate experience behind you. The way I think about elite colleges is a dual track system:

    – undergraduate is where the wealthy and well connected send their kids to get educated and socialize with the children of other wealthy people. Of course with the qualifier that these students are able handle the coursework and the intensity. These are the people who for the most part, end up bringing in the dollars as alumni and after whom the schools name buildings, endowed professorships, etc.

    – graduate school is where people who didn’t get a chance to go as undergraduates have their second chance in life to learn from the very best (people like Scott, LOL!). In my experience, there is more socioeconomic diversity at the graduate level because many of the people who get in as undergraduates tend to leave after they get their bachelors degrees, leaving a lot of room for people who went to other schools to apply to their graduate schools. Of course I am talking about academic departments. Professional schools like business schools, medicine or law schools are probably as hard to get in as it is to get in as undergraduate.

  143. Christopher Chang Says:

    Ethan #140:

    My comment was agnostic re: “keeping the current system” (I explicitly acknowledged that the test-prep arms race may be serious enough to require an adjustment to the system). Before a useful discussion can be had there, we have to agree that we want TJ-level public schools at all, as opposed to keeping taxpayer dollars out of the picture and leaving this to private schools.

    Again, the parents sending their kids to TJ or Stuyvesant today, or CCNY in the past, are *not* as privileged as the significant cohort of parents sending their kids to private school. I understand that it’s distressing to see even-less-privileged taxpayers pay into this without getting much benefit, and it is reasonable to use the democratic process to address this inequity. But let’s not lose sight of the big picture here: in the US, these public schools counter hereditary privilege far more than they perpetuate it, and they have to have rather high standards to continue serving this function. Some of your earlier comments got this backwards.

  144. Jason Li Says:

    On the topic of what a good entrance exam for a school like TJHS might look like, I’d like to bring up Canada’s system for gifted education, at least in Ontario where I grew up, and my personal experiences.

    The gifted programs are state-wide, and any public school large enough to support at least one gifted class would have one. Since the program was a branch of public school, it was free. To enter, you have to take something similar to an IQ test and score in the top 2%. (I remember there were many sessions in the test and the entire thing lasted over an hour, so it was definitely very comprehensive and less prone to statistical error.) Once you’re in, you automatically stay in the program until 12th grade.

    I was in the gifted program from grades 5-7 before I moved to the US. Everyone in the class was extremely smart/talented—for example, I went on to take the USAMO in high school in the US, and yet I wasn’t even among the best math students in my gifted class. However, not everyone was motivated, which I suppose is the result of testing for IQ alone. The class was about a third to a half Asian, definitely nowhere close to 75%, but Asians were still overrepresented. Not everyone came from wealthy families either, since my town wasn’t particularly affluent, even though it was richer than average. I also never heard of specific test prep classes designed for you to score well on the entrance exam. I guess the fact that it’s a vanilla IQ test does make it harder to prepare for/pay for prep, compared to SAT-like tests, so it does help with social mobility. Then again, this was like 2005, so the test prep industry wasn’t as prevalent back then, and I have no idea what it’s like now. Anyway, my point is there’s positives and negatives to making a prestigious school’s exam rely on innate intelligence alone.

    I didn’t do very well in my classes, at least relative to my peers, and looking back, I didn’t learn a whole lot. I definitely connected a lot more with my peers though, and nerd culture was respected there, not looked down upon, for which I’m eternally thankful. So ultimately it was the other students who defined the class, not the teachers/curriculum.

    Just wanted to share my experience in a program with a different style of entrance exam that looks for “natural” intelligence over test prep. I do believe an ideal entrance exam is more a hybrid of the two: test for things that are hard to study for to combat the test prep industry (not necessarily IQ), but still test for hard work/conscientiousness.

    Forgot to add—this was also a general curriculum, so there was no emphasis on science/tech for example. All classes were advanced level. Maybe that was why I didn’t do so well there…

  145. fred Says:

    Like many other terms, the meaning of the word “elite” has been coopted by radical ideologies.

    Elite: a select group that is superior in terms of ability or qualities to the rest of a group or society.

    Your country only sends its *elite* athletes to the Olympics, and, if they win, you’ll be very happy.
    If your kid has cancer and she’s under the care of an *elite* team of specialists, you’ll be more hopeful.
    If you’re visiting a foreign country and get kidnapped for ransom, and your country sends one of their *elite* military squads to rescue you, you’ll be very happy.
    If you disagree with any of the above, you’re either deluding yourself or some variation of a Taliban.

    We’re now told that the word “elite” refers to a bunch of privileged crooks, thieves, and impostors, and in the process the idea of nurturing the very best in any given field gets wrapped up in it (which isn’t by accident, it’s the intended goal).

    Achieving equality of outcome means it’s okay to handicap the gifted.
    The goal is leveling society for the lowest common denominator, and people with exceptional brain, beauty, and/or athletic abilities are the problem.

    Kneel comrade!

  146. anon Says:

    Ethan #117, You always had Penrose on your side. 🙂 It’s kind of a joke that it took so much time, but that won’t make him right on everything. If you doubt the latter, try to make sense of his claims about CCC (to be clear: I don’t refer to whether his theory will be proven right some day, but to Penrose claiming that he *already found* statistically meaningful signatures).

  147. Ethan Says:

    Christopher Chang #143

    “But let’s not lose sight of the big picture here: in the US, these public schools counter hereditary privilege far more than they perpetuate it, and they have to have rather high standards to continue serving this function. Some of your earlier comments got this backwards.”

    It depends whose privilege we are talking about. If you are talking about old money/WASP privilege, yes, you are absolutely right. If we are talking about the new aristocracy born out of the so called “meritocracy” around the 1960s when Ivy League schools started to expand their admission criteria to focus on merit, then public schools like TJHS are in 2020 a way this new aristocracy passes their privileges to the next generation, which are not built on land ownership but on being part of the professional class that makes a living in the so called “knowledge economy”.

    For people who belong to this new aristocracy, it’s perfectly normal to have their parents move to certain ZIP codes so that they can attend good public schools, mingle with the children of other like minded professionals and attend high schools that have a high proportion of graduating seniors attend America’s most prestigious schools but this is not the experience of the average American high schooler. Thus, speaking of these high schools as bastions of privilege is perfectly adequate. So is thinking about ways of making them more inclusive, specially given that public dollars are at stake.

    France is farther ahead in this process of convincing the government that using public money to keep this “knowledge aristocracy” system going and the results for society at large are there for everybody to see. Us becoming more like France wouldn’t be good for the country in the long term.

  148. Ethan Says:

    Anon #146,

    Of course, I always thought of Penrose’s arguments as more compelling than those by the people who push the view “brains are just computers”. For a long time, I found the latter arguments intellectually lacking as popular as they are with people like Elon Musk. If Penrose’s stature continues to increase in the decades to come among physicists, because of his Nobel Prize, there is plenty of material in youtube that future generations can lean on to learn how a great minds like Penrose’s think.

  149. Ethan Says:

    Christopher Chang #143

    This book in particular https://www.amazon.com/gp/product/1524731633/ , makes the case that the rise of the new aristocracy, because of its ‘extractive ethos”, is responsible for the decline of the United States. I don’t agree with all the points the author makes. There are clearly things better today in American society than in the 1960s for large classes of people such as ethnic and religious minorities, the disabled, and the list goes but the debate is not as innocent as you portray it. Human beings are human beings. The instinct of passing one’s privilege to the next generation (even if one is a first generation college student like me) is there and not exclusive to the old WASP American elite. To deny it doesn’t help the conversation.

  150. Filip Dimitrovski Says:

    Ethan #148:
    There is this thing that we use for testing COVID… It’s called PCR, you’ve heard of it, polymerase chain reaction. It was discovered a guy named Kary Mullis. Eventually, Mullis won the Nobel Prize in Chemistry in 1993. Also, Mullis was an HIV and AIDS denier, he thought he encountered aliens and believed astrology should be part of education.

    I’m glad Penrose got the Nobel prize, but his brain/mind theories based on Gödel
    incompleteness will end up sounding equally funny for future generations.

  151. Ethan Says:

    Filip Dimitrovski #150

    I think that comparing Kary Mullis’s views on AIDS to Penrose’s views on the non algorithmic nature of the human mind is a bad analogy.

    Penrose is at his core a mathematician -of the kind our society doesn’t produce any more I must say due to academia’s extreme specialization compared to when Penrose was a PhD student- and his argument is based on analyzing the mathematical consequences of Godel’s Incompleteness theorems.

    I think your analogy is more apt for the people who believe that “brains are like computers” because computer scientists, for the most part, know next to nothing about the human brain, let alone the human mind, yet they like to borrow terms from biology to boost their claims. The name of the field itself “artificial intelligence” is a marketing term https://amturing.acm.org/award_winners/mccarthy_1118322.cfm ” Given that McCarthy was primarily a mathematician and technologist who had little use for puffery, it is ironic that his most widely recognized contribution turned out to be in the field of marketing, specifically in choosing a brand name for the field. Having noticed that the title of the Automata Studies book didn’t stir up much excitement, when he subsequently moved to Dartmouth College he introduced the name artificial intelligence at a 1956 conference there and saw that it was embraced both by people working in the field and the general public.”.

    Ditto of using the name “artificial neural networks” for a particular type of machine learning architecture which makes many believe that replicating a human brain is a matter of building a computer with the same number of artificial neural networks as the average human brain’s.

    At the end of the day, this is a controversial debate. I am so confident of my position that as I said, I have an ongoing bet with someone, started 5 years ago, that AGI won’t happen in the next 25 years (the original bet was for a 30 year mark) . I am happy to have another one with you or Scott!

  152. Christopher Chang Says:

    Ethan #147:

    Not a fan of the US’s education system becoming like France’s, either. But that’s not a realistic danger.

    Again, these schools are dominated by the *least* entrenched subset of the professional class: recent Asian immigrants. Contrary to your “new aristocracy born out of the so called ‘meritocracy’ around the 1960s” claim, many of these families were not even in the US at all until several decades after that. It’s the proposed change that would bring us closer to the perpetuation-of-privilege world you describe, where most of the benefit would come from living in the right ZIP code. Whereas right now, being in the right ZIP code is quite obviously NOT enough; many of these kids have to work very hard to actually get in, to the point where the arms-race component of the admissions process is eating enough into the overall social gain that allowing a little bit more perpetuation of privilege may be an acceptable price to pay for tamping down the arms race.

  153. Ethan Says:

    Filip Dimitrovski #150

    As I said, I once heard someone I respect greatly describe computer science as a psychotic field. That some in the field continue to confuse their own marketing terms with reality -such as in the case of artificial intelligence or artificial neural networks- lends credence to this claim. It’s something very specific to computer science that you don’t see in other engineering fields that are more grounded in reality.

  154. Ethan Says:

    Filip Dimitrovski #150

    And to continue with the “psychotic” nature of computer science, particularly in the context of the main point of conversation of this thread. Around one year ago, Princeton professor Arvind Narayanan, made the news with this much needed reality check for the field of artificial intelligence,

    https://www.cs.princeton.edu/~arvindn/talks/MIT-STS-AI-snakeoil.pdf

    Confusing imagined realities with actual realities has a name, it’s called the fallacy of reification https://en.wikipedia.org/wiki/Reification_(fallacy) .

    Reification is responsible for a lot of pain that happens in society, whether it’s the concept of IQ, the definition of mental disorders, people confusing the luck of being born at the right place at the right time with their own personal virtue (triggering debates like this) or, in my view, the notion that brains are computers. Reification has now even hit the world of physics, with the defenders of string theory claiming that empirical verification is no longer needed in physics and that the field should abandon Karl Popper’s requirement that it be falsifiable.

    So the disagreement we are having is not a minor one but about different worldviews about what “reality” is.

  155. Ethan Says:

    Christopher Chang #152

    I hear you and you have a point. The argument I have been hammering since my first comment in this post is that the subject matter is not as simple as “one exam that gives everyone an equal chance to prove his/her intellectual acumen”. I see the attempt by TJHS to make it more inclusive a noble goal which nobody should confuse with “lowering the bar” because “the bar” is a very subjective term. Ie, someone has to define what “the bar” is. Using supervised machine learning terminology, someone defines the “ground truth”. Why should the plea of recent immigrants of Asian origin who are brought up in families who have a strong work ethic -by the very fact that they immigrated to the US- take preference over giving opportunities to Black Americans tho to this day continue to suffer the social consequences of America’s original sin?

    This issue is very complex. And the new aristocracy that thrives in the “knowledge economy” is also very real and they have their own agendas as to why the public should fund the education of their children.

    I’ll leave it here because at this point positions are very clear. Note that I haven’t taken sides. The point I have been hammering is that the issue at hand is very complex and it cannot be easily dismissed as Fairfax County trying to kill merit admissions to TJHS.

  156. anon Says:

    Ethan #151, This sounds like a complicated opinion. Let me defend the merits of a much simpler one: If you’re finite, then you’re computable. You’re not infinite.

  157. Ethan Says:

    anon #156

    Great way of putting it. I do believe that we, human beings, are spiritual beings that transcend finite settings and that’s probably what Penrose is trying to say -although he is an atheist- with his argument that understanding -what human minds do- is not algorithmic.

  158. Nilima Nigam Says:

    Ethan, there’s a (minor?) correction I’d like to make to your assertion

    “One of the many people I know who went to an IIT (not a close friend) once made a comment boasting that he never paid a dime for his education (ie, that he went to Indian public schools all they way through). Since India’s uber constrained system that you describe means that people game it, this is de facto Indian tax payers paying for the Indian elite who knows how to game the system.”

    To be very clear: an overwhelming majority of students *entering* the IIT system now have availed of expensive coaching for multiple years, and are *not* subsidized by the tax payer. IIT itself is (part) government funded. The bulk of these students are also from private schools now (confusingly, private schools in India are called ‘public schools’, as in ‘Delhi Public School’.)

    Beyond that: yes, it’s a pretty complex situation. No, minds will likely not be substantively changed in one direction or the other.

    Humans, especially human parents, tend to strongly prioritize local optimization (ie, how can I ensure that my kid has as good/better a life compared to me), over global optimization (ie, there are 7 billion humans, all equal, and how can we ensure a fairer shot at a decent meal to everyone). This tendency is in itself neither good nor bad. This is just reality. Addressing historical wrongs is important, but I sincerely hope a little more humility and acknowledgement about the inadequacy of the enterprise were in evidence. For example, I personally do not dare assess the relative merits of the wrongs done by the US unto a young Black American born in 2004, compared to that done unto a young Iraqi kid born the same year (whose home/schools the Americans bombed out of existence). There is perhaps some moral calculus which makes one kid more important than the other. Or it’s that dumb birth lottery we refer to, and ‘fairness’/’justice’ in this context is just as local a consideration as others.

  159. Recent TJ Alum Says:

    Ethan #142,

    Thank you for the response!

    I agree that the number of admits from TJ to top colleges is one factor that motivates many parents to send their children to TJ exam prep programs. And I also agree that the question of selecting students for TJ is not an easy one. After reading many of the arguments in the comments, I’m tentatively supportive of the idea of using a lottery to select some proportion, although not all, of the incoming class.

    (As a sidenote, it seems that the Fairfax county school board will most likely remove the admissions test, but is still undecided on a replacement process and whether it will involve a lottery: https://www.washingtonpost.com/local/education/thomas-jefferson-high-admissions-change/2020/10/07/0a1f8faa-08a7-11eb-9be6-cf25fb429f1a_story.html)

    I’m still a little unsure about whether the college admissions statistics from TJ are as impressive as they first look, though. I found some (self-reported, so take this with a grain of salt) data on college applications and admissions from the high school I would have gone to if I hadn’t gone to TJ. In a recent year, there were 9 applicants to MIT, and 1 was accepted. If these numbers are to be believed, this isn’t all that different from the admissions odds at TJ– the main difference is that there’s a larger pool of students interested in STEM and eager to go to famous colleges at TJ.

    My opinion then is that the main advantage of going to TJ is similar to the main advantages you outlined regarding going to elite colleges for undergrad/grad school: the opportunity to take challenging coursework and learn from great teachers, friendship (or, more cynically, networking opportunities) with like-minded peers, and brand name recognition later in life. These can definitely lead to a lot of personal growth. Whether going to TJ leads to measurable improvements in career/earning prospects, or in one’s capacity to contribute to society down the line, is more debatable. Unlike colleges like Harvard, TJ is relatively young (and so doesn’t have as extensive of an alumni network), not as well known, and in the end is only a high school. Certainly, TJ alumni often go on to have great careers, but it is hard to say whether it is because of TJ that they did, or whether they would have succeeded regardless.

    In any case, given that TJ is funded by taxpayers, and given that going to TJ confers opportunities that can lead to (at the least) great personal growth and satisfaction, I agree that it is very important to select a diverse class of admits.

  160. jonathan Says:

    “The school also happens to be only 19% White (it’s now ~75% Asian)”

    Honestly, I find these numbers shocking. This is a ratio of ~4:1 asians to whites, compared to about 10:1 non-hispanic whites to asians among 15-19 year olds (per the latest census estimates). In other words, asians are ~40x overrepresented on a per-capita basis.

    I find it extremely unlikely that asians have 40x as many smart nerds per capita as whites. So what could explain this ratio?

    While I hesitate to suggest it, the explanation that seems most likely to me is that the current admissions criteria are not picking the sort of kid you’re talking about — the bright and curious kid who messes around with computers and math problems and science experiments in his spare time. They’re picking the kid who spends all his time on test prep.

  161. Ethan Says:

    Recent TJ Alum #159

    Thank you for your thoughtful response. I agree with a lot of what you say.

    I prefer not to say much more about me beyond what I have already said for privacy reasons but if I know how great the American higher education system is with respect to other countries’ is because I have some first hand knowledge of the subject matter. It’s a national treasure that one could start his or her life going to a bad high school, drop out of the system, come back and through the different options available end up becoming a world class scholar. I cited Craig Venter’s case but there are many other cases of people who have benefited from the system, including for the most part many private citizens like me.

    We should think about ways of making it more inclusive, not less. The article you mentioned says,

    “The changes would take effect immediately, meaning this year’s crop of eighth-graders — many of whom have spent months, if not years, preparing for the test — will not sit down to take the two-part exam on math, reading and science.”

    Something that explains why ~ 75% of the students are Asian. “Test taking” is a skill that, in my experience, educated immigrants from either India or China know well because that was probably their ticket to the US given these countries’ education systems emphasis on standardized tests for college admissions.

    Seeking ways to make TJHS more inclusive to students who come from different backgrounds is a net plus in the long run even for TJHS as it seeks to establish its brand because becoming a successful professional/scholar takes a lot more than just being good at test taking. The percentage of Asian Americans in society at large is 5.6 %, so a class experience that exposes students to a world where 75% are of Asian heritage doesn’t seem to me the best class experience to produce future leaders no matter how good these students are taking tests.

  162. Ethan Says:

    Nilima Nigam #159

    The guy I have in mind is in his 50s and he was referring to neither he personally or his parents having paid any fees to attend either K-12 or IIT. Obviously I don’t know a lot about the Indian system and you clearly know more than me. He didn’t mention whether his family paid for any coaching support.

    With respect to Black Americans, American society has a distinctive moral obligation with them that it doesn’t have with other ethnic minorities, including Native Americans, who have not had to suffer in their ancestry the effects of slavery, Jim Crow laws or more recently attacks by people like William Shockley or James Watson who seek to blame them for the abuses Black Americans have endured for centuries. Identifying “being black” with “being intrincsically less smart” is yet another instance of the reification fallacy among the intellectual class of the late XIX-the century, early XX-th century that we know has no basis in science due to recent developments decoding the human genome (Craig Venter explains here https://www.youtube.com/watch?v=6vS7AO9XYj4 ) .

    I am not a fan of reparations because the past cannot be changed but admissions to high school or college is about the future, particularly about educating tomorrow’s leaders. Acknowledging America’s systemic racism against Black Americans implies understanding that the definition of “merit” cannot be based exclusively on doing well in some exam. Equating “merit” with “doing well in some exam” is a talking point that has become a dogma with some, but if you think carefully about it, there is nothing intrinsically unbiased in equating these two terms. All exams are intrinsically biased towards people who do well in them, irrespective of the reason why do well in them (cultural upbringing, natural talent, etc). People who grow up in white upper middle class homes and children of successful Asian immigrants have an implicit advantage when it comes to doing well in standardized tests over Black Americans who were born out of wedlock (70% of Black Americans fit this description). TJHS re-thinking its admission procedures in light of the demographic data of recent graduates is a net positive in my opinion both for the school and society.

  163. Ashley Lopez Says:

    Scott,

    Is it really the case that being a nerd is a sufficient condition to be bullied? I have not seen that pattern (at least here in India). In fact I remember some of the worst in the class being nice to the nerdy ones. Actually I have not seen a correlation between being good at studies and lack of social skills either.

    Of course kids who lack social skills tend to get bullied. But I would – ahem – say that to some extend that is a good thing. Math may be the awesomest thing, but life is not about solving math problems. My ex-manager used to say, while talking about upbringing of kids, that kids need to learn to operate in environments with no externally imposed rules and structures. I would say that is true (wouldn’t you too?).

  164. Ashley Lopez Says:

    Jonathan #160,

    “They’re picking the kid who spends all his time on test prep” – Somehow I would tend to agree with you. Why not add an interview to this entrance exam, to assess originality and creativity? That should weed out these ‘entrance coached’ types, and would be the simplest and the best possible solution, don’t you think?

  165. Scott Says:

    Ashley Lopez #163: No, I would not say that it’s a good thing that kids who lack social skills tend to get bullied, any more than I’d say it’s a good thing that people who lack survivial skills tend to get murdered or raped.

    Of course it’s important to learn social skills, for many purposes more important than to learn intellectual ones! But “social skills” are a wildly relative concept. A person whose social skills are perfectly adequate, or even exemplary, for navigating social situations of their choosing, might suffer if they’re thrust involuntarily into an environment dominated by bullies, or even just by people whose worldviews irresolvably clash with their own. Most of the happiness I’ve achieved in my life, is only because I’ve arranged everything to try to minimize the power that such people have over me.

  166. jonathan Says:

    Thinking about this a bit more, I think there are three issues that people too readily conflate:

    (1) Desire for demographic ratios in proportion to the general population, as a desired end in itself.

    (2) Fear that the current admissions criteria may be selecting on the wrong qualities (e.g. parental resources and time devoted to test prep, rather than talent, potential, curiosity, etc).

    (3) Fear that the admissions criteria, even if successfully selecting on the qualities we want, is causing parents to sacrifice their childrens’ childhood in the name of entry to elite institutions, which may not really have much social value added anyway.

    Hopefully we can find a way to address these concerns without throwing the whole concept of academic merit out the window, or abandoning the highly useful (if imperfect) tool of standardized testing.

  167. Ethan Says:

    Ashley Lopez #163

    As someone who was bullied in high school, I agree with Scott. I can’t see how being bullied is a good thing given the scars it leaves in you.

    In my comment #162 I mentioned the special obligation American society has towards Black Americans. Here you have a group of people who have been “institutionally bullied” by American society for centuries in every shape and form -including morally bullied with the racist theories of IQ endorsed by Lewis Terman, William Shockley and more recently James Watson- and the results are there for everybody to see how bad it has been for them as a group.

    You say you are writing from India. Indian society is plagued by the problems of the caste system so I can’t hardly see how being bullied is good for Indian society either. Why is that the United States is benefiting from so many of them trying to get out of India to make a better living for them and their families here in the United States? Don’t you think that a more egalitarian society that aspires to treat everybody equally would be better for India given that most of those people would seek to stay there contributing to Indian society?

  168. Recent TJ Alum Says:

    Jonathan #160, Ethan #161,

    One final thing I would like to add before signing off is that the statistics can be interpreted in many ways. Fairfax county’s population is 19% Asian and 61% White. Furthermore, admissions statistics for the most recent admitted class (http://www.fcag.org/TJ%20Admissions%20class%20of%202024.pdf) indicate that 56% of the applicants to TJ are Asian. The remaining difference between Asian applicant percentages and admit percentages (56% to 73%) is not nearly as stark.

    Certainly, there are many things that are undesirable about the current process. One would be the low admit rates of Black and Hispanic students comparative to applicant numbers. Another is the proliferation of test prep. However, if Asian students tend to apply to the school in greater numbers (and consequently get admitted at higher percentages), it is in the end due to their individual rights to do so. Perhaps part of this could be explained by insufficient countywide awareness of the existence of TJ, and if so, I would support efforts to raise awareness of the opportunities it offers. However, if higher Asian applicant numbers persist, I don’t believe that they would be an intrinsic problem.

    I think that intents matter: seeking to purposely increase the numbers of underrepresented minorities at TJ is a great goal, but intentionally decreasing the number of Asians would be a terrible one in my opinion. If the number of Asian admits decreases as a natural result of making the process more equitable and less “gameable”, I think that is fine. But I would caution against blaming Asians for the problems of the school as it is today (not saying that either of you are; just wished to add this as a general comment).

  169. fred Says:

    There’s one very important factor:

    When I grew up as a teenage “nerd” (in Europe during the 80s and early 90s) and I wanted to learn more about something like computer 3D graphics or electronics, I had to commute for one hour to the only English bookstore in the city, and hope there was something relevant in their rather small technical/computer section… and if there was something, usually it was very mediocre. Hobby magazines were only coming up once a month, and you had no control over their content (even identifying and then ordering interesting older issues was a challenge).
    So learning anything independently was ridiculously slow. With so much friction in the way of progress, it was easy to just give up (even if you started very motivated).
    College was really the only place where the bandwidth of information was much much higher.

    But, today, any kid who is curious about something technical/scientific has access (within minutes) to a nearly unlimited amount of the very best resources (papers, videos, code samples, discussion forums, access to tools/materials through online stores,…), regardless of location.

    So this makes traditional education somewhat irrelevant, at least when it comes to kids who are really driven and self-learners.
    So maybe there’s a point in saying that those kids really don’t need as much attention, that maybe it’s more important to target the median of the population, and use better teachers to try and inspire them more.

  170. Elizabeth Says:

    Bullying, peer pressure, and all vulgar forms of peer socialization cause regression to the mean.   Taking evolution seriously means that being human is something to be eventually overcome.   I wouldn’t accept any society that forces people to suffer through “peers” that are far below them in intelligence. The animal kingdom is full of superficial diversity, but compared with the unique heights of human intelligence that diversity is all breadth and no depth.

  171. Eric Cordian Says:

    Hi Scott,

    Excellent post. I’ve thought about such issues a lot since I was in the K-12 school system.

    A huge irony to me is that every decent public school I attended during my childhood is no longer in business, and every inner city Hellhole where staff who couldn’t be fired bullied the kids and treated them like crap still is.

    I’ve often pondered why the “system” selects for the worst schools, and makes sure they don’t go out of business, while the good schools wind up as vacant lots, community centers, or condominium housing.

    Which brings me to “exam schools.”

    One of the worst schools I ever attended was an exam school for “gifted” students. I took the exam, got an extremely high score, and since everyone was telling me how lucky I was to get into the place, failed to do any due diligence on it.

    So I walk into the place on my first day, and discovered the staff was mostly a collection of crackpots with power and control issues and delusions of Godhood, who fancied themselves as saviors of poor inner city kids. They handed out demerits at the drop of a hat, and treated us all like doormats who should be grateful when they wiped their boots on us.

    After a Latin class in which we were screamed at for an hour in rote drill by an instructor as he marched down the rows of classroom seats, and an English teacher who spent most of the class trying to sell the more affluent students $750 weekends at a ski lodge he owned, when he wasn’t bullying an overweight kid in the class to the point of tears, I began to doubt that high school was a life experience I was really interested in participating in.

    I left a few years later after being assaulted by the Vice Principal who left black bruises all over my neck and shoulders because I was trying very hard not to laugh during a government anti-drug film featuring the sad story of “Tusko the Elephant.”

    Now, this school was very highly regarded in the community, because the students did above average academically, and many of them managed to get into good universities.

    But this was only because the school was getting to cherry pick the top fraction of students with its entrance exam. Most of these kids could easily have done college level work without attending high school at all. For all I knew, the school was making them stupider. Because the students were not a random sample of kids in the community, there was no way to compare the results of this school’s teaching methods with those of its competitors.

    So the staff continued to con the community into thinking they were exceptional teachers. People would suggest admitting some students to the school by lottery, instead of by the entrance exam, so there would be a cohort of typical students whose academic achievement could be compared with that of the other high schools in the city, but the school was very well connected politically, and managed to torpedo any proposed changes to its admission policy.

    So that’s the fundamental problem I have with exam schools. You can’t tell if the students are clever because the teachers are skilled at teaching, or because the school gets to skim off the smartest students.

    I will also observe that I have never received good service from any organization whose attitude towards me was that they were doing me some sort of favor I didn’t deserve.

    So my solution to exam schools is to let them admit, say, 75% of their students through their entrance exam, and 25% through lottery. Compare how the 25% do against regular schools, and if achievement isn’t exceptional, come to terms with the fact that the teachers aren’t anything special, and no one there is getting a better education than they would in a regular school.

    We need to make sure exceptional students have a place to go where they can get a better than average education, but at the same time, instrument the process well enough that we can weed out schools that are just selling the community a bill of goods and taking credit for their students’ pre-existing accomplishments.

  172. Ethan Says:

    Recent TJ Alum #168 & Jonathan #160/#166

    Those are all great points. I think that this post by Scott has been a great brainstorming session & perhaps Fairfax County could learn a thing or two about engaging the different stakeholders in the conversation. I am also signing off.

    fred #169

    “But, today, any kid who is curious about something technical/scientific has access (within minutes) to a nearly unlimited amount of the very best resources (papers, videos, code samples, discussion forums, access to tools/materials through online stores,…), regardless of location. So this makes traditional education somewhat irrelevant, at least when it comes to kids who are really driven and self-learners.”

    I couldn’t agree more with this. Bryan Caplan has written that higher education today is primarily about virtue signalling in his book https://en.wikipedia.org/wiki/The_Case_Against_Education that Scott has written about. I don’t fully agree with Bryan Caplan’s views on this topic but it is not possible not to see an element of “virtue signalling” in America’s obsession with credentials. I think that a better way to think about this is Peter Thiel’s insight that the college system today, certainly in America, is like the Roman Catholic Church prior to the Protestant Reformation. Peter Thiel explains himself here https://www.youtube.com/watch?v=-HM2Kp5Xj6s . Again, I do not agree with every detailed analogy Peter Thiel makes -particularly this notion that the professorate class is like the do nothing Roman Catholic priests of the 16th century as any tenured professor would tell you; I think that academics themselves are suffering from this effect given how hard it is to get tenure these days compared with a few decades ago- but the general analogy is actually pretty good: here is your piece of paper that will protect you from society’s future evils.

  173. Ethan Says:

    Eric Cordian #171

    “We need to make sure exceptional students have a place to go where they can get a better than average education, but at the same time, instrument the process well enough that we can weed out schools that are just selling the community a bill of goods and taking credit for their students’ pre-existing accomplishmen”

    This is a very good point. A few years ago, there was a debate about whether the military academies such as West Point should be abolished, check https://www.washingtonpost.com/opinions/why-we-dont-need-west-point/2015/01/23/fa1e1488-a1ef-11e4-9f89-561284a573f8_story.html , since the alternative system of educating the military elite, ROTC, seems to be doing as well if not a better job in educating these leaders,

    “No evidence shows that officers who attended civilian colleges, or any one of the U.S. Senior Military Colleges such as the Citadel, are lesser leaders than their service-academy colleagues. Tom Ricks, a Pulitzer Prize-winning defense journalist, put it succinctly: “After covering the U.S. military for nearly two decades, I’ve concluded that graduates of the service academies don’t stand out compared to other officers.” After all, perhaps the most preeminent Army leader in recent times, Colin Powell, is a product of the ROTC, not West Point.”

    Maybe “exam schools” make sense in theory, but looking at whether they deliver the goods they promise is a healthy exercise of accountability.

  174. Gerard Says:

    Eizabeth #170

    I agree. It seems like some of the absolutists of equality of outcome won’t be satisfied until the universe reaches complete thermal equilibrium, AKA “heat death”.

  175. Arul Says:

    Why should PKC be nominated for Nobel Prize? It is not proven P\neq NP. Even with a proof of P\neq NP foundations of PKC is still not clear.

  176. Filip Dimitrovski Says:

    I did an analysis of comment count per person/name and the mean is 2.1 and median is 1. On the other hand, Ethan has 37, winning the first place with over 30 more than the second. ????

    Ethan #154: All I’m saying is Penrose’s theoretical ideas suck and the scientific consensus is that he’s misusing the Godel’s theorems. These don’t have to do much with our current physical universe or the false AI marketing that we’ll be simulating humans soon.

    * I didn’t include Scott in the comment list.

  177. Patrick Dennis Says:

    I wish that someone would notice that Gorsuch’s reasoning in the LGBTQ case is perfectly suited to choking out any remaining breath of affirmative action based upon race.

  178. Ethan Says:

    Filip Dimitrovski #176

    Two points:

    1- the ad hominem was unwarranted

    2- “consensus” is not part of the scientific method no matter how much the media and AI propagandists push the notion that “consensus” equates “settled scientific fact”. It’s very ironic that you mention “consensus” in this thread about gifted education. Generally speaking, “appealing to consensus” is a form of intellectual bullying. If you think that I am going to take said bullying over Roger Penrose’s impeccable argument, right there you have why AI, as a field, is known for over-promising and under-delivering, with two well known AI winters (one in the 1960s, another in the 1980s; the second is the context where Penrose write his book “The Emperor’s New Mind”). We are at the end of another AI hype cycle for the same reasons -check https://www.technologyreview.com/2020/03/27/950247/ai-debate-gary-marcus-danny-lange/ – although I don’t think we will have a full blown AI Winter this time given that companies like Google, Facebook and Microsoft have used machine learning successfully for a very narrow set of application such as email auto-completion. Self-driving cars? You can forget about it in the near-medium term. “Consensus” is for people who live in worlds decoupled from reality. For people like me, a GPT-6 system that can pass for a human would do to falsify Roger Penrose’s argument. And as I said, precisely because I believe Penrose has the upper hand over that “consensus” you appeal to, I am not losing my sleep over the possibility of HAL9000 or Skynet (pick the fictional horror system of your choice) taking over humanity.

  179. Nilima Nigam Says:

    Ethan #162:
    not being a moral philosopher, it is hard for me to assert categorically whether the descendents of a community targetted by genocide are more or less entitled to consideration than the descendents of a community enslaved. Not being in the USA, I have no insight into whether Native Americans are treated better than Black Americans. And why the kids of immigrants are reserved for particular opproprium is interesting to me, but not terribly so. This is just how your country seems to work: a hint of discounting the worth of some is necessary in order to lift others.

    Approximately the only compellingly clear argument I’ve read is that as a society, your country could opt not to have *taxpayer* money going to schools which have specialized programs. This is a straightforward and compelling argument. Much else is either a tautology – selection criteria are biased to select, systems which require resources and yield incentives will lead to gaming. I’ve yet to read a reason why there isn’t a proposal to just rename all the schools in the district ‘TJHS’ without changing their individual criteria. After all, if the admissions process in this one school is bust, the curriculum isn’t really doing what it should, the kids in there (Asian) really shouldn’t be there, and the whole thing is just a big exercise in getting a stamp from the school so as to get into MIT – just hand out the stamp. Leave this school alone. Problem solved.

  180. Ethan Says:

    Filip Dimitrovski #176

    One more thing, and please do not take this as mi returning you the favor in your ad hominem attack, the link you point to says,

    “About me: I’m a Computer Science student living in Skopje”

    For readers unaware, Skopje is now part of North Macedonia. part of the former Yugoslavia, part of the former block of Eastern Communist block. While we are 30 years removed from the fall of the Berlin Wall, you don’t remove the legacy of Communism, and thus of arguments such as “appeal to consensus” that could send those who disagree with the consensus to the gulag, in that short period of time.

    The United States, where I live and the country I am blessed to be a citizen of, has a healthy tradition of free exchange of ideas that extends beyond the legalism of the 1st amendment and that exists precisely to debate unpopular ones. It’s no accident that this is the place where you can find 15 out of the top 20 universities as measured by their production of original scientific research (according to the Shanghai ranking) whereas the countries of Eastern Europe are not known for hosting universities that produce this kind of research output.

    All this to say that attempting to shutting down debate with arguments of the kind “your idea is not popular” is unlikely to work over here if one day you decide to come to the US to pursue a professional career. The subject matter of this thread is a good example of how things work around here. Apparently, as I write this, the admission exam to TJHS was eliminated in a meeting yesterday irrespective of the “consensus” among those who benefited from the old system and wanted to keep it!

  181. Ethan Says:

    Nilima Nigam #179

    The issue of the systematic mistreatment of Black Americans is a very complex issue that is hard to fully grasp from the outside. This is not to say that American society has not mistreated members of other ethnic minorities. Americans of Irish, Italian, Jewish and -believe it or not- German heritage all have within their communities stories of discrimination of their ancestors who were first generation immigrants. Native Americans were the people who were here before the arrival of the Europeans but the US Constitution never treated them as less than a person as it did with Black slaves https://en.wikipedia.org/wiki/Three-Fifths_Compromise . The three fifths comprise wasn’t repealed for almost 100 years and it took a Civil War to do so.

    The issue of taxpayer dollars allocation is THE ISSUE on the matter of TJHS. If this was a private school, there would be no argument as far as I can tell. For example, America’s most prestigious private universities are not subject to the same rules as the public schools even though the former’s receipt of public dollars in the form of their non profit tax status and federally guaranteed loans doesn’t immunize them completely from government influence. Just this week, the US Department of Justice announced a lawsuit against Yale for their admission practices https://www.nbcnews.com/news/us-news/feds-sue-yale-allege-discrimination-against-applicants-n1242697 . The only reason the federal government can do that is because Yale (and Harvard although in that case the US government didn’t initiate the lawsuit although it supported the position of the plaintiffs) receive public dollars. There are private universities in the US (not the most well known and prestigious ones, notably the conservative Hillsdale College) that refuse to receive any form of public money precisely to do as the please when it comes to admissions and other practices.

  182. Ashley Lopez Says:

    Scott #163, Ethan #167,

    I should have been very careful in my choice of words when attempting to communicate such thoughts. When there is bullying there would be always some who got it real bad. I don’t know how to answer that question.

  183. Ashley Lopez Says:

    (continuing on my previous comment) however, I still do hold there is something wrong if all you learn is “Lisp or the surreal number system” at school. OTOH most stars in science and math (Scott included 🙂 ) came from ‘normal’ schooling backgrounds.

  184. OhMy Goodness Says:

    Ethan #180

    Volume in no way equates to quality and it seems as though your argument is based on the Marxist position that quantity is in fact the best quality. If you look at the most Nobel’s received by the graduates of an undergraduate institution per capita you find Ecole Normal Superior tops the list in the French system that you decry. If you consider the US universities that have produced the most Nobels per capita again you find universities that historically have strongly considered test results. Your apparent triumphal attitude about a high school in VA discontinuing consideration of test results so it can be the same as almost all the other high schools in the US suggests some strong personal attachment to conformism more than anything else. I am never sure about your comments about race. Recent genetic research has established that there are biogeographical genetic markers indicative of those with African or Asian or North European or South European ancestry so whether you want to call it race or not there are populations that have different characteristics. I certainly don’t see this as negative in any way nor a suggestion that all shouldn’t be equal under the law.

    anon #156

    I think Christian Evangelicals have been replaced by Turing Evangelicals in the CS community. The results of measurement of a quantum observable is not deterministically computable prior to the measurement. It is finite but not deterministically computable. Similarly an agent that enjoys free choice is not deterministically computable.

    It is well established that messy biological systems use quantum mechanics including olfaction and positional determination vis a vis the Earth’s magnetic field. I don’t understand the widespread and certain opposition here to human cognition including some quantum process. It makes more sense to me than the self referential language games or the denial of the existence of free choice. As an example, who could possibly compute that a politician in office for 47 years and achieved none of his stated objectives could receive such widespread support for his election to president-but there you have it that human behavior is often illogical. 🙂

    I agree with Ethan on this and am confident no ghost in the machine by the date he states no matter how strong the hopes and dreams are by some that quantum mechanics is ultimately found to be really just classical but previously misunderstood physics.

  185. Filip Dimitrovski Says:

    Ethan: I try to read all comments before replying, so the discrepancy was obvious, and I wrote some code for fun and practice. My comment was a tease at most, sorry if I offended you.

    Anyway, since you mentioned my country, in Macedonia the education situation is as follows:
    Most public high schools are “gymnasiums” where all subjects are studied. For example, everybody has to study physics, which starts with statics, dynamics, kinematics, waves and optics, thermodynamics up to the photoelectric effect and quanta. A huge proportion of the pupils come with terrible math skills from elementary school (6-14yr), so most of the exercises are known beforehand and kids learn them by heart and just swap the variables. You have to pick a “curriculum” which contains additional two subjects. If you pick natural sciences & math, you get linear algebra, programming, analysis, physics etc. If you pick humanities, you get psychology and sociology.
    There’s no AP or SAT. However, in order to graduate you need to pass a public exam (matura) and the results are used for getting into college.
    Here’s the catch: if you take a look at international olympiads, most participants from our country didn’t go to public schools. Even though the curriculum is good and the teachers are often good, it’s not a good environment for competitive kids. Most of them go to a single private high school in the country that’s known for its science team awards. There, they get education and peers that suit them as well as coaches for math/physics/CS competitions. The pattern is as follows: you are gifted and you go to this private high school -> they coach you and let you focus on what you are good at -> you get good IMO/IOI results -> you get accepted to MIT/Harvard and move out this country (a poor, corrupt one if I may say).

  186. OhMyGoodness Says:

    Filip #185

    As I remember there were student demonstrations in Czechia last year against including any mathematics at all in the matura and they were successful in having math removed. Math is insufficiently egalitarian for the neo-masses I guess.

  187. Nilima Nigam Says:

    Ethan #181
    I accept the issues are incredibly complex, and that of slavery in the US is quite outside my expertise. As with any such issue, I try to approach the issue of moral ‘sorting’ with humility. Unravelling these issues, and deciding how to dole out justice, requires humility and a willingness to accept good intentions are not enough – one needs to ask pointed questions, and address very narrowly-specified goals. All else is redolent of hubris. A good day is when I can help an individual. Anytime I feel the urge to ‘fix’ something for more than one individual, complex, fascinating, likely infuriating person – it’s time to take a deep breath.

    Having lived in Canada during the Truth and Reconciliation Commission (2008-2015), I’ve become very curious about how the native peoples of North America have been treated.

    Here are some interesting facts about Native Americans in the USA.

    – up until 1973, your country ran a network of residential schools. The intent was, I’m sure, noble in conception: to bring education to the native population, and teach them how to succeed in American society. The actual system perpetrated what would be deemed crimes against humanity – kids were taken from families and put in this network. Kids were frequently subjected to really awful conditions, including abuse. Not to mention what happens to children forcibly taken from their families, culture and language for years.

    Laws ‘allowing’ native parents to have the ‘right’ to not send their kids to these schools were not enacted until 1978. I cannot think of any parallel legislation in the USA affecting any other minorities in this way.

    – For at least 2 decades – through the ’60s and ’70s, Native American women would be sterilized by the Indian Health Service, very frequently without their consent. Native American organizations estimate between 25% -40% of native women of child-bearing age were sterilized during this period. Forced sterilization was, of course, not a favor restricted to native women – Black women were also sterilized without consent. However, what’s different here is that a population which was actively hunted down to near-extinction was, within my lifetime, further constrained from growing their numbers. Again, I understand the intent was noble- to shield kids from growing up in poverty. Or something.

    – Native populations are also overrepresented- by shocking amounts – in the incarceration system. Their school systems continue, on average, to be underfunded. Many native communities lack basic amenities- clean water, hospitals. Lands they’ve not ceded, or are theirs by treaty, continue to be considered the de facto property of the USA (see, for instance, pipeline disputes). The noble intent here is: they’re isolated, and could be much better off moving to urbanized environments.

    All of this to say: there’s great darkness in the history of the USA (and pretty much any other country on the planet). Some of this darkness was brought about by well-meaning reformers of the day trying to do noble things on average, rather than looking at the individual. We’d do well to learn caution when ‘fixing’ problems in broad strokes.

  188. Ethan Says:

    Ashley Lopez #183

    I think that as it pertains to the main topic of this thread this is the line that divides the different people who have expressed their opinions:

    ” I still do hold there is something wrong if all you learn is “Lisp or the surreal number system” at school. OTOH most stars in science and math (Scott included ???? ) came from ‘normal’ schooling backgrounds.”

    Based on results alone I don’t think it is possible not to argue that on any given year, people are born that will end up becoming stars in their respective professional fields and others that will lead respectable but mediocre careers. And this is irrespective of the argument “nature vs nurture” as to what causes these people to end up becoming stars.

    There are two questions that usually get conflated in conversations like this:

    1- Is it possible to identify these people early in life in a non invasive way that might result in “early sterilization” due to the additional celebrity that comes from said identification? This is where the issue of “is there an exam/test that allows for the identification of these people” comes into play.

    2- What’s the best way to nurture exceptional talent.

    I personally believe that in most cases the answer to 1- is no even though you can find anecdotal evidence to support the other point of view. I base this conclusion on studying the life of geniuses and extraordinary achievers whose contributions have stood the test of time as measured in decades (people like Feynman) and longitudinal studies like Lewis Terman’s. Terry Tao comes to mind as the most obvious example brought by those who support the opposite view, but even in that case, if his father had been like Ramanujan’s maybe he would not have been identified at all because classrooms, specially public school classrooms, have people who come from all sorts of socioeconomic backgrounds so it is not as simple as “here is a test to see who’s the star”. This is not to say that there isn’t value in administering these tests but doing well in them is, statistically speaking, more indicative of high socioeconomic background and conscientiousness than of genius/giftedness. BTW, this statement is compatible with the fact that most employers love people who exhibit conscientiousness which is why they like to hire college graduates.

    Then there is 2-. And I don’t think that society has figured out a way to do this properly. You would think that the children of parents who are high achievers themselves would benefit from their lessons and while no question having high achieving parents is correlated with metrics like high scores in standardized testing, educational achievement and income, it is also the fact that high schools that cater to demographics like these are also known for things like these https://www.theatlantic.com/magazine/archive/2015/12/the-silicon-valley-suicides/413140/ or this http://mdpsychfoundation.org/student-suicide-rate-at-mit-appears-to-be-higher-than-us-average/ . In Terry Tao’s case, his family -with extremely good judgement in my opinion- chose to send him to Flinders University where he could be near his parents in his young age before being sent to Princeton to learn from the best mathematicians in the world. Again, I can see Terry Tao’s father hand behind all this more than any particular choices by society. This is one reason why I said in a previous comment that I am not convinced that going to undergraduate to an elite institution is a good idea for everyone, particularly those who don’t know the world of privilege first hand.

    So if you create a school that advertises itself “this is a great STEM school and the access mechanism is taking an exam”, the inevitable result is going to be the aforementioned test prep arms race, specially as the school increases in prestige. You might get the “surprise factor” right when it comes to attracting students genuinely interested in STEM in the first classes, but once the school brand is known, you’ll get for the most part “prestige chasers” who know how to game the system rather than the Feynmans of the world.

    If you add the “public money” dimension, you have a very complicated situation.

  189. Ethan Says:

    OhMy Goodness #184

    You know nothing about me, beyond a few comments I have made here, so you are imputing me views I do not hold. I don’t decry the French system. What I am saying is that it is not an egalitarian system when it comes to access. That’s an assertion that is compatible with ENS being a Fields Medal factory and with the French elite having convinced the public to fund the institutions they send their kids to. Similarly, how do you know what I think about the US system?

    So there is no ambiguity about my positions: I believe that the ideology of “early identification” + “focusing society’s resources on those identified early” is evil and a current day remnant of the eugenic ideal of “fit vs unfit”.

    At the same time, I believe that the American higher education system, as expensive and as denounced as it is as inefficient by some with particular political agendas, is a treasure because it allows for both “access” and “excellence” in the way the French system doesn’t. It doesn’t mean it is perfect, but this notion that except for “exam high schools” that identify promising talent our country is doomed as a bastion of intellectual excellence is a talking point that has no basis on reality. And as with any talking point, you have to ask who benefits from perpetuating the myth. One group that clearly benefits from perpetuating the myth is white supremacists or in general people who believe that Black Americans are inherently inferior to other Americans. I personally see the support to lawsuits against Harvard and Yale among some white Americans who allegedly support Asian Americans as a proxy to white supremacy.

    And to be crystal clear, I am not saying that Scott is part of this group of white supremacists -obviously, he isn’t. But it’s too naive not to see certain forces at work, especially given the US centuries long history of closeted racists who have used all sorts of excuses to perpetuate the superiority of whites over Blacks.

  190. Ethan Says:

    Filip Dimitrovski #185

    I didn’t know the specifics of North Macedonia but I have definitely met people in person from Eastern Europe (and Iran) who follow this pattern,

    “: you are gifted and you go to this private high school -> they coach you and let you focus on what you are good at -> you get good IMO/IOI results -> you get accepted to MIT/Harvard and move out this country (a poor, corrupt one if I may say).”

    And obviously, the US is blessed to have all these people come here to contribute to our economic prosperity. At the same time, don’t you think it would be better for Macedonia (and in fact Iran) for those societies to be less corrupt and more egalitarian when it comes to access and to nourishing their local talent? As they stand now, those societies export their best talent which, in most cases, never comes back.

    So I think that all the examples provided seem to support the notion that a system like the American system that includes expanded access and reward of excellence irrespective of where you are in your personal life journey is the best of all possible scenarios and that’s why so many people want to come here during the most productive parts of their lives.

    I certainly don’t find a lot of empirical support to the idea that if you are born in the US and you are gifted you need schools like TJHS to develop your full potential. By being born in the US, you are already very far ahead with respect to other gifted people who must first find their way here following the aforementioned pattern.

  191. ira Says:

    Filip #185

    But Skopje probably has the best Roma music in the world, no ?

  192. Ethan Says:

    Nilima Nigam #187

    I must confess that I know less about the details of the oppression to Native Americans. I have learned quite a few things from reading your post.

    One thing I agree 100% is this,

    “All of this to say: there’s great darkness in the history of the USA (and pretty much any other country on the planet). Some of this darkness was brought about by well-meaning reformers of the day trying to do noble things on average, rather than looking at the individual. We’d do well to learn caution when ‘fixing’ problems in broad strokes.”

    All countries have their demons and the United States is no different. I have however two observations about this:

    – It seems to me that the demons of the United States tend to be “transversal” in society along the lines of attributes like “race” whereas in other countries that I know better in Europe, their demons tend to be geographically located. This is not a minor point because I think that while all humans are tribal, not all forms of tribalism are equally corrosive. Mobility, a genuine American trait when compared with other countries, has been a very effective tool in diluting tribalism.

    – Related to the first point: in the US there has been rarely anyone tribalism that has been unanimously or near unanimously accepted by society and changes to end bad practices have always come from some Americans unhappy with what they perceived were injustices perpetrated by other Americans. Speaking of the USA as a whole being unjust to this or that group of people is inaccurate. It’s the US and state governments taht have committed abuses, in most cases to the disgust of large portions of the population. As I said, I am less familiar with the suffering of Native Americans but even in the case of Black Americans, slavery -despite the 3-5ths compromise- was decried by several of the Founding Fathers. The second US president, John Adams, was both on the anti-slavery camp but pragmatic enough to understand that it was a hot issue in the context of national politics. Ditto of Alexander Hamilton. This dynamism is responsible for the abolition of slavery that followed the civil war and for the civil rights legislation that was passed in the second half of the 20th century. An overwhelming majority of whites unhappy with the status quo brought about the changes on both cases. I must say that this dynamic continues to this day. You speak of Native American genocide but some say that there is currently an ongoing African American genocide due to America’s liberal abortion laws https://www.nytimes.com/2019/07/06/us/black-abortion-missouri.html “Those seeking to outlaw abortion lament what they see as an undoing of the fabric of black families. They liken the high abortion rates among black women to a cultural genocide, and sometimes raise the specter of eugenics and population control when discussing abortion rights, as Justice Clarence Thomas of the Supreme Court did in a recent concurring opinion.”.

    The one thing that I can say about the United States that is different from other countries I am familiar with is that the relationship between the people with their government is one of deep distrust. This “distrust of government” runs high among Americans of different political persuasions. Here one has to be very careful with not imputing to “all Americans” or even to a majority of Americans the evil actions by “the federal government” or “some who work in the government”. The most meaningful criticism I have read about bad policy ideas pushed by government entities in the United States usually come from Americans disagreeing with those policies, not from foreigners trying to make sense of those injustices.

  193. Raoul Ohio Says:

    Ira #191:

    I’m a little weak on Roma music. Any youtube suggestions?

  194. Ashley Lopez Says:

    Ethan #167,

    The caste issues in India are, you can say, tremendously non uniform. There are locations in the country where things happen that I don’t even want to speak about, and others where it is just the opposite. I would tend to say mostly it is the latter, but that would be based on my personal experiences.

    And, regarding a more egalitarian society, that had to be an end in itself right? I mean, that should be so irrespective of the effects of it.

  195. Filip Dimitrovski Says:

    Ethan #190

    And obviously, the US is blessed to have all these people come here to contribute to our economic prosperity. At the same time, don’t you think it would be better for Macedonia (and in fact Iran) for those societies to be less corrupt and more egalitarian when it comes to access and to nourishing their local talent?

    I’d say my country “exports” at leasts 2 extremely talented math/CS high schoolers to USA annually. For some reason IMO and IOI olympiad medals are the only signals I’m aware of for young students to get noticed by US colleges.

    Our country put some effort this year into making a math-specific public high school with a (hard) entry exam so that they can attract the students that went to the private school. Other neighbouring countries are much better at this, but the export will always happen. There was some survey for Macedonian gifted kids and their willingness to leave abroad and it was around 50%, which I think is way too optimistic for our country considering the differences in opportunities abroad, I’d put it at 80%.

    The corruption I talked about is not education related, it comes from high level govt and I don’t think they know what math talent is, so no, the state cannot be “less corrupt when it comes to […] nourishing local talent”.

  196. Ethan Says:

    OhMy Goodness #184

    And to supplement what I said in my comment #189. During the Cold War, the expression https://en.wikipedia.org/wiki/Useful_idiot was coined to refer to those in the West who were manipulated by Communist propaganda because there was an intersection between legitimate interests of the manipulated by the propaganda and the Communist forces of the Soviet Union that sought to topple democracies worldwide to replace them with Communist regimes. Typically “useful idiots” didn’t grasp the full picture of what was going on, thus the expression.

    As it has happened with every generation of immigrants, most Asian immigrants have the noble goal of providing their children with the best education they can afford and obviously would rather have the public pay for it than having to pay for said education themselves. This is an aspect that is conveniently manipulated by the forces in American society interested in perpetuating the myth of white supremacy. I see them at work more in the case of the Harvard and Yale lawsuits that in the case of TJHS because in the US where you go to high school matters less in the grand scheme of things than in countries like France as opposed to gaining a spot in Harvard or Yale (check for example where the majority of US Supreme Court justices have gone to school as undergraduates or for law school).

  197. Ethan Says:

    Filip Dimitrovski #195

    Interesting! If history is any guide, the export of talent en masse -there will always be individual cases of people who want to give it a try at living in the US- won’t stop until the standards of living increase significantly in Macedonia and the other countries that send their best talent to the US. That was the experience with previous waves of immigration to the United States. If you look at the number of greencards per country of origin https://www.statista.com/statistics/200037/main-countries-of-last-residence-for-us-green-card-recipients/ , the ranking is dominated by Mexico, Cuba, China, India and the Dominican Republic. I can make convincingly the case that the main reason those are the leading countries today is lack of opportunity over there when compared with the US. You don’t see in the list anymore Ireland, Italy or Germany, all countries that used to send large numbers of immigrants to the US until around the 1960s. The economic prosperity in Europe that followed the end of WWII, I think, had a lot to do with that!

  198. Ethan Says:

    Ashley Lopez #194

    “And, regarding a more egalitarian society, that had to be an end in itself right? I mean, that should be so irrespective of the effects of it.”

    To me this means equality of opportunity irrespective of one’s socioeconomic background. This is very hard to do and I don’t know of any society, besides the United States, that offers said opportunities to the same degree. I say this understanding that the American deal today is probably not as good as it used to be a few decades ago.

    Offering equal opportunity is inherently inefficient and expensive because it begins by acknowledging that different people have different interests -thus central planning approaches don’t work very well- and that great talent comes in all shapes and forms, not only from families who spend a fortune coaching their kids to do well in standardized tests. Talent searches that focus on offering standardized tests to kids early in their lives are antithetical tot this philosophy which is why in the US you have both programs like Johns Hopkins’ Center for Talented Youth as well as the staunchest critics of said searches. Cases like Craig Venter’s require both institutional support in the form of the community college system and the open mindedness of America’s best universities to have several tracks of admission besides the traditional undergraduate admission system at the age of 18. It also requires a society that values achievement and “where you are going” more than “where you came from”. Putting all these things together is very hard and bottom up. Countries with top down approaches like France are good at producing excellence in narrow areas -such as the aforementioned production of Fields Medals- but they do not offer the breath of “equality of opportunity” at large irrespective of one’s chosen area of interest.

    Also, “equality of opportunity” is not a binary thing, it’s more like a continuum. In the United States there are states that do this better than others at things are very different at the high school level, the college level and the professional level irrespective of whether your profession requires a college degree. In the United States it is often said that states are laboratories of democracy with reference to different policy approaches to government issues. I think that the same could be said on economic matters: different states have adopted different approaches and not two states are equal in every aspect. Filip Dimitrovski spoke of people from his country going to MIT/Harvard. When it comes to access to quality public education, Massachusetts has been the state leading the rankings for many years https://www.usnews.com/news/best-states/rankings/education . It also has a high concentration of top schools besides MIT and Harvard. Check https://www.usnews.com/best-colleges/ma . And yet, if after graduation you want to “make it big”, Massachusetts is mostly known these days for biotechnology. In the late 1970s and during the 1980s the East Coast was also the center of high tech, but now that center is located in the West Coast (fundamentally Silicon Valley, and to a lesser degree Seattle). If want to “make it” in finance you go to New York City. If you want to “make it” in energy, you go to Texas. If your thing is medicine, then there are good medical schools and hospitals all around the place.

    In the United States, this geographical diversity of interests has been a great asset overall.

    The USA’s idiosyncrasies when it comes to rewarding great talent irrespective of how it shows up is why I say that people who are born in the United States have already won the biggest lottery there is on the planet. It’s up to them to figure out how to make a best use of the different opportunities available. In my professional life I have met people who have had all sorts of experiences, from having a traditional educational background to people who went to community college later in their life and ended up transferring to a good school from where they got their undergraduate degrees in their 30s. And as I said, I got a PhD degree in a STEM field from one of America’s top academic powerhouses in my 30s. So this is a feature of the United States’ that I have personally benefited from. I see a focus on “early selection” and “focus of resources” on those identified early -particularly if it is done with public money with the intent of optimizing resources- as a threat to this openness that will end up being bad for the United States in the long run.

  199. Ethan Says:

    OhMy Goodness #184

    Another way to frame what I am saying is under Peter Thiel’s framework “zero to one” . Check https://www.amazon.com/Zero-One-Notes-Startups-Future/dp/0804139296 and https://www.youtube.com/watch?v=SO_00POR-Po .

    His basic theory is that exceptional startups -of the kind that transform society such as Microsoft, Apple or Google- as well as the founders that produce them happen exactly once in history because the right combination of the talent of the founders as well as the societal combination that produced them together with the necessary conditions for their creations to be successful happen only once in history. Microsoft didn’t produce the first operating system, Apple didn’t develop the first personal computer and Google didn’t produce the first search engine. All three produced what later became known as “the final word” in their respective product categories. In the case of Apple, it had a second life, when Steve Jobs returned to the company, producing “the final word” when it came to music players, smart phones and tablets.

    Applying this framework to the issue of identifying top talent, my contention is that:

    – The next Einstein won’t be a Jew living in Germany

    – The next Turing won’t be a gay man living in England

    – The next John von Neumann won’t be a Hungarian immigrant in the US

    – The next Feynman won’t be a Jew from Queens

    It is tempting to try to use induction to guess “the formula” that produced them as ground truth to try to create environments that will produce the next generation of “geniuses”. Studying the life of geniuses that most that you can say is that they happen to show up in great civilizations: Einstein in Germany at the beginning of the 20th century, Turing in England in the period between the two world wars, von Neumann & Feynman in the US during the era known as “Big Science”. When it comes to great startups, there are a few specialized clusters that produce them -mostly in the US like the aforementioned Silicon Valley, Seattle and Boston-; attempts at replicating these elsewhere in the world have been largely failures. For example it seems unlikely to me that China will ever compete with the US in producing breakthrough technologies due to their political system that emphasizes stability and compliance over risk taking and extreme creativity.

    Given this reality, thinking in terms of “early identification” + “focusing resources on those identified early” gives you at most the French system of mathematics or the Soviet system of the same. I love mathematics but great talent appears in many areas outside mathematics. It seems to me that the empirical evidence indicates that the best way to foster the development of exceptional talent is to create an ecosystem like the one we have in higher education in the Untied States and that it is impossible to identify with a high degree of specificity where that exceptional talent will show up. Take for example Jack Dorsey, co-founder of Twitter and Square. He went to https://en.wikipedia.org/wiki/Bishop_DuBourg_High_School . While the school has produced some other famous people, it’s the most unlikely place for anyone to try to find the next important technology entrepreneur. What Jack Dorsey had going for him is the existence of Silicon Valley in the United States and he was able to find his way there. Had someone with exactly his same natural talent been born in some remote place in the UK, it is very unlikely that he would have produced these companies.

  200. OhMyGoodness Says:

    Ethan #196

    You stated above that I know nothing about you but that isn’t true in so far as your writings here provide some information about you. Based on your writings it seems to me that race is an extremely important characteristic for you since your writings here so often focus on race. I have always found it a logical puzzle that people who see things in racial terms find others so frequently racists. In the broadest sense isn’t that the essence of a racist-one whose world view is strongly influenced by race?

    I find the argument very strange that white supremacists are championing the cause of Asians. Does that imply they consider Asians white and only African Americans non-white? A fundamental principle of the USA is that all are equal under the law. What test do you use to determine if someone is a moral proponent of equal under the law without regard for race etc or an immoral white supremacist?

    I noted previously that my preference would be to blind educational entrance requirements with respect to race, religion, sex, and family name. This is fully consistent with full equality for all and I can’t imagine the logical contortions required to class this as white supremacy.

  201. fred Says:

    Scott on the Lex Fridman podcast!

  202. Ethan Says:

    OhMyGoodness #200

    Indeed you know nothing about me. My writings reflect very narrowly where I stand on several issues. This notion that I care about race issues for example. Of course I care, but not in the sense you imply. It’s America’s original sin and conversations about race continue to divide the country several hundred years after its founding! Unless you live in a bubble, it’s impossible not to care about race issues if you care about the United States, particularly its future and how to build a more egalitarian one aligned with its founding vision.

    There are people alive today that until 1964 were forced to go to separate bathrooms for being Black. I know some of these people in my close circle without being Black myself. Nilima Nigam brought to this conversation evidence of mistreatment of Native Americans. Princeton recently decided to remove the name of one of its most prominent alums, Woodrow Wilson, as an acknowledgment of its racist views. To believe, as you seem to, that all these tensions play no role in conversations about admissions to America’s most prestigious schools is a little bit naive if you were to ask me.

    With respect to your own writings, I could say you seem obsessed in cherry picking data that continues to perpetrate racist ideas linking race to intelligence by people like William Shockley or James Watson that have been thoroughly debunked by recent advances in genetics. It seems to me that you are projecting your own narrow “Charles Murray”-centric view of the debate around college admissions and talent identification on my writings. Based on how you write about France’s ENS for example, it’s pretty evident you know very little about the idiosyncrasies and nuances of France’s higher education system. That France’s own president is proposing the abolition of one of their most prestigious and allegedly meritocratic schools should give you an indication that things are not as simple over there as you seem to think about them. Ditto of Laurent Lafforgue’s criticism of the French education system at large using the profile of students currently enrolled at ENS as part of that criticism.

    It’s perfectly human to try to pass one’s own privilege to the next generation. In fact, the largest Christian denomination there is, Roman Catholicism, instituted celibacy among its clergy not for theological reasons but precisely to stop nepotistic abuses in the medieval church. To say that a system that ignores these issues and focuses exclusively on a test score irrespective of all other societal considerations of the applicant is equivalent to “merit” is either tone deaf or malicious.

    With respect to the lawsuits against Harvard and Yale, it doesn’t take a genius -only having studied American history in detail- to see elements (again “elements”, I don’t say it’s the only motivation behind the lawsuits; clearly the Asian Americans supporting them have a legitimate interest in forcing these schools to increase their share of admitted Asian Americans) of white supremacy in the Yale lawsuit. Current Yale students, including students of Asian heritage, are not fooled by what is going on https://yaledailynews.com/blog/2020/10/09/students-criticize-doj-admissions-lawsuit-against-yale/

    “Several weeks ago, student organizations affiliated with the Asian American community at Yale — including the Asian American Students Alliance and Asian Students for Ethnic Studies — signed a statement expressing concern over the DOJ allegation that preceded Thursday’s lawsuit.

    These 23 organizations agreed that the DOJ’s allegation is “a thinly veiled ploy to use Asian Americans to attack the system of affirmative action and maintain unequal access to higher education, at the expense of Black, Indigenous and Latinx individuals.”

    They characterized the lawsuit as “white supremacy under the guise of meritocracy,” explaining that “race-neutral” admissions cannot exist in a country that is “anything but.” They condemned the DOJ for treating the Asian American community as a monolith and claiming that affirmative action benefits many Asian American students.”

    So!

  203. OhMyGoodness Says:

    Ethan #202

    Please show one post where I linked race with intelligence. Surprised that your evidential standard includes a petition from Asian Students For Ethnic Studies. One of your favorite whipping boys (Shockley) was unhinged with paranoia and alienated from everyone including his own children at death so an exceedingly easy target to include. I believe you depend heavily by besting straw horses So! yes nothing fruitful to come from continuing this dialogue.

  204. Ethan Says:

    “With respect to your own writings, I could say you seem obsessed in cherry picking data that continues to perpetrate racist ideas linking race to intelligence by people like William Shockley or James Watson that have been thoroughly debunked by recent advances in genetics.”

    The spell checker failed me here. What I wanted to say is,

    “With respect to your own writings, I could say you seem obsessed in cherry picking data that continues to PERPETUATE racist ideas linking race to intelligence by people like William Shockley or James Watson that have been thoroughly debunked by recent advances in genetics.”

    Using proxies to avoid talking directly about controversial issues themselves is a very old tactic in politics. In the case that concerns us, expressing deep concern for Asian Americans (or immigrants or Asian descent) who want to get into America’s best and most prestigious universities as a proxy for other considerations. As I said in my previous comment #202, Yale students themselves of Asian descent are not fooled.

    Whether your intent is to perpetuate these ideas is hard to say, but I see a lot of correlation between your arguments and the discredited arguments by Charles Murray -some of which he has distanced himself from in recent years https://twitter.com/charlesmurray/status/1129547945123074049?lang=en .

    I think that the lesson from the statue toppling we are seeing these days is that while it is true that history is written by the winners, the descendants of the losers have long memories. If you had told 100 years ago Woodrow Wilson that Princeton would be so disgusted with his racist views that they would be disowning his name, he would probably have not believed it.

    To me the message is clear: if you care about your legacy, be fair minded. If in the long run, your friends won’t be there to protect you or your name if have made a mark maligning entire classes of people.

  205. Ethan Says:

    OhMyGoodness #203

    “yes nothing fruitful to come from continuing this dialogue.”

    I agree with this!

    Then to your question,

    “Please show one post where I linked race with intelligence. ”

    Whether you are doing this willingly or unwillingly, here comes an example,

    “I noted previously that my preference would be to blind educational entrance requirements with respect to race, religion, sex, and family name. This is fully consistent with full equality for all and I can’t imagine the logical contortions required to class this as white supremacy.”

    I go back to the issue of using proxies (or “coded language” if you prefer) to avoid addressing points directly.

    Speaking of “equality under the law” in the abstract is like being for “world peace”. It’s totally meaningless. Who isn’t for world peace? On the issue of race, we have one state, California, that passed an initiative under the the name of “equality” designed to decrease the enrollment of minorities in California’s most prestigious campuses https://en.wikipedia.org/wiki/1996_California_Proposition_209 . The notion that things exist “in the abstract” without consideration to the context where they happen is for 8th grade civics classes. In the real world, people are rarely going to openly advocate for unfair policies, particularly on controversial issues. They always use proxies.

    If your intention wasn’t to link race with intelligence, you surely sound like those who do, advancing similar “Charles Murray”-esque arguments. And note that your question is about me providing evidence to support my contentions.

    I ask you to explicitly state your position on links, if any, between race and intelligence as to avoid any misunderstandings on my behalf or other readers who might have gotten the wrong impression. If you think that the two issues aren’t intrinsically related, you can state say it in plain language. If you think that the issue is nuanced, then you can also state your nuances. The readers of this blog tend to be smart people who understand nuance.

    I still recall the conversation where I gave the example of Nigerians who are successful in the US as evidence that race, intrinsically, has nothing to do with intelligence and that the most likely explanation for the underachievement of US born Blacks was the centuries old history of discrimination against them. You went around deviating the conversation towards family values or something.

    I hope you step forward to clarify your position.

  206. OhMyGoodness Says:

    Ethan #204

    Not much time because embroiled in a family disagreement right now. One of my 8 year old fraternal twin daughters (the one that parses every sentence for logical weakness) has requested an explanation as to why her bedroom is 46 steps from her parents bedroom while her sisters is only 42 steps. I answered that the time difference is negligible but she doesn’t accept that as an acceptable answer. She suspects some kind of favoritism.

    As I have stated clearly before I don’t believe race provides a priori indication of a persons intelligence and hope this reiteration satisfies you that I mean no one ill will of any kind because of race.

  207. Daniel Says:

    @ Ethan,

    I understand OhMyGoodness just fine and I don’t think anything he said (including what you cite) links race and intelligence unless you are dead set on finding it a priori. I also think you sound like twat.

  208. Scott Says:

    This thread has … err … reached the end of its useful life, 🙂 so get in any final comments and then I’ll close it down tonight.

  209. Ethan Says:

    OhMyGoodness #206

    “As I have stated clearly before I don’t believe race provides a priori indication of a persons intelligence and hope this reiteration satisfies you that I mean no one ill will of any kind because of race.”

    Great! Now, here is my follow up.

    Assume that someone scores 1600 in the SAT, which of the following three is more likely in your opinion:

    1- Someone coached for the SAT with what has been called “the test prep arms race”

    2- Someone grew up in an environment (according to the College Board’s own data for example upper middle class) that values the kind of skills measured by the SAT

    3- Someone has an “intrinsic/genetic” talent that makes this person score 1600 in the SAT without coaching or without the inherent advantage of growing up in an environment that values the kind of skills measured by the SAT

    I see 1- or 2- the most likely possibilities and 3- as the least likely possibility, which begs the question of why is that “scoring high on the SAT or a similarly standardized exam” became associated with “merit”. The main merit that I see on both 1- and -2 is that of the parents of the test taker, not of the test taker himself or herself.

    The SAT statistics of those admitted to the nation’s most prestigious schools also say that “Asians” on average score higher than “whites” who themselves score higher than “Blacks”. If we agree that race, a priori, is not related to intelligence, what do you attribute the differences among the different groups? And if one reason Blacks score the least among the three groups is the nation’s history of institutional discrimination towards Blacks that lasts to this day, don’t you think that when it comes to educating tomorrow’s leaders -which is the stated reason America’s best universities consider socioeconomic background as part of their holistic admission system- relying exclusively (or even mainly) on SAT scores or other types of standardized testing is a disservice to the nation?

    Now, I can understand the rationale that because someone who belongs to group 1- or 2- will in all likelihood display a high degree of conscientiousness and very likely “good manners” and that’s preferable to someone with a different socioeconomic background, but I find it hard to accept rationales of the kind “relying exclusively or primarily on SAT scores or other types of standardized tests for admissions” is equivalent to “meritocracy of the applicant”. Thus, you have to ask what’s the real motivation behind the lawsuits against Harvard or Yale that rely primarily on SAT scores of admitted students to make their case (in Harvard’s case, they got a full trial, even if Harvard ended up winning the first round).

    If you can comment before the thread is closed, I would love to hear your opinion.

  210. Arul Says:

    It would be nice if you can comment why PKC should be nominated for Nobel if it were the case that computer science could be squeezed in as a part of physics? PKC is not provably demonstrated yet. Perhaps are you talking Merkle’s puzzles which are I think provable?

  211. Anon2 Says:

    Ethan #147
    “knowledge aristocracy”? You don’t need to reinvent the wheel here, we already have a name for these people.

    “middle class”

    Your proposition basically comes down to weaken/destabilize the middle class, so it’s fair for the bottom class, which ironically sounds like a conspiracy from the upper class.

  212. Scott Says:

    Arul #210: Oh right, forgot to respond to that.

    Public-key crypto has already been honored twice (or three times if you count Goldwasser-Micali) with the Turing Award, which everyone regards as the “Nobel of CS.” So it seems clear that, if in the 1960s we’d gotten to name our new prize for Alfred Nobel like the economists did, then there’d by now be several “Nobels” for PKC.

    The case in favor is obvious: PKC presents an amazing combination of fundamental conceptual interest, surprise, mathematical nontriviality, and actual impact in transforming the way people live (without PKC, electronic commerce would look radically different if it existed at all).

    That it will all go away if it turns out that P=NP by an efficient algorithm, seems to me like an incredibly weak argument against. How many Nobel-Prizewinning discoveries in physics, medicine, economics might be rendered obsolete by some even-more-astounding future development? If (as I think) the answer is “dozens,” then why do people apply a standard to CS theory that they don’t apply anywhere else?

  213. Arul Says:

    “How many Nobel-Prizewinning discoveries in physics, medicine, economics might be rendered obsolete by some even-more-astounding future development? If (as I think) the answer is “dozens,” then why do people apply a standard to CS theory that they don’t apply anywhere else?”. I think somehow they might perceive CS as mathematics. Perhaps it is. The border between mathematics and CS is not very clear I think. That is my opinion though.

  214. Ethan Says:

    Anon2 #211

    I think it’s more the “upper middle class” who make this “knowledge aristocracy”. By this I mean the people top 20% people in terms of income, excluding the top 1%. The top 1% are too wealthy to care about these issues.

    Rather than “weaken/destabilize” them, a better description for my proposal is disruption. Capitalism works through the process known as creative destruction.

    If you think carefully about it, this is what this “aristocracy” did in the 1960s with the help of insiders: they disrupted the world of privilege of the old WASP elite. 50+ years in power is too many years. They deserve to be disrupted themselves for the good of the United States, namely, to make the future leaders of the United States more representative of the country as a whole. That’s at least I the way I would put it :-). Any country whose elite is not representative of the underlying society ends up being disrupted through political instability. That’s, in my opinion, what the WASP elite understood, paving the way of the knowledge aristocracy born in the 1960s.

  215. Ascent Says:

    @ Ethan

    “If we agree that race, a priori, is not related to intelligence, what do you attribute the differences among the different groups?”

    Being statistically over-represented in committing violent crime. Having higher rates of teen pregnancy. And the one you even mentioned: a high rate of single parenting. On top of this, a culture that normalizes this behavior.

    Going back to the single parenting issue you acknowledged, we could start with addressing that aspect of the problem. Why not provide additional tax dollars to intervene in these households and make sure the children there are not neglected or put at a disadvantage? Why instead do you jump all the way downstream with efforts to lower standards at the high school or university level under the guise of fairness?

    Employers are not completely stupid. If you what you propose succeeds, then employers will value an applicant of Asian descent who went to, say, Harvard far more than a black applicant with the same educational pedigree. The first had to overcome racist policies designed to keep them out of such elite colleges, while the second was accepted using much lower standards. This is exactly what happens in parts of Canada where Native American law students fail their classes, get the failures changed to passes by a PC administration. They can’t find jobs in industry because everybody is in the know about this situation, so they end up working for tribes, which in turn causes damage because such graduates (not all of them, but many) are incompetent. And round and round.

    Finally, your characterization of Asians as useful idiots implies that Asians are just too dumb to be acting with any agency in their own best interests. Maybe if you are, say, Chinese yourself, and your US-born-but-Asian-looking kid gave up her spot at Yale or Harvard (or any of the top 20 schools) to a black student, then you’d be worth listening to. But I suspect you’re just virtue signaling, and like all virtue-signalers, you are offloading the costs (monetary and social) to a demographic you dislike and is currently an acceptable target of such racist attitudes.

  216. Ethan Says:

    Ascent #215

    What do you mean by “what I propose”? I see a pattern of people imputing me “proposals” I know nothing about. Just to be clear about what I propose: that top American schools are allowed to do holistic admissions as they see fit instead of being forced to admit their students based on some arbitrary standardized test like the SAT. If you think carefully about it, and you study the history of the SAT, choosing the SAT or IQ tests as standard bearers of “merit” is completely arbitrary. In the case of the IQ tests, Lewis Terman completely distorted the intent of Binet: it transformed the test from a reasonable measure of performance to identify students who might benefit from extra help to a tool predictive of biological capacity.

    From the rest of the stuff that you say, I kind of agree that a top priority should be improving the social situation of African Americans, but the two things go hand in hand.

    With respect to what I know or I don’t know, as I said, I got a PhD in a STEM field from a top American school. While I was a graduate student, I took some classes where there were undergraduates and I did an investigative labor to get a grasp about their upbringing. In every case where the undergraduate was of White or Asian heritage, the word “privilege” cames to mind. This said, these students were outstanding and they shouldn’t be penalized either. Nobody, that we know of, chooses his/her family and that applies also to those who were born in more favorable environments.

    All I am saying is that it is very naive to equate doing well in a standardized test, particularly one coachable like with intrinsic merit of the applicant, thus I salute America’s top schools for using the method of holistic admissions. The reality is that academic stars with perfect GPAs from good high schools and perfect SAT scores already have a much high probability of being admitted to one or more of America’s top schools than other students. At the same time, the reality is that every year, America’s top schools admit undergraduate students who are less than perfect on those two metrics that have something different to offer who end up becoming leaders in their fields of choice. So a different way of putting this is that these schools do know that where you are academically at the age of 18 is not a necessary predictor of your full potential and have an open minded, and inclusive, view of merit because they don’t want to miss on educating future leaders that have non traditional backgrounds.

    So I’ll stop here.

  217. Anon5 Says:

    Ethan, when you say “They deserve to be disrupted themselves for the good of the United States, namely, to make the future leaders of the United States more representative of the country as a whole” who is “they” and what is exactly does it mean to be “disrupted”?

    What you’ve posted (30+ posts according to Filip) is all so tidy but vague. Does “they” include “you”? If so, to what extent, will you give up your raise or job position to a person of color? Does it include your children, if you have any – are you willing to limit their educational potential? What exactly have you sacrificed? Because it’s hard to take any of what you say seriously when you leave out the painful details and exactly which people are going to feel that pain.

  218. Jer Says:

    Capitalism works through the process known as creative destruction.

    LOL! It also works through a process known as competition. Something which you apparently cannot abide.

  219. Ethan Says:

    “At the same time, the reality is that every year, America’s top schools admit undergraduate students who are less than perfect on those two metrics that have something different to offer who end up becoming leaders in their fields of choice”

    And to give the clearest example there is: Barack Obama, 44th president of the United States!

    You also have Sonia Sotomayor, US Supreme Court Justice https://en.wikipedia.org/wiki/Sonia_Sotomayor “Sotomayor entered Yale Law School in the fall of 1976, once more on a scholarship. While she believes she again benefited from affirmative action to compensate for somewhat lower standardized test scores, a former dean of admissions at Yale has said that given her record at Princeton, it probably had little effect. At Yale she fit in well although she found there were again few Latino students. She was known as a hard worker but she was not considered among the star students in her class.”

    But it doesn’t end there. I have attended talks of Nobel Prize winners in science who confessed not having been stellar students in undergraduate. One of them, whose name I prefer not to give because that would give away the name of the school I went to, said explicitly that he was an average graduate student known for taking the opposite views to what was the “mainstream view” and he was lucky that his prediction was right and verified empirically and that’s how he got his Nobel Prize.

    All this to say, that people must not forget that top American schools do holistic admissions instead of formulaic admissions because it works. It’s hard to argue with the results. I would ask the question differently: why is that there is so many people obsessed with destroying this system given the spectacular results it produces?